Health Assessment Jarvis Final Updated & Combined

Ace your homework & exams now with Quizwiz!

The nurse is incorporating a person's spiritual values into the health history. Which of these questions illustrates the "community" portion of the FICA (faith and belief, importance and influence, community, and addressing or applying in care) questions? a."Do you believe in God?" b."Are you a part of any religious or spiritual congregation?" c."Do you consider yourself to be a religious or spiritual person?" d."How does your religious faith influence the way you think about your health?"

B

The nurse is preparing to perform a physical assessment. Which statement is true about the physical assessment? The inspection phase: a. Usually yields little information. b. Takes time and reveals a surprising amount of information. c. May be somewhat uncomfortable for the expert practitioner. d. Requires a quick glance at the patients body systems before proceeding with palpation.

B

The nurse is reviewing concepts related to one's heritage and beliefs. The belief in divine or superhuman power(s) to be obeyed and worshipped as the creator(s) and ruler(s) of the universe is known as: a. Culture. b. Religion. c. Ethnicity. d. Spirituality.

B

The nurse knows that testing kinesthesia is a test of a person's: a. Fine touch. b. Position sense. c. Motor coordination. d. Perception of vibration.

B

The nurse will perform a palpated pressure before auscultating blood pressure. The reason for this is to: A) hear the Korotkoff sounds more clearly. B) detect the presence of an auscultatory gap. C) avoid missing a falsely elevated blood pressure. D) identify phase IV of the Korotkoff sounds more readily.

B

The nurse would use bimanual palpation technique in which situation? a. Palpating the thorax of an infant b. Palpating the kidneys and uterus c. Assessing pulsations and vibrations d. Assessing the presence of tenderness and pain

B

The two parts of the nervous system are the: a. Motor and sensory. b. Central and peripheral. c. Peripheral and autonomic. d. Hypothalamus and cerebral.

B

When assessing a patient's pain, the nurse knows that an example of visceral pain would be: A) hip fracture. B) cholecystitis. C) second-degree burns. D) pain after a leg amputation.

B

When assessing a patient, the nurse notes that the left femoral pulse as diminished, 1+/4+. What should the nurse do next? a. Document the finding. b. Auscultate the site for a bruit. c. Check for calf pain. d. Check capillary refill in the toes.

B

When assessing an older adult, the nurse keeps in mind that which vital sign changes occur with aging? A) Increase in pulse rate B) Widened pulse pressure C) Increase in body temperature D) Decrease in diastolic blood pressure

B

When performing a physical assessment, the first technique the nurse will always use is: a. Palpation. b. Inspection. c. Percussion. d. Auscultation.

B

During an assessment of an infant, the nurse notes that the fontanels are depressed and sunken. The nurse suspects which condition? a.Rickets b.Dehydration c.Mental retardation d.Increased intracranial pressure

B Depressed and sunken fontanels occur with dehydration or malnutrition. Mental retardation and rickets have no effect on the fontanels. Increased intracranial pressure would cause tense or bulging and possibly pulsating fontanels.

A female patient has denied any abuse when answering the Abuse Assessment Screen, but the nurse has noticed some other conditions that are associated with intimate partner violence. Examples of such conditions include:

ANS: depression. Depression is one of the conditions that is particularly associated with intimate partner violence. Abused women also have been found to have more chronic health problems, such as neurologic, gastrointestinal, and gynecologic symptoms, and chronic pain, as well as suicidality and posttraumatic stress disorder symptoms.

In performing an assessment on a 49-year-old woman who has imbalanced nutrition as a result of dysphagia, which data would the nurse expect to find? a. Increase in hair growth b. Inadequate nutrient food intake c. Weight 10% to 20% over ideal d. Sore, inflamed buccal cavity

B

In response to a question regarding the use of alcohol, a patient asks the nurse why the nurse needs to know. What is the reason for needing this information? a.This information is necessary to determine the patient's reliability. b.Alcohol can interact with all medications and can make some diseases worse. c.The nurse needs to be able to teach the patient about the dangers of alcohol use. d.This information is not necessary unless a drinking problem is obvious.

B

The area of the nervous system that is responsible for mediating reflexes is the: a. Medulla. b. Cerebellum. c. Spinal cord. d. Cerebral cortex.

C

The nurse has implemented several planned interventions to address the nursing diagnosis of acute pain. Which would be the next appropriate action? a. Establish priorities. b. Identify expected outcomes. c. Evaluate the individuals condition, and compare actual outcomes with expected outcomes. d. Interpret data, and then identify clusters of cues and make inferences.

C

The nurse is assessing a 30-year-old unemployed immigrant from an underdeveloped country who has been in the United States for 1 month. Which of these problems related to his nutritional status might the nurse expect to find? A) Obesity B) Hypotension C) Osteomalacia D) Coronary artery disease

C

The nurse is comparing the concepts of religion and spirituality. Which of the following is an appropriate component of one's spirituality? a. Belief in and the worship of God or gods b. Attendance at a specific church or place of worship c. Personal effort made to find purpose and meaning in life d. Being closely tied to one's ethnic background

C

The nurse is describing a weak, thready pulse on the documentation flow sheet. Which statement is correct? a. "Is easily palpable; pounds under the fingertips." b. "Has greater than normal force, then suddenly collapses." c. "Is hard to palpate, may fade in and out, and is easily obliterated by pressure." d. "Rhythm is regular, but force varies with alternating beats of large and small amplitude."

C

During a physical education class, a student is hit in the eye with the end of a baseball bat. When examined in the emergency department, the nurse notices the presence of blood in the anterior chamber of the eye. This finding indicates the presence of: a. Hypopyon. b. Hyphema. c. Corneal abrasion. d. Pterygium.

B. Hyphema. Hyphema is the term for blood in the anterior chamber and is a serious result of blunt trauma (a fist or a baseball) or spontaneous hemorrhage and may indicate scleral rupture or major intraocular trauma. (See Table 14-7 for descriptions of the other terms.)

When performing a peripheral vascular assessment on a patient, the nurse is unable to palpate the ulnar pulses. The patient's skin is warm and capillary refill time is normal. Next, the nurse should: a. Check for the presence of claudication. b. Refer the individual for further evaluation. c. Consider this finding as normal, and proceed with the peripheral vascular evaluation. d. Ask the patient if he or she has experienced any unusual cramping or tingling in the arm.

C

When using a Doppler ultrasonic stethoscope, the nurse recognizes venous flow when which sound is heard? a. Low humming sound b. Regular "lub, dub" pattern c. Swishing, whooshing sound d. Steady, even, flowing sound

C

Which of these conditions is due to an inadequate intake of both protein and calories? A) Obesity B) Bulimia C) Marasmus D) Kwashiorkor

C

Which of these conditions is due to an inadequate intake of both protein and calories? a. Obesity b. Bulimia c. Marasmus d. Kwashiorkor

C

The nurse is preparing to percuss the abdomen of a patient. The purpose of the percussion is to assess the __________ of the underlying tissue. a. Turgor b. Texture c. Density d. Consistency

C

The nurse is reviewing information about evidence-based practice (EBP). Which statement best reflects EBP? a. EBP relies on tradition for support of best practices. b. EBP is simply the use of best practice techniques for the treatment of patients. c. EBP emphasizes the use of best evidence with the clinicians experience. d. The patients own preferences are not important with EBP.

C

The nurse is reviewing the blood supply to the arm. The major artery supplying the arm is the _____ artery. a. Ulnar b. Radial c. Brachial d. Deep palmar

C

The nurse is taking a family history. Important diseases or problems about which the patient should be specifically asked include: a.Emphysema. b.Head trauma. c.Mental illness. d.Fractured bones.

C

The nurse is taking temperatures in a clinic with a tympanic thermometer. Which statement is true regarding use of the tympanic thermometer? A) A tympanic temperature is more time consuming than a rectal temperature. B) The tympanic method is more invasive and uncomfortable than the oral method. C) There is a reduced risk of cross-contamination compared with the rectal route. D) The tympanic membrane most accurately reflects the temperature in the ophthalmic artery.

C

The nurse is testing superficial reflexes on an adult patient. When stroking up the lateral side of the sole and across the ball of the foot, the nurse notices the plantar flexion of the toes. How should the nurse document this finding? a. Positive Babinski sign b. Plantar reflex abnormal c. Plantar reflex present d. Plantar reflex 2+ on a scale from "0 to 4+"

C

The nurse is assessing a patient in the hospital who has received numerous antibiotics and notices that his tongue appears to be black and hairy. In response to his concern, what would the nurse say? a."We will need to get a biopsy to determine the cause." b."This is an overgrowth of hair and will go away in a few days." c."Black, hairy tongue is a fungal infection caused by all the antibiotics you have received." d."This is probably caused by the same bacteria you had in your lungs."

C. black, hairy tongue is a fungal infection caused by all the antibiotics you have received

The nurse is assessing an 80-year-old patient. Which of these findings would be expected for this patient? a.Hypertrophy of the gums b.Increased production of saliva c.Decreased ability to identify odors d.Finer and less prominent nasal hair

C. decreased ability to identify odors

A 92-year-old patient has had a stroke. The right side of his face is drooping. The nurse might also suspect which of these assessment findings? a.Epistaxis b.Rhinorrhea c.Dysphagia d.Xerostomia

C. dysphagia

A 59-year-old patient tells the nurse that he has ulcerative colitis. He has been having "black stools" for the last 24 hours. How would the nurse best document his reason for seeking care? a.J.M. is a 59-year-old man seeking treatment for ulcerative colitis. b.J.M. came into the clinic complaining of having black stools for the past 24 hours. c.J.M. is a 59-year-old man who states that he has ulcerative colitis and wants it checked. d.J.M. is a 59-year-old man who states that he has been having "black stools" for the past 24 hours.

D

A 78-year-old man has a history of a cerebrovascular accident. The nurse notes that when he walks, his left arm is immobile against the body with flexion of the shoulder, elbow, wrist, and fingers and adduction of the shoulder. His left leg is stiff and extended and circumducts with each step. What type of gait disturbance is this individual experiencing? a. Scissors gait b. Cerebellar ataxia c. Parkinsonian gait d. Spastic hemiparesis

D

A patient is describing his symptoms to the nurse. Which of these statements reflects a description of the setting of his symptoms? a."It is a sharp, burning pain in my stomach." b."I also have the sweats and nausea when I feel this pain." c."I think this pain is telling me that something bad is wrong with me." d."This pain happens every time I sit down to use the computer."

D

A patient tells the nurse that he is allergic to penicillin. What would be the nurse's best response to this information? a."Are you allergic to any other drugs?" b."How often have you received penicillin?" c."I'll write your allergy on your chart so you won't receive any penicillin." d."Describe what happens to you when you take penicillin

D

A 22-year-old woman comes to the clinic because of a severe sunburn and states, "I was just out in the sun for a couple of minutes." The nurse begins a medication review with her, paying special attention to which medication class? a. Nonsteroidal antiinflammatory drugs for pain b. Tetracyclines for acne c. Proton pump inhibitors for heartburn d. Thyroid replacement hormone for hypothyroidism

Tetracyclines for acne Drugs that may increase sunlight sensitivity and give a burn response include sulfonamides, thiazide diuretics, oral hypoglycemic agents, and tetracycline.

The nurse is assessing voice sounds during a respiratory assessment. Which of these findings indicates a normal assessment? Select all that apply

Voice sounds are faint, muffled, and almost inaudible when the patient whispers "one, two, three" in a very soft voice When the patient speaks in a normal voice, the examiner can hear a sound but cannot distinguish exactly what is being said. As the patient says a long "ee-ee-ee" sound, the examiner also hears a long "ee-ee-ee" sound.

The nurse is listening to the breath sounds of a patient with severe asthma. Air passing through narrowed bronchioles would produce which of these adventitious soudns?

Wheezes

During an assessment of an adult, the nurse has noted unequal chest expansion and recongnizes that this occurs in which situation?

When part of the lung is obstructed or collapsed

During auscultation of the lungs, the nurse expects decreased breath sounds to be heard in which situation?

When the bronchial tree is obstructed

1. During a routine well-person checkup, the daughter of an older patient mentions to the nurse notices she has noticed her mother is less attentive and sometimes unable to recall events from a previous day. The daughter said this does not happen all the time, but that she has noticed it twice in the last month. Which test would be best to assess this patient's mental status? a. Timed Up and Go Test (TUG) b. Montreal Cognitive Assessment (MoCA) c. Mini-Mental State Examination (MMSE) d. Geriatric Depression Scale, short form

c

1. The nurse is conducting a hearing screening. Which technique will the nurse use during the whisper test? a. The nurse pulls the pinna up and back. b. The nurse covers their lips to obscure them from view. c. The nurse asks the patient to repeat 3 letters or numbers. d. The nurse stands 4 feet away from the patient and whispers three different words.

c

1. The nurse is preparing to assess an older adult and discovers that the older adult is in severe pain. Which statement about pain and the older adult is true? a. Pain is inevitable with aging. b. Older adults with cognitive impairments feel less pain. c. Alleviating pain should be a priority over other aspects of the assessment. d. Completion of the assessment should take priority so that care decisions can be made.

c

1. When performing a health history, the nurse would note immunizations under which category? a. Family history b. Personal history c. Past medical history d. History of present illness

c

1. When using the various instruments to assess an older person's ADLs, what should the nurse keep in mind as a disadvantage of these instruments? a. Reliability of the tools b. Lack of confidentiality during the assessment c. Self or proxy reporting of functional activities d. Insufficient details concerning the deficiencies identified

c

1. The nurse is completing an assessment on a patient who was just admitted from the emergency department. Which assessment findings would require immediate attention? (Select all that apply.) a. Sudden restlessness b. Temperature: 38.6°C c. Oxygen saturation: 95% d. Heart rate: 130 beats per minute e. Systolic blood pressure: 150 mm Hg f. Respiratory rate: 22 breaths per minute

a, b, d

1. When auscultating heart sounds, which technique should the nurse use? a. Listen with the bell. b. Listen with the diaphragm. c. Listen with both the diaphragm and bell working from apex to base in a Z pattern. d. Listen with both the bell and diaphragm comparing sides of the heart as progress from apex to base.

b

1. Which statement is true regarding the recording of data from the history and physical examination? a. Use long, descriptive sentences to document findings. b. Record the data as soon as possible after the interview and physical examination. c. The examiner should avoid taking any notes during the history and examination because of the possibility of decreasing the rapport with the patient. d. If the information is not documented, then it can be assumed that it was done as a standard of care.

b

When assessing tactile fremitus, the nurse recalls that it is normal to feel tactile fremitus most intensely over which locations?

between the scapulae

The nurse knows that a normal finding when assessing the respiratory system of an elderly adult is:

decreased mobility of the thorax

The nurse keeps in mind that a thorough skin assessment is very important because the skin holds information about a person's: a. Support systems. b. Circulatory status. c. Socioeconomic status. d. Psychological wellness.

circulatory status. The skin holds information about the body's circulation, nutritional status, and signs of systemic diseases as well as topical data on the integument itself.

the nurse notices that a woman in an exercise class is unable to jump rope. The nurse knows that to jump rope, ine's shoulder has to be capable of:

circumduction

The articulation of the mandible and the temporal bone is known as the:

temporomandibular joint

A 35-year-old recent immigrant is being seen in the clinic for complaints of a cough that is associated with rust-colored sputum, low-grade afternoon fevers, and night sweats for the past 2 months. The nurse's preliminary analysis, based on this history, is that this patient may be suffering from:

tuberculosis

The nurse is reviewing the characteristics of breath sounds. Which statement about bronchovesicular breath sounds is true? They are:

expected near the major airways

The nurse is examining a 2-month-old infant and notices asymmetry of the infant's gluteal folds. The nurse should assess for other signs of what disorder?

hip dislocation

During precussion, the nurse knows that a dull percussion note elicited over a lung lobe most likely results from:

increased density of lung tissue

During an assessment, the nurse knows that expected assessment findings in the normal adult lung include the presence of:

muffled voice sounds and symmetrical tactile fremitus

A patient has been diagnosed with osteoporosis and asks the nurse, "What is osteoporosis?" The nurse explains to the patient that osteoporosis is defined as:

loss of bone density

A professional tennis player comes into the clinic complaining of a sore elbow. The nurse will assess for tenderness at the:

medial and lateral epicondyle

During the aging process, the hair can look gray or white and begin to feel thin and fine. The nurse knows that this occurs because of a decrease in the number of functioning: a. Metrocytes. b. Fungacytes. c. Phagocytes. d. Melanocytes.

melanocytes. In the aging hair matrix, the number of functioning melanocytes decreases so the hair looks gray or white and feels thin and fine. The other options are not correct.

A patient is visiting the clinic for an evaluation of a swollen, painful knuckle. The nurse notices that the knuckle above his ring on the left hand is swollen and that he is unable to remove his wedding ring. This joint is called the _____ joint.

metacarpophalangeal

A teenage girl has arrived complaining of pain in her left wrist. She was playing basketball when she fell and landed on her left hand. The nurse examines her hand and would expect a fracture if the girl complains:

of sharp pain that increases with movement

A 68-year-old woman has come in for an assessment of her rheumatoid arthritis, and the nurse notices raised, firm, nontender nodules at the olecranon bursa and along the ulna. These nodules are most commonly diagnosed as:

olecranon bursitis

When performing a musculoskeletal assessment, the nurse knows that the correct approach for the examination should be"

proximal to distal

The nurse educator is preparing an education module for the nursing staff on the epidermal layer of skin. Which of these statements would be included in the module? The epidermis is: a. Contains mostly fat cells. b. Consists mostly of keratin. c. Is replaced every 4 weeks. d. Contains sensory receptors.

replaced every 4 weeks. The epidermis is thin yet tough, replaced every 4 weeks, avascular, and stratified into several zones.

During an examination of the anterior thorax, the nurse keeps in mind that the trachea bifurcates anteriorly at the:

sternal angle

The nurse is assessing a 1-week-old infant and is testing his muscle strength. The nurse lifts the infant with hands under the axillae and notices that the infant starts to "slip" between the hands. The nurse should:

suspect that the infant may have weakness of the shoulder muscles

When auscultating the lungs of an adult patient, the nurse notes that over the posterior lower lobes low-pitched, soft breath sounds are heard, with inspiration being longer than expiration. The nurse interprets that these are:

vesicular breath sounds and are normal in that location.

During an examination, the nurse finds that a patient has excessive dryness of the skin. The best term to describe this condition is: a. Xerosis. b. Pruritus. c. Alopecia. d. Seborrhea.

xerosis. Xerosis is the term used to describe skin that is excessively dry. Pruritus refers to itching, alopecia refers to hair loss, and seborrhea refers to oily skin.

During an interview the patient states, "I can feel this bump on the top of both of my shoulders--it doesn't hurt but I am curious about what it might be." The nurse should tell the patient that is:

your acromion process

a. A patient newly diagnosed with type 2 diabetes mellitus does not know how to check his own blood glucose levels with a glucometer. b. A teenager who was stung by a bee during a soccer match is having trouble breathing. c. An older adult with a urinary tract infection is also showing signs of confusion and agitation. problem

1. a = First-level priority problem 2. b = Second-level priority problem 3. c = Third-level priority

When examining children affected with Down syndrome (trisomy 21), the nurse looks for the possible presence of: a.Ear dysplasia. b.Long, thin neck. c.Protruding thin tongue. d.Narrow and raised nasal bridge.

A With the chromosomal aberration trisomy 21, also known as Down syndrome, head and face characteristics may include upslanting eyes with inner epicanthal folds, a flat nasal bridge, a small broad flat nose, a protruding thick tongue, ear dysplasia, a short broad neck with webbing, and small hands with a single palmar crease.

The nurse is performing a health interview on a patient, named Salil, who has a language barrier, and no interpreter is available. Which of the following is the best example of an appropriate question for the nurse to ask in this situation? 1. "Does Salil take medicine?" 2. "Do you sterilize the bottles?" 3. "Do you have nausea and vomiting?" 4. "He has been taking his medicine, hasn't he?"

ANS: 1 In a situation where there is a language barrier and no interpreter available, use simple words avoiding medical jargon. Avoid using contractions and pronouns. Use nouns repeatedly and discuss one topic at a time.

Because the physical environment in which an interview takes place is an important consideration for the success of an interview, the interviewer should: 1.reduce noise by turning off televisions and radios. 2.place the distance between the interviewer and the patient about 2 feet or closer. 3.provide a dim light that makes a room cozier and will help the patient relax. 4.arrange seating across a desk or table to allow the patient some personal space.

ANS: 1 Reduce noise. Multiple stimuli are confusing. Turn off the television, radio, and any unnecessary equipment.

A 14-year-old boy who has been diagnosed with Osgood-Schlatter disease reports painful swelling just below the knee for the past 5 months. Which response by the nurse is appropriate?

"Your disease is due to repeated stress on the patellar tendon. It is usually self-limited and your symptoms should resolve with rest."

A patient tells the nurse that he is very nervous, is nauseated, and feels hot. These types of data would be: a. Objective. b. Reflective. c. Subjective. d. Introspective.

C

The nurse is reviewing the function of the cranial nerves (CNs). Which CN is responsible for conducting nerve impulses to the brain from the organ of Corti? a. I b. III c. VIII d. XI

C. VIII The nerve impulses are conducted by the auditory portion of CN VIII to the brain.

After completing a diet assessment on a 30-year-old woman, the nurse suspects that she may be deficient in iron. Laboratory studies to obtain to verify this condition would be: A) hemoglobin and hematocrit. B) cholesterol and triglycerides. C) urinalysis. D) serum albumin.

A

After completing an initial assessment of a patient, the nurse has charted that his respirations are eupneic and his pulse is 58 beats per minute. These types of data would be: a. Objective. b. Reflective. c. Subjective. d. Introspective.

A

How should the nurse document mild, slight pitting edema the ankles of a pregnant patient? a. 1+/0-4+ b. 3+/0-4+ c. 4+/0-4+ d. Brawny edema

A

A patient has been admitted to the emergency department for a suspected drug overdose. his respirations are shallow, with an irregular patter, with a rate of 12 per minute. The nurse interprets this respiration pattern as which of the following?

Hypoventilation

The nurse is performing a general survey. Which action is a component of the general survey? A) Observing the patient's body stature and nutritional status B) Interpreting the subjective information the patient has reported C) Measuring the patient's temperature, pulse, respirations, and blood pressure D) Observing specific body systems while performing the physical assessment

A

The nurse will use which technique of assessment to determine the presence of crepitus, swelling, and pulsations? a. Palpation b. Inspection c. Percussion d. Auscultation

A

The wife of a 65-year-old man tells the nurse that she is concerned because she has noticed a change in her husband's personality and ability to understand. He also cries very easily and becomes angry. The nurse recalls that the cerebral lobe responsible for these behaviors is the __________ lobe. a. Frontal b. Parietal c. Occipital d. Temporal

A

The nurse is performing an assessment on a 7-year-old child who has symptoms of chronic watery eyes, sneezing, and clear nasal drainage. The nurse notices the presence of a transverse line across the bridge of the nose, dark blue shadows below the eyes, and a double crease on the lower eyelids. These findings are characteristic of: a.Allergies. b.Sinus infection. c.Nasal congestion. d.Upper respiratory infection.

A Chronic allergies often develop chronic facial characteristics and include blue shadows below the eyes, a double or single crease on the lower eyelids, open-mouth breathing, and a transverse line on the nose.

During an examination of a 3-year-old child, the nurse notices a bruit over the left temporal area. The nurse should: a.Continue the examination because a bruit is a normal finding for this age. b.Check for the bruit again in 1 hour. c.Notify the parents that a bruit has been detected in their child. d.Stop the examination, and notify the physician.

A Bruits are common in the skull in children under 4 or 5 years of age and in children with anemia. They are systolic or continuous and are heard over the temporal area.

When examining a patient's CN function, the nurse remembers that the muscles in the neck that are innervated by CN XI are the: a.Sternomastoid and trapezius. b.Spinal accessory and omohyoid. c.Trapezius and sternomandibular. d.Sternomandibular and spinal accessory.

A The major neck muscles are the sternomastoid and the trapezius. They are innervated by CN XI, the spinal accessory.

A patient's laboratory data reveal an elevated thyroxine (T4) level. The nurse would proceed with an examination of the _____ gland. a.Thyroid b.Parotid c.Adrenal d.Parathyroid

A The thyroid gland is a highly vascular endocrine gland that secretes T4 and triiodothyronine (T3). The other glands do not secrete T4.

Statement by the patient would indicate a need for further teaching? A. "The best time to check my breasts is a week before my cycle." B. "I will check my breasts in the shower one week after my cycle." C. "I will exam my breast in a clock-like sequence so that I don't miss any surface area." D. "I will be sure to check my arm pits."

A The week before a woman's cycle is not the best time to check for lumps because the breasts become sore and tender and may also be "lumpy" a week prior to the cycle. Answer 2 is incorrect because the week after a woman's cycle is the best time to do the examination, and the shower allows for ease of palpation. Answer 3 in incorrect because a clock-like sequence is the proper way to perform a BSE. Answer 4 is incorrect because under the arms houses lymph nodes and the axillary tail of Spence, which is a common site for breast cancer.

During an examination, the nurse knows that the best way to palpate the lymph nodes in the neck is described by which statement? a.Using gentle pressure, palpate with both hands to compare the two sides. b.Using strong pressure, palpate with both hands to compare the two sides. c.Gently pinch each node between one's thumb and forefinger, and then move down the neck muscle. d.Using the index and middle fingers, gently palpate by applying pressure in a rotating pattern.

A Using gentle pressure is recommended because strong pressure can push the nodes into the neck muscles. Palpating with both hands to compare the two sides symmetrically is usually most efficient.

The uterus is usually positioned tilting forward and superior to the bladder. This position is known as: A) anteverted and anteflexed. B) retroverted and anteflexed. C) retroverted and retroflexed. D) superiorverted and anteflexed.

ANS: A The uterus is freely movable, not fixed, and usually tilts forward and superior to the bladder (a position labeled as anteverted and anteflexed).

A patient is complaining of pain in his joints that is worse in the morning, is better after he has moved around for awhile, and then gets worse again if he sits for long period of time. The nurse should assess for other signs of what problem?

Rheumatoid arthritis

During assessment of a patient's pain, the nurse keeps in mind that certain nonverbal behaviors are associated with chronic pain. Which of these behaviors are associated with chronic pain? Select all that apply. A) Sleeping B) Moaning C) Diaphoresis D) Bracing E) Restlessness F) Rubbing

A D F

While measuring a patient's blood pressure, the nurse uses proper technique to obtain an accurate reading. Which of these situations will result in a falsely high blood pressure reading? Select all that apply. A) The person supports his or her own arm during the blood pressure reading. B) The blood pressure cuff is too narrow for the extremity. C) The arm is held above level of the heart. D) The cuff is wrapped loosely around the arm. E) The person is sitting with his or her legs crossed. F) The nurse does not inflate the cuff high enough.

A B D E

. The nurse is reviewing data collected after an assessment. Of the data listed below, which would be considered related cues that would be clustered together during data analysis? Select all that apply. a. Inspiratory wheezes noted in left lower lobes b. Hypoactive bowel sounds c. Nonproductive cough d. Edema, +2, noted on left hand e. Patient reports dyspnea upon exertion f. Rate of respirations 16 breaths per minute

A C E F

When examining the eye, the nurse notices that the patient's eyelid margins approximate completely. The nurse recognizes that this assessment finding: a. Is expected. b. May indicate a problem with extraocular muscles. c. May result in problems with tearing. d. Indicates increased intraocular pressure.

A is expected The palpebral fissure is the elliptical open space between the eyelids, and, when closed, the lid margins approximate completely, which is a normal finding.

The nurse notices that a school-aged child has bluish-white, red-based spots in her mouth that are elevated about 1 mm to 3 mm. What other signs would the nurse expect to find in this patient? a. Pink, papular rash on the face and neck b. Pruritic vesicles over her trunk and neck c. Hyperpigmentation on the chest, abdomen, and back of the arms d. Red-purple, maculopapular, blotchy rash behind the ears and on the face

A red-purple, maculopapular, blotchy rash behind the ears and on the face With measles (rubeola), the examiner would assess a red-purple, blotchy rash on the third or fourth day of illness that appears first behind the ears and spreads over the face and then over the neck, trunk, arms and legs. It looks coppery and does not blanch. The bluish-white, red-based spots in the mouth are known as Koplik's spots.

The nurse is assessing a patient's headache pain. Which questions reflect one or more of the critical characteristics of symptoms that should be assessed? Select all that apply. a."Where is the headache pain?" b."Did you have these headaches as a child?" c."On a scale of 1 to 10, how bad is the pain?" d."How often do the headaches occur?" e."What makes the headaches feel better?" f."Do you have any family history of headaches?"

A, C, D, E

A patient has been admitted with chronic arterial symptoms. During the assessment, the nurse should expect which findings? Select all that apply. a. Patient has a history of diabetes and cigarette smoking. b. Skin of the patient is pale and cool. c. His ankles have two small, weeping ulcers. d. Patient works long hours sitting at a computer desk. e. He states that the pain gets worse when walking. f. Patient states that the pain is worse at the end of the day.

A,B,E

The nurse is asking questions about a patient's health beliefs. Which questions are appropriate? Select all that apply. a. "What is your definition of health?" b. "Does your family have a history of cancer?" c. "How do you describe illness?" d. "What did your mother do to keep you from getting sick?" e. "Have you ever had any surgeries?" f. "How do you keep yourself healthy?"

A,C,D,F

A patient is recovering from several hours of orthopedic surgery. During an assessment of the patient's lower legs, the nurse will monitor for signs of acute venous symptoms. Signs of acute venous symptoms include which of the following? Select all that apply. a. Intense, sharp pain, with the deep muscle tender to the touch b. Aching, tired pain, with a feeling of fullness c. Pain that is worse at the end of the day d. Sudden onset e. Warm, red, and swollen calf f. Pain that is relieved with elevation of the leg

A,D,E

During an interview, the nurse would expect that most of the interview will take place at which distance? 1.Intimate zone 2.Personal distance 3.Social distance 4.Public distance

ANS: 3 Social distance, 4 to 12 feet, is usually the distance category for most of the interview. Public distance, over 12 feet, is too much distance; the intimate zone is inappropriate, and the personal distance will be used for the physical assessment.

The following statement could be found at which phase of the interview? "Mr. S., I would like to ask you some questions about your health and your usual daily activities so that we can better plan your stay here." 1. During the summary 2. Closing the interview 3. During the body of the interview 4. Opening/introducing the interview

ANS: 4 When gathering a complete history, give the reason for the interview during the opening or introduction of the interview.

34. The nurse is reviewing statistics for lactose intolerance. In the United States, the incidence of lactose intolerance is higher in adults of which ethnic group? A) African-Americans B) Hispanics C) Whites D) Asians

ANS: A A recent study found lactose-intolerance prevalence estimates as follows: 19.5% for African-Americans, 10% for Hispanics, and 7.72% for whites

2. A patient visits the clinic to ask about smoking cessation. He has smoked heavily for 30 years and wants to stop cold turkey. He asks the nurse, What symptoms can I expect if I do this? Which of these symptoms should the nurse share with the patient as possible symptoms of nicotine withdrawal? Select all that apply. a.Headaches b.Hunger c.Sleepiness d.Restlessness e.Nervousness f.Sweating

ANS: A, B, D, E Symptoms of nicotine withdrawal include vasodilation, headaches, anger, irritability, frustration, anxiety, nervousness, awakening at night, difficulty concentrating, depression, hunger, impatience, and the desire to smoke (see Table 6-7).

The nurse is performing a digital examination of a patient's prostate gland and notices that characteristics of a normal prostate gland include which of the following? Select all that apply. A) The gland protruding 1 cm into the rectum B) Heart-shaped with a palpable central groove C) Flat with no groove palpable D) Boggy and soft consistency E) Smooth surface, elastic, or rubbery consistency F) Fixed mobility

ANS: A, B, E The size should be 2.5 cm long by 4 cm wide, and it should not protrude more than 1 cm into the rectum. The prostate should be heart-shaped, with a palpable central groove, a smooth surface, and elastic, rubbery consistency. Abnormal findings include a flat shape with no palpable groove, boggy with a soft consistency, and fixed mobility.

The nurse is examining a 62-year-old man and notes that he has gynecomastia bilaterally. The nurse should explore his history for which related conditions? Select all that apply. A) Obesity B) Malnutrition C) Hyperthyroidism D) Type 2 diabetes mellitus E) Liver disease F) History of alcohol abuse

ANS: A, C, E, F Gynecomastia occurs with obesity, Cushing's syndrome, liver cirrhosis, adrenal disease, hyperthyroidism, and numerous drugs: alcohol and marijuana use, estrogen treatment for prostate cancer, antibiotics (metronidazole, isoniazid), digoxin, ACE inhibitors, diazepam, and tricyclic antidepressants.

The nurse is aware that intimate partner violence (IPV) screening should occur with which situation?

ANS: As a routine part of each health care encounter Many nursing professional organizations have called for routine, universal screening for IPV to assist women in getting help for the problem.

3. The nurse is conducting a class on alcohol and the effects of alcohol on the body. How many standard drinks (each containing 14 grams of alcohol) per day in men are associated with increased deaths from cirrhosis, cancers of the mouth, esophagus, and injuries? a. 2 b. 4 c. 6 d. 8

ANS: B In men, alcohol consumption of at least four standard drinks per day is associated with increased deaths from liver cirrhosis, cancers of the mouth, esophagus and other areas, and deaths from injuries and other external causes.

During a discussion for a men's health group, the nurse relates that the group with the highest incidence of prostate cancer is: A) Asian Americans. B) African-Americans. C) American Indians. D) Hispanics.

ANS: B According to the American Cancer Society (2010), African-American men have a higher rate of prostate cancer than other racial groups.

During a digital examination of the rectum, the nurse notices that the patient has hard feces in the rectum. The patient complains of feeling "full," has a distended abdomen, and states that she has not had a bowel movement "for several days." The nurse suspects which condition? A) Rectal polyp B) Fecal impaction C) Rectal abscess D) Rectal prolapse

ANS: B A fecal impaction is a collection of hard, desiccated feces in the rectum. The obstruction often results from decreased bowel motility, in which more water is reabsorbed from the stool. See Table 25-2 for descriptions of rectal polyp and abscess; See Table 25-1 for description of rectal prolapse

10. A patient has hypoactive bowel sounds. The nurse knows that a potential cause of hypoactive bowel sounds is: A) diarrhea. B) peritonitis. C) laxative use. D) gastroenteritis.

ANS: B Diminished or absent bowel sounds signal decreased motility from inflammation as seen with peritonitis, with paralytic ileus after abdominal surgery, or with late bowel obstruction

A woman has just been diagnosed with HPV, or genital warts. The nurse should counsel her to receive regular examinations because this virus makes her at a higher risk for _____ cancer. A) uterine B) cervical C) ovarian D) endometrial

ANS: B HPV is the virus responsible for most cases of cervical cancer, not the other options.

During the neurologic assessment of a "healthy" 35-year-old patient, the nurse asks him to relax his muscles completely. The nurse then moves each extremity through full range of motion. Which of these results would the nurse expect to find? a. Firm, rigid resistance to movement b. Mild, even resistance to movement c. Hypotonic muscles as a result of total relaxation d. Slight pain with some directions of movement

B

41. The nurse suspects that a patient has appendicitis. Which of these procedures are appropriate for use when assessing for appendicitis or a perforated appendix? Select all that apply. A. Test for Murphy's sign. B. Test for Blumberg's sign. C. Test for shifting dullness. D. Perform iliopsoas muscle test. E. Test for fluid wave.

ANS: B, D Testing for Blumberg's sign (rebound tenderness) and performing the iliopsoas muscle test should be used to assess for appendicitis. Murphy's sign is used to assess for an inflamed gallbladder or cholecystitis. Testing for a fluid wave and shifting dullness is done to assess for ascites

The nurse is palpating an ovarian mass during an internal examination of a 63-year-old woman. Which findings of the mass's characteristics would suggest the presence of an ovarian cyst? Select all that apply. A) Heavy and solid B) Mobile and fluctuant C) Mobile and solid D) Fixed E) Smooth and round F) Poorly defined

ANS: B, E An ovarian cyst (fluctuant ovarian mass) is usually asymptomatic, and would feel like a smooth, round, fluctuant, mobile, nontender mass on the ovary. A mass that is heavy, solid, fixed, and poorly defined suggests malignancy. A benign mass may feel mobile and solid.

The nurse is discussing epidermal appendages with a newly graduated nurse. Which of these would be included in the discussion?

Sweat glands Epidermal appendages include hair, sebaceous glands, sweat glands, and nails.

8. The nurse is asking an adolescent about illicit substance abuse. The adolescent answers, Yes, Ive used marijuana at parties with my friends. What is the next question the nurse should ask? a. Who are these friends? b. Do your parents know about this? c. When was the last time you used marijuana? d. Is this a regular habit?

ANS: C If a patient admits to the use of illicit substances, then the nurse should ask, When was the last time you used drugs? and How much did you take that time? The other questions may be considered accusatory and are not conducive to gathering information.

A 5-year-old boy is being admitted to the hospital to have his tonsils removed. Which information should the nurse collect before this procedure? a.Child's birth weight b.Age at which he crawled c.Whether the child has had the measles d.Child's reactions to previous hospitalizations

D

The nurse is preparing for an internal genitalia examination of a woman. Which order of the examination is correct? A) Bimanual, speculum, rectovaginal B) Speculum, rectovaginal, bimanual C) Speculum, bimanual, rectovaginal D) Rectovaginal, bimanual, speculum

ANS: C The correct sequence is speculum examination, then bimanual examination after removing the speculum, and then rectovaginal examination. The examiner should change gloves before performing the rectovaginal examination to avoid spreading any possible infection

During an assessment, a patient tells the nurse that her fingers often change color when she goes out in cold weather. She describes these episodes as her fingers first turning white, then blue, then red with a burning, throbbing pain. The nurse suspects that she is experiencing: a. Lymphedema. b. Raynaud disease. c. Deep-vein thrombosis. d. Chronic arterial insufficiency.

B

A patient is being assessed for range of joint movement, The nurse asks him to move his arm in toward the center of his body. This movement is called:

Adduction

The ankle joint is the articulation of the tibia, the fibula, and the:

talus

A patient has a long history of chronic obstructive pulmonary disease. During the assessment, the nurse is most likely to observe which of these?

An anteroposterior-to-transverse diameter ratio of 1:1

The nurse is bathing an 80-year-old man and notices that his skin is wrinkled, thin, lax, and dry. This finding would be related to which factor? a. Increased vascularity of the skin b. Increased numbers of sweat and sebaceous glands c. An increase in elastin and a decrease in subcutaneous fat d. An increased loss of elastin and a decrease in subcutaneous fat

An increased loss of elastin and a decrease in subcutaneous fat in the elderly An accumulation of factors place the aging person at risk for skin disease and breakdown: the thinning of the skin, the decrease in vascularity and nutrients, the loss of protective cushioning of the subcutaneous layer, a lifetime of environmental trauma to skin, the social changes of aging, the increasingly sedentary lifestyle, and the chance of immobility.

A black patient is in the intensive care unit because of impending shock after an accident. The nurse would expect to find what characteristics in this patient's skin? a. Ruddy blue. b. Generalized pallor. c. Ashen, gray, or dull. d. Patchy areas of pallor.

Ashen, gray, or dull Pallor due to shock (decreased perfusion and vasoconstriction) in black-skinned people will cause the skin to appear ashen, gray, or dull. See Table 12-2.

A 65-year-old patient with a history of heart failure comes to the clinic with complaints of "being awakened from sleep with shortness of breath." Which action by the nurse is most appropriate?

Assess for other signs and symptoms of paroxysmal nocturnal dyspea.

A 21-year-old woman has been on a low-protein liquid diet for the past 2 months. She has had adequate intake of calories and appears well nourished. After further assessment, what would the nurse expect to find? a. Poor skin turgor b. Decreased serum albumin c. Increased lymphocyte count d. Triceps skinfold less than standard

B

When performing a physical examination, safety must be considered to protect the examiner and the patient against the spread of infection. Which of these statements describes the most appropriate action the nurse should take when performing a physical examination? a. Washing ones hands after removing gloves is not necessary, as long as the gloves are still intact. b. Hands are washed before and after every physical patient encounter. c. Hands are washed before the examination of each body system to prevent the spread of bacteria from one part of the body to another. d. Gloves are worn throughout the entire examination to demonstrate to the patient concern regarding the spread of infectious diseases.

B

A patient reports excruciating headache pain on one side of his head, especially around his eye, forehead, and cheek that has lasted approximately to 2 hours, occurring once or twice each day. The nurse should suspect: a.Hypertension. b.Cluster headaches. c.Tension headaches. d.Migraine headaches.

B Cluster headaches produce pain around the eye, temple, forehead, and cheek and are unilateral and always on the same side of the head. They are excruciating and occur once or twice per day and last to 2 hours each.

A patient is unable to differentiate between sharp and dull stimulation to both sides of her face. The nurse suspects: a.Bell palsy. b.Damage to the trigeminal nerve. c.Frostbite with resultant paresthesia to the cheeks. d.Scleroderma.

B Facial sensations of pain or touch are mediated by CN V, which is the trigeminal nerve. Bell palsy is associated with CN VII damage. Frostbite and scleroderma are not associated with this problem.

The nurse has just completed a lymph node assessment on a 60-year-old healthy female patient. The nurse knows that most lymph nodes in healthy adults are normally: a.Shotty. b.Nonpalpable. c.Large, firm, and fixed to the tissue. d.Rubbery, discrete, and mobile.

B Most lymph nodes are nonpalpable in adults. The palpability of lymph nodes decreases with age. Normal nodes feel movable, discrete, soft, and nontender.

A patient comes to the clinic complaining of neck and shoulder pain and is unable to turn her head. The nurse suspects damage to CN ______ and proceeds with the examination by _____________. a.XI; palpating the anterior and posterior triangles b.XI; asking the patient to shrug her shoulders against resistance c.XII; percussing the sternomastoid and submandibular neck muscles d.XII; assessing for a positive Romberg sign

B The major neck muscles are the sternomastoid and the trapezius. They are innervated by CN XI, the spinal accessory. The innervated muscles assist with head rotation and head flexion, movement of the shoulders, and extension and turning of the head.

A male patient with a history of acquired immunodeficiency syndrome (AIDS) has come in for an examination and he states, "I think that I have the mumps." The nurse would begin by examining the: a.Thyroid gland. b.Parotid gland. c.Cervical lymph nodes. d.Mouth and skin for lesions.

B The parotid gland may become swollen with the onset of mumps, and parotid enlargement has been found with human immunodeficiency virus (HIV).

The nurse is assessing an obese patient for signs of metabolic syndrome. This condition is diagnosed when three or more certain risk factors are present. Which of these assessment findings are risk factors for metabolic syndrome? Select all that apply. A) Fasting plasma glucose level less than 100 mg/dL B) Fasting plasma glucose level greater than or equal to 110 mg/dL C) Blood pressure reading of 140/90 mm Hg D) Blood pressure reading of 110/80 mm Hg E) Triglyceride level of 120 mg/dL

B C

A 69-year-old patient has been admitted to an adult psychiatric unit because his wife thinks he is getting more and more confused. He laughs when he is found to be forgetful, saying "I'm just getting old!" After the nurse completes a thorough neurologic assessment, which findings would be indicative of Alzheimer disease? Select all that apply. a. Occasionally forgetting names or appointments b. Difficulty performing familiar tasks, such as placing a telephone call c. Misplacing items, such as putting dish soap in the refrigerator d. Sometimes having trouble finding the right word e. Rapid mood swings, from calm to tears, for no apparent reason f. Getting lost in one's own neighborhood

B, C, E, F

The nurse should use which test to check for large amounts of fluid around the patella?

Ballottement

When reviewing the musculoskeletal system, the nurse recalls that hematopoiesis takes place in the:

Bone Marrow

The nurse notices that a patient's palpebral fissures are not symmetric. On examination, the nurse may find that damage has occurred to which cranial nerve (CN)? a.III b.V c.VII d.VIII

C Facial muscles are mediated by CN VII; asymmetry of palpebral fissures may be attributable to damage to CN VII (Bell palsy).

While performing a well-child assessment on a 5 year old, the nurse notes the presence of palpable, bilateral, cervical, and inguinal lymph nodes. They are approximately 0.5 cm in size, round, mobile, and nontender. The nurse suspects that this child: a.Has chronic allergies. b.May have an infection. c.Is exhibiting a normal finding for a well child of this age. d.Should be referred for additional evaluation.

C Palpable lymph nodes are normal in children until puberty when the lymphoid tissue begins to atrophy. Lymph nodes may be up to 1 cm in size in the cervical and inguinal areas but are discrete, movable, and nontender.

When examining the face of a patient, the nurse is aware that the two pairs of salivary glands that are accessible to examination are the ___________ and ___________ glands. a.Occipital; submental b.Parotid; jugulodigastric c.Parotid; submandibular d.Submandibular; occipital

C Two pairs of salivary glands accessible to examination on the face are the parotid glands, which are in the cheeks over the mandible, anterior to and below the ear; and the submandibular glands, which are beneath the mandible at the angle of the jaw. The parotid glands are normally nonpalpable.

When checking for proper blood pressure cuff size, the nurse knows that which guideline is correct? A) The standard cuff size is appropriate for all sizes. B) The length of the rubber bladder should equal 80% of the arm circumference. C) The width of the rubber bladder should equal 80% of the arm circumference. D) The width of the rubber bladder should equal 40% of the arm circumference.

D

A patient has been admitted to the emergency department with a possible medical diagnosis of pulmonary embolism. The nurse expects to see which assessments findings related to this condition?

Chest pain that is worse on deep inspiration, dyspnea

A patient tells the nurse that he has noticed that one of his moles has started to burn and bleed. When assessing his skin, the nurse would pay special attention to the danger signs for pigmented lesions and would be concerned with which additional finding? a. Color variation b. Border regularity c. Symmetry of lesions d. Diameter of less than 6 mm

Color variation Abnormal characteristics of pigmented lesions are summarized in the mnemonic ABCD: asymmetry of pigmented lesion, border irregularity, color variation, and diameter greater than 6 mm.

A 35-year-old pregnant woman comes to the clinic for a monthly appointment. During the assessment, the nurse notices that she has a brown patch of hyperpigmentation on her face. The nurse continues the skin assessment aware that another finding may be: a. Keratoses. b. Xerosis. c. Chloasma. d. Acrochordons.

Cholasma In pregnancy, skin changes can include striae, linea nigra (a brownish black line down the midline), chloasma (brown patches of hyperpigmentation), and vascular spiders. Keratoses are raised, thickened areas of pigmentation that look crusted, scaly, and warty. Xerosis is dry skin. Acrochordons, or "skin tags" occur more often in the aging adult.

A patient states, "I can hear a crunching or grating sound when I kneel." she also states that "it is very difficult to get out of bed in the morning because of stiffness and pain in my joints." The nurse should assess for signs of what problem?

Crepitation

A 19-year-old college student is brought to the emergency department with a severe headache he describes as, "Like nothing I've ever had before." His temperature is 40° C, and he has a stiff neck. The nurse looks for other signs and symptoms of which problem? a.Head injury b.Cluster headache c.Migraine headache d.Meningeal inflammation

D The acute onset of neck stiffness and pain along with headache and fever occurs with meningeal inflammation. A severe headache in an adult or child who has never had it before is a red flag. Head injury and cluster or migraine headaches are not associated with a fever or stiff neck.

An elderly woman is brought to the emergency department after being found lying on the kitchen floor 2 days, and she is extremely dehydrated. What would the nurse expect to see upon examination? a. Smooth mucous membranes and lips b. Dry mucous membranes and cracked lips c. Pale mucous membranes d. White patches on the mucous membranes

Dry mucous membranes and cracked lips With dehydration, mucous membranes look dry and lips look parched and cracked. The other responses are not found in dehydration.

The nurse is explaining the mechanism of the growth of long bones to a mother of a toddler. Where does lengthening of the bones occur?

Epiphyses

Fibrous brands running directly from one bone to another that strengthen the joint to help prevent movement in undesirable directions are called:

Ligaments

A woman who is 8 months pregnant comments that she had noticed a change in posture and is having lower back pain. The nurse tells her that during pregnancy women have a posture shift to compensate for the enlarging fetus. This shift in posture is known as:

Lordosis

During auscultation of the lungs of an adult patient, the nurse notices the presence of bronchophony. The nurse should assess for signs of which conditions?

Pulmonary consolidation

When considering the biocultural differences in the respiratory systems, the nurse knows that which statement is true?

The largest chest volumes are found in whites

The nurse is assessing the lungs of an older adult. Which of these describes normal changes in the respiratory system of the older adult?

The lungs are less elastic and distensible, which decreases their ability to collapse and recoil.

A newborn infant is in the clinic for a well-baby check. The nurse observes the infant for the possibility of fluid loss because of which of these factors? a. Subcutaneous fat deposits are high in the newborn. b. Sebaceous glands are over productive in the newborn. c. The newborns skin is more permeable than that of the adult. d. The amount of vernix caseosa dramatically rises in the newborn.

The newborn's skin is more permeable than that of the adult. The newborn's skin is thin, smooth, and elastic and is relatively more permeable than that of the adult, so the infant is at greater risk for fluid loss. The subcutaneous layer in the infant is inefficient, not thick, and the sebaceous glands are present but decrease in size and production. Vernix caseosa is not produced after birth.

When inspecting the anterior chest of an adult, the nurse should include which assessment?

The shape and configuration of the chest wall.

An Inuit visiting Nevada from Anchorage has come to the clinic in July during the hottest part of the day. It so happens that the clinic's air conditioning is broken and the temperature is very hot. The nurse knows that which of these statements is true about the Inuit sweating tendencies?

They will sweat more on their faces and less on their trunks and extremities. Inuits have made an interesting environmental adaptation whereby they sweat less than whites on their trunks and extremities but more on their faces.

The nurse is examining a 6-month-old infant and place the infant's feet flat on the table and flexes his knees up. The nurse notes that the right knee is significantly lower than the left. Which of these statements is true of this finding?

This is a positive Allis sign and suggest hip dislocation.

The nurse is auscultating the chest in an adult. Which technique is correct?

Use the diaphragm of the stethoscope held firmly against the chest.

1. An 85-year-old man has been hospitalized after a fall at home, and his 86-year-old wife is at his bedside. She tells the nurse that she is his primary caregiver. What should the nurse assess the patient's wife for as a sign of possible caregiver burnout? a. Depression b. Weight gain c. Hypertension d. Social phobias

a

1. In which situation should the examiner auscultate for carotid bruits? a. Middle-aged or older patient b. Pregnant patient with gestational diabetes c. Patient that reports abdominal pain d. Patient with enlarged, tender cervical lymph nodes

a

1. The examiner is assessing the extraocular muscles. Which of the following tests would be inappropriate? a. Confrontation test b. Corneal light reflex c. Six cardinal positions of gaze d. Cranial nerve III, IV, and VI testing

a

1. The nurse is assessing an older adult's advanced activities of daily living (AADLs), which would include a. Recreational activities b. Meal preparation c. Balancing the checkbook d. Self-grooming activities

a

1. The nurse is preparing to perform a functional assessment of an older patient. What is an appropriate approach for the nurse to take? a. Observe the patient's ability to perform the tasks. b. Ask the patient's wife how he does when performing tasks. c. Review the medical record for information on the patient's abilities. d. Ask the patient's physician for information on the patient's abilities.

a

1. When standing with their eyes closed, feet together, and arms at their sides, a patient sways and starts to fall. How should the nurse document this finding? a. Positive Romberg sign b. Positive Babinski sign c. Positive Ortolani sign d. Positive modified Allen test

a

An 80-year-old woman is visiting the clinic for a checkup. She states, "I can't walk as much as I used to." The nurse is observing for motor dysfunction in her hips and should have her:

abduct her hip while she is lying on her back

A 40-year-old man had come into the clinic with complaints of "Extreme tenderness in my toes." The nurse notices that his toes are slightly swollen, reddened, and warm to the touch. His complaints would suggest:

acute gout

A patient is especially worried about an area of skin on her feet that has turned white. The health care provider has told her that her condition is vitiligo. The nurse explains to her that vitiligo is: a. Caused by an excess of melanin pigment b. Caused by an excess of apocrine glands in her feet c. Caused by the complete absence of melanin pigment d. Related to impetigo and can be treated with an ointment

caused by the complete absence of melanin pigment. Vitiligo is the complete absence of melanin pigment in patchy areas of white or light skin on the face, neck, hands, feet, body folds, and around orifices—otherwise the depigmented skin is normal.

The nurse notes hyperresonant percussion tones when percussing the thorax of an infant. The nurse's best action would be to:

consider this a normal finding

A newborn infant has Down syndrome. During the skin assessment, the nurse notices a transient mottling in the trunk and extremities in response to the cooler examination room temperature. The infant's mother also notices the mottling and asks what it is. The nurse knows that this mottling is called: a. Caf au lait. b. Carotenemia. c. Acrocyanosis. d. Cutis marmorata.

cutis marmorata. Persistent or pronounced cutis marmorata occurs with Down syndrome or prematurity and is a transient mottling in the trunk and extremities in response to cooler room temperatures. A café au lait spot is a large round or oval patch of light-brown pigmentation. Carotenemia produces a yellow-orange color in light-skinned persons. Acrocyanosis is a bluish color around the lips, hands and fingernails, and feet and toenails.

1. The nurse is assessing an older adult's functional ability. Which definition correctly describes one's functional ability? a. It denotes an older person's cognitive level. b. It is the measure of the expected changes of aging that one is experiencing. c. It describes the individual's motivation to live independently. d. It refers to one's ability to perform activities necessary to live in modern society.

d

1. When beginning to assess a person's spirituality, which question by the nurse would be most appropriate? a. "Do you believe in God?" b. "What religious faith do you follow?" c. "Do you believe in the power of prayer?" d. "How does your spirituality relate to your health care decisions?"

d

The nurse is assessing a patient's ischial tuberosity. To palpate the ischial tuberosity, the nurse knows that it is best to have the patient:

flex the hip

A patient comes in for a physical, and she complains of "freezing to death" while waiting for her examination. The nurse notes that her skin is pale and cool and attributes this finding to: a. Venous pooling. b. Peripheral vasodilation. c. Peripheral vasoconstriction. d. Decreased arterial perfusion.

peripheral vasoconstriction. A chilly or air-conditioned environment causes vasoconstriction, which results in false pallor and coolness. See Table 12-1.

During a neonatal examination, the nurse notices that the newborn infant has six toes. This finding is documented as:

polydactyly.

A patient comes to the clinic complaining of a cough that is worse at night but not as bad during the day. The nurse recongnizes that this may indicate:

postnasal drip or sinusitis

A woman in her 26th week of pregnancy states that she is "not really short of breath" but feels that she is aware ofher breathing and the need to breathe. What is the nurse's best reply?

"What you are experiencing is normal. Some women may interpret this as shortness of breath, but it is a normal finding and nothing is wrong."

The nurse is preparing for a certification course in skin care and needs to be familiar with the various lesions that may be identified on assessment of the skin. Which of the following definitions are correct? Select all that apply. a. Petechiae: Tiny punctate hemorrhages, 1 to 3 mm, round and discrete, dark red, purple, or brown in color b. Bulla: Elevated, circumscribed lesion filled with turbid fluid (pus) c. Papule: Hypertrophic scar d. Vesicle: Known as a friction blister e. Nodule: Solid, elevated, and hard or soft growth that is larger than 1 cm

-Petechiae: Tiny punctate hemorrhages, 1 to 3 mm, round and discrete, dark red, purple, or brown in color. -Vesicle: Also known as a friction blister. -Nodule: Solid, elevated, hard or soft, larger than 1 cm An elevated, circumscribed lesion filled with turbid fluid (pus) is a pustule. A hypertrophic scar is a keloid. A bulla is larger than 1 cm and contains clear fluid; a papule is solid, elevated, but less than 1 cm.

When assessing muscle strength, the nurse observes that a patient has complete range of motion against gravity with full resistance. What grade should the nurse record using a 0 to 5 point scale?

5

Of the 33 vertebrae in the spinal column, there are:

5 lumbar

A 21-year-old patient has a head injury resulting from trauma and is unconscious. There are no other injuries. During the assessment what would the nurse expect to find when testing the patient's deep tendon reflexes? a. Reflexes will be normal. b. Reflexes cannot be elicited. c. All reflexes will be diminished but present. d. Some reflexes will be present, depending on the area of injury.

A

A 30-year-old woman has recently moved to the United States with her husband. They are living with the woman's sister until they can get a home of their own. When company arrives to visit with the woman's sister, the woman feels suddenly shy and retreats to the back bedroom to hide until the company leaves. She explains that her reaction to guests is simply because she does not know how to speak "perfect English." This woman could be experiencing: a. Culture shock. b. Cultural taboos. c. Cultural unfamiliarity. d. Culture disorientation.

A

A 59-year-old patient has a herniated intervertebral disk. Which of the following findings should the nurse expect to see on physical assessment of this individual? a. Hyporeflexia b. Increased muscle tone c. Positive Babinski sign d. Presence of pathologic reflexes

A

A 6-month-old infant has been brought to the well-child clinic for a check-up. She is currently sleeping. What should the nurse do first when beginning the examination? a. Auscultate the lungs and heart while the infant is still sleeping. b. Examine the infants hips, because this procedure is uncomfortable. c. Begin with the assessment of the eye, and continue with the remainder of the examination in a head-to-toe approach. d. Wake the infant before beginning any portion of the examination to obtain the most accurate assessment of body systems

A

A 67-year-old patient states that he recently began to have pain in his left calf when climbing the 10 stairs to his apartment. This pain is relieved by sitting for approximately 2 minutes; then he is able to resume his activities. The nurse interprets that this patient is most likely experiencing: a. Claudication. b. Sore muscles. c. Muscle cramps. d. Venous insufficiency.

A

A 75-year-old man has a history of hypertension and was recently changed to a new antihypertensive drug. He reports feeling dizzy at times. How should the nurse evaluate his blood pressure? A) Assess blood pressure and pulse in the supine, sitting, and standing positions. B) Have the patient walk around the room and assess his blood pressure after activity. C) Assess his blood pressure and pulse at the beginning and end of the examination. D) Take the blood pressure on the right arm and then 5 minutes later take the blood pressure on the left arm.

A

A man is at the clinic for a physical examination. He states that he is very anxious about the physical examination. What steps can the nurse take to make him more comfortable? a. Appear unhurried and confident when examining him. b. Stay in the room when he undresses in case he needs assistance. c. Ask him to change into an examining gown and to take off his undergarments. d. Defer measuring vital signs until the end of the examination, which allows him time to become comfortable.

A

A newly admitted patient is in acute pain, has not been sleeping well lately, and is having difficulty breathing. How should the nurse prioritize these problems? a. Breathing, pain, and sleep b. Breathing, sleep, and pain c. Sleep, breathing, and pain d. Sleep, pain, and breathing

A

A patient has hard, nonpitting edema of the left lower leg and ankle. The right leg has no edema. Based on these findings, the nurse recalls that: a. Nonpitting, hard edema occurs with lymphatic obstruction. b. Alterations in arterial function will cause edema. c. Phlebitis of a superficial vein will cause bilateral edema. d. Long-standing arterial obstruction will cause pitting edema.

A

A patient is at the clinic to have her blood pressure checked. She has been coming to the clinic weekly since she changed medications 2 months ago. The nurse should: a. Collect a follow-up data base and then check her blood pressure. b. Ask her to read her health record and indicate any changes since her last visit. c. Check only her blood pressure because her complete health history was documented 2 months ago. d. Obtain a complete health history before checking her blood pressure because much of her history information may have changed.

A

A patient tells the nurse that she has had abdominal pain for the past week. What would be the nurse's best response? a."Can you point to where it hurts?" b."We'll talk more about that later in the interview." c."What have you had to eat in the last 24 hours?" d."Have you ever had any surgeries on your abdomen

A

A woman who has lived in the United States for a year after moving from Europe has learned to speak English and is almost finished with her college studies. She now dresses like her peers and says that her family in Europe would hardly recognize her. This nurse recognizes that this situation illustrates which concept? a. Assimilation b. Heritage consistency c. Biculturalism d. Acculturation

A

An Asian-American woman is experiencing diarrhea, which is believed to be "cold" or "yin." The nurse expects that the woman is likely to try to treat it with: a. Foods that are "hot" or "yang." b. Readings and Eastern medicine meditations. c. High doses of medicines believed to be "cold." d. No treatment is tried because diarrhea is an expected part of life.

A

Barriers to incorporating EBP include: a. Nurses lack of research skills in evaluating the quality of research studies. b. Lack of significant research studies. c. Insufficient clinical skills of nurses. d. Inadequate physical assessment skills.

A

During a nutritional assessment, why is it important for the nurse to ask a patient what medications he or she is taking? a. Certain drugs can affect the metabolism of nutrients. b. The nurse needs to assess the patient for allergic reactions. c. Medications need to be documented in the record for the physician's review. d. Medications can affect one's memory and ability to identify food eaten in the last 24 hours.

A

During an assessment of a 62-year-old man, the nurse notices the patient has a stooped posture, shuffling walk with short steps, flat facial expression, and pill-rolling finger movements. These findings would be consistent with: a. Parkinsonism. b. Cerebral palsy. c. Cerebellar ataxia. d. Muscular dystrophy.

A

During an examination of a child, the nurse considers that physical growth is the best index of a child's: A) general health. B) genetic makeup. C) nutritional status. D) activity and exercise patterns.

A

During the assessment of an 80-year-old patient, the nurse notices that his hands show tremors when he reaches for something and his head is always nodding. No associated rigidity is observed with movement. Which of these statements is most accurate? a. These findings are normal, resulting from aging. b. These findings could be related to hyperthyroidism. c. These findings are the result of Parkinson disease. d. This patient should be evaluated for a cerebellar lesion.

A

During the taking of the health history, a patient tells the nurse that "it feels like the room is spinning around me." The nurse would document this finding as: a. Vertigo. b. Syncope. c. Dizziness. d. Seizure activity.

A

Expert nurses learn to attend to a pattern of assessment data and act without consciously labeling it. These responses are referred to as: a. Intuition. b. The nursing process. c. Clinical knowledge. d. Diagnostic reasoning.

A

If a 29-year-old woman weighs 156 pounds, and the nurse determines her ideal body weight to be 120 pounds, then how would the nurse classify the woman's weight? a. Obese b. Mildly overweight c. Suffering from malnutrition d. Within appropriate range of ideal weight

A

Illness is considered part of life's rhythmic course and is an outward sign of disharmony within. This statement most accurately reflects the views about illness from which theory? a. Naturalistic b. Biomedical c. Reductionist d. Magicoreligious

A

In a person with an upper motor neuron lesion such as a cerebrovascular accident, which of these physical assessment findings should the nurse expect? a. Hyperreflexia b. Fasciculations c. Loss of muscle tone and flaccidity d. Atrophy and wasting of the muscles

A

In teaching a patient how to determine total body fat at home, the nurse includes instructions to obtain measurements of: a. Height and weight. b. Frame size and weight. c. Waist and hip circumferences. d. Mid-upper arm circumference and arm span.

A

The nurse is asking a patient for his reason for seeking care and asks about the signs and symptoms he is experiencing. Which of these is an example of a symptom? a.Chest pain b.Clammy skin c.Serum potassium level at 4.2 mEq/L d.Body temperature of 100° F

A

The nurse is assessing the mental status of a child. Which statement about children and mental status is true? a. All aspects of mental status in children are interdependent. b. Children are highly labile and unstable until the age of 2 years. c. Children's mental status is largely a function of their parents level of functioning until the age of 7 years. d. A child's mental status is impossible to assess until the child develops the ability to concentrate.

A

The nurse is assessing the vital signs of a 3-year-old patient who appears to have an irregular respiratory pattern. How should the nurse assess this child's respirations? A) Count the respirations for 1 full minute, noticing rate and rhythm. B) Check the child's pulse and respirations simultaneously for 30 seconds. C) Check the child's respirations for a minimum of 5 minutes to identify any variations in respiratory pattern. D) Count the patient's respirations for 15 seconds and multiply by four to obtain the number of respirations per minute.

A

The nurse is evaluating patients for obesity-related diseases by calculating the waist-to-hip ratios. Which one of these patients would be at increased risk? A) 29-year-old woman whose waist is 33 inches and whose hips are 36 inches B) 32-year-old man whose waist is 34 inches and whose hips are 36 inches C) 38-year-old man whose waist is 35 inches and whose hips are 38 inches D) 46-year-old woman whose waist is 30 inches and whose hips are 38 inches

A

The nurse is evaluating patients for obesity-related diseases by calculating the waist-to-hip ratios. Which one of these patients would be at increased risk? a. 29-year-old woman whose waist measures 33 inches and hips measure 36 inches b. 32-year-old man whose waist measures 34 inches and hips measure 36 inches c. 38-year-old man whose waist measures 35 inches and hips measure 38 inches d. 46-year-old woman whose waist measures 30 inches and hips measure 38 inches

A

The nurse is obtaining a history from a 30-year-old male patient and is concerned about health promotion activities. Which of these questions would be appropriate to use to assess health promotion activities for this patient? a."Do you perform testicular self-examinations?" b."Have you ever noticed any pain in your testicles?" c."Have you had any problems with passing urine?" d."Do you have any history of sexually transmitted diseases?"

A

The nurse is performing a nutritional assessment on an 80-year-old patient. The nurse knows that physiologic changes can directly affect the nutritional status of the older adult and include: a. Slowed gastrointestinal motility. b. Hyperstimulation of the salivary glands. c. Increased sensitivity to spicy and aromatic foods. d. Decreased gastrointestinal absorption causing esophageal reflux.

A

The nurse is performing a nutritional assessment on an 80-year-old patient. The nurse knows that physiological changes that directly affect the nutritional status of the elderly include: A) slowed gastrointestinal motility. B) hyperstimulation of the salivary glands. C) an increased sensitivity to spicy and aromatic foods. D) decreased gastrointestinal absorption causing esophageal reflux.

A

The nurse is preparing to measure the vital signs of a 6-month-old infant. Which action by the nurse is correct? A) Measure respirations and then pulse and temperature. B) Measure vital signs more frequently than in an adult. C) Explain procedures and encourage the infant to handle the equipment. D) Allow the infant to become familiar with the nurse by performing the physical examination first and then measuring the vital signs.

A

The nurse is preparing to use a stethoscope for auscultation. Which statement is true regarding the diaphragm of the stethoscope? The diaphragm: a. Is used to listen for high-pitched sounds. b. Is used to listen for low-pitched sounds. c. Should be lightly held against the persons skin to block out low-pitched sounds. d. Should be lightly held against the persons skin to listen for extra heart sounds and murmurs.

A

The nurse is providing care for a 68-year-old woman who is complaining of constipation. What concern exists regarding her nutritional status? a. Absorption of nutrients may be impaired. b. Constipation may represent a food allergy. c. The patient may need emergency surgery to correct the problem. d. Gastrointestinal problems will increase her caloric demand.

A

The nurse is reviewing a patient's medical record and notes that he is in a coma. Using the Glasgow Coma Scale, which number indicates that the patient is in a coma? a. 6 b. 12 c. 15 d. 24

A

The nurse is reviewing percussion techniques with a newly graduated nurse. Which technique, if used by the new nurse, indicates that more review is needed? a. Percussing once over each area b. Quickly lifting the striking finger after each stroke c. Striking with the fingertip, not the finger pad d. Using the wrist to make the strikes, not the arm

A

The nurse is reviewing the principles of nociception. During which phase of nociception does the conscious awareness of a painful sensation occur? A) Perception B) Modulation C) Transduction D) Transmission

A

The nurse is reviewing venous blood flow patterns. Which of these statements best describes the mechanism(s) by which venous blood returns to the heart? a. Intraluminal valves ensure unidirectional flow toward the heart. b. Contracting skeletal muscles milk blood distally toward the veins. c. High-pressure system of the heart helps facilitate venous return. d. Increased thoracic pressure and decreased abdominal pressure facilitate venous return to the heart.

A

The nurse is testing the deep tendon reflexes of a 30-year-old woman who is in the clinic for an annual physical examination. When striking the Achilles heel and quadriceps muscle, the nurse is unable to elicit a reflex. The nurse's next response should be to: a. Ask the patient to lock her fingers and pull. b. Complete the examination, and then test these reflexes again. c. Refer the patient to a specialist for further testing. d. Document these reflexes as 0 on a scale of 0 to 4+.

A

The nurse knows that determining whether a person is oriented to his or her surroundings will test the functioning of which structure(s)? a. Cerebrum b. Cerebellum c. CNs d. Medulla oblongata

A

The nurse knows that developing appropriate nursing interventions for a patient relies on the appropriateness of the __________ diagnosis. a. Nursing b. Medical c. Admission d. Collaborative

A

The nurse knows that one advantage of the tympanic thermometer is that: A) its rapid measurement is useful for uncooperative younger children. B) it is the most accurate method for measuring temperature in newborn infants. C) it is an inexpensive means of measuring temperature. D) studies strongly support use of the tympanic route in children under age 6 years.

A

The nurse recognizes that working with children with a different cultural perspective may be especially difficult because: a. Children have spiritual needs that are influenced by their stages of development. b. Children have spiritual needs that are direct reflections of what is occurring in their homes. c. Religious beliefs rarely affect the parents' perceptions of the illness. d. Parents are often the decision makers, and they have no knowledge of their children's spiritual needs.

A

The nurse recognizes which of these persons is at greatest risk for undernutrition? a. 5-month-old infant b. 50-year-old woman c. 20-year-old college student d. 30-year-old hospital administrator

A

The patients record, laboratory studies, objective data, and subjective data combine to form the: a. Data base. b. Admitting data. c. Financial statement. d. Discharge summary.

A

To accurately assess a rectal temperature on an adult, the nurse would: A) use a lubricated blunt tip thermometer. B) insert the thermometer 2 to 3 inches into the rectum. C) leave the thermometer in place up to 8 minutes if the patient is febrile. D) wait 2 to 3 minutes if the patient has recently smoked a cigarette.

A

When assessing a patient's pulse the nurse should also notice which of these characteristics? A) Force B) Pallor C) Capillary refill time D) Timing in the cardiac cycle

A

When considering a nutritional assessment, the nurse is aware that the most common anthropometric measurements include: a. Height and weight. b. Leg circumference. c. Skinfold thickness of the biceps. d. Hip and waist measurements.

A

When evaluating the temperature of older adults, the nurse remembers which aspect about an older adult's body temperature? A) It is lower than that of younger adults. B) It is about the same as that of a young child. C) It depends on the type of thermometer used. D) It varies widely because of less effective heat control mechanisms.

A

When percussing over the liver of a patient, the nurse notices a dull sound. The nurse should: a. Consider this a normal finding. b. Palpate this area for an underlying mass. c. Reposition the hands, and attempt to percuss in this area again. d. Consider this finding as abnormal, and refer the patient for additional treatment.

A

When preparing to perform a physical examination on an infant, the nurse should: a. Have the parent remove all clothing except the diaper on a boy. b. Instruct the parent to feed the infant immediately before the examination. c. Encourage the infant to suck on a pacifier during the abdominal examination. d. Ask the parent to leave the room briefly when assessing the infants vital signs.

A

When reviewing the demographics of ethnic groups in the United States, the nurse recalls that the largest and fastest growing population is: a. Hispanic. b. Black. c. Asian. d. American Indian.

A

Which of these techniques uses the sense of touch to assess texture, temperature, moisture, and swelling when the nurse is assessing a patient? a. Palpation b. Inspection c. Percussion d. Auscultation

A

Which technique is correct when the nurse is assessing the radial pulse of a patient? Count the: A) pulse for 1 minute if the rhythm is irregular. B) pulse for 15 seconds and multiply by four, if the rhythm is regular. C) initial pulse for a full 2 minutes to detect any variation in amplitude. D) pulse for 10 seconds and multiply by six, if the patient has no history of cardiac abnormalities.

A

During a well-baby checkup, a mother is concerned because her 2-month-old infant cannot hold her head up when she is pulled to a sitting position. Which response by the nurse is appropriate? a."Head control is usually achieved by 4 months of age." b."You shouldn't be trying to pull your baby up like that until she is older." c."Head control should be achieved by this time." d."This inability indicates possible nerve damage to the neck muscles."

A Head control is achieved by 4 months when the baby can hold the head erect and steady when pulled to a vertical position. The other responses are not appropriate.

The nurse suspects that a patient has hyperthyroidism, and the laboratory data indicate that the patient's T4 and T3 hormone levels are elevated. Which of these findings would the nurse most likely find on examination? a.Tachycardia b.Constipation c.Rapid dyspnea d.Atrophied nodular thyroid gland

A T4 and T3 are thyroid hormones that stimulate the rate of cellular metabolism, resulting in tachycardia. With an enlarged thyroid gland as in hyperthyroidism, the nurse might expect to find diffuse enlargement (goiter) or a nodular lump but not an atrophied gland. Dyspnea and constipation are not findings associated with hyperthyroidism.

While discussing the history of a 6-month-old infant, the mother tells the nurse that she took a significant amount of aspirin while she was pregnant. What question would the nurse want to include in the history? a. "Does your baby seem to startle with loud noises?" b. "Has your baby had any surgeries on her ears?" c. "Have you noticed any drainage from her ears?" d. "How many ear infections has your baby had since birth?

A. "Does your baby seem to startle with loud noises?" Children at risk for a hearing deficit include those exposed in utero to a variety of conditions, such as maternal rubella or to maternal ototoxic drugs.

The nurse is assessing a patient who may have hearing loss. Which of these statements is true concerning air conduction? a. Air conduction is the normal pathway for hearing. b. Vibrations of the bones in the skull cause air conduction. c. Amplitude of sound determines the pitch that is heard. d. Loss of air conduction is called a conductive hearing loss.

A. Air conduction is the normal pathway for hearing. The normal pathway of hearing is air conduction, which starts when sound waves produce vibrations on the tympanic membrane. Conductive hearing loss results from a mechanical dysfunction of the external or middle ear. The other statements are not true concerning air conduction.

A 2-week-old infant can fixate on an object but cannot follow a light or bright toy. The nurse would: a. Consider this a normal finding. b. Assess the pupillary light reflex for possible blindness. c. Continue with the examination, and assess visual fields. d. Expect that a 2-week-old infant should be able to fixate and follow an object.

A. Consider this a normal finding. By 2 to 4 weeks an infant can fixate on an object. By the age of 1 month, the infant should fixate and follow a bright light or toy.

The nurse is performing an ear examination of an 80-year-old patient. Which of these findings would be considered normal? a. High-tone frequency loss b. Increased elasticity of the pinna c. Thin, translucent membrane d. Shiny, pink tympanic membrane

A. High-tone frequency loss A high-tone frequency hearing loss is apparent for those affected with presbycusis, the hearing loss that occurs with aging. The pinna loses elasticity, causing earlobes to be pendulous. The eardrum may be whiter in color and more opaque and duller in the older person than in the younger adult.

A 68-year-old woman is in the eye clinic for a checkup. She tells the nurse that she has been having trouble reading the paper, sewing, and even seeing the faces of her grandchildren. On examination, the nurse notes that she has some loss of central vision but her peripheral vision is normal. These findings suggest that she may have: a. Macular degeneration. b. Vision that is normal for someone her age. c. The beginning stages of cataract formation. d. Increased intraocular pressure or glaucoma.

A. Macular degeneration. Macular degeneration is the most common cause of blindness. It is characterized by the loss of central vision. Cataracts would show lens opacity. Chronic open-angle glaucoma, the most common type of glaucoma, involves a gradual loss of peripheral vision. These findings are not consistent with vision that is considered normal at any age.

When performing an otoscopic examination of a 5-year-old child with a history of chronic ear infections, the nurse sees that his right tympanic membrane is amber-yellow in color and that air bubbles are visible behind the tympanic membrane. The child reports occasional hearing loss and a popping sound with swallowing. The preliminary analysis based on this information is that the child: a. Most likely has serous otitis media. b. Has an acute purulent otitis media. c. Has evidence of a resolving cholesteatoma. d. Is experiencing the early stages of perforation.

A. Most likely has serous otitis media. An amber-yellow color to the tympanic membrane suggests serum or pus in the middle ear. Air or fluid or bubbles behind the tympanic membrane are often visible. The patient may have feelings of fullness, transient hearing loss, and a popping sound with swallowing. These findings most likely suggest that the child has serous otitis media. The other responses are not correct.

During an examination, the nurse notices that the patient stumbles a little while walking, and, when she sits down, she holds on to the sides of the chair. The patient states, "It feels like the room is spinning!" The nurse notices that the patient is experiencing: a. Objective vertigo. b. Subjective vertigo. c. Tinnitus. d. Dizziness.

A. Objective vertigo. With objective vertigo, the patient feels like the room spins; with subjective vertigo, the person feels like he or she is spinning. Tinnitus is a sound that comes from within a person; it can be a ringing, crackling, or buzzing sound. It accompanies some hearing or ear disorders. Dizziness is not the same as true vertigo; the person who is dizzy may feel unsteady and lightheaded.

The nurse is examining a patient's retina with an ophthalmoscope. Which finding is considered normal? a. Optic disc that is a yellow-orange color b. Optic disc margins that are blurred around the edges c. Presence of pigmented crescents in the macular area d. Presence of the macula located on the nasal side of the retina

A. Optic disc that is a yellow-orange color The optic disc is located on the nasal side of the retina. Its color is a creamy yellow-orange to a pink, and the edges are distinct and sharply demarcated, not blurred. A pigmented crescent is black and is due to the accumulation of pigment in the choroid.

A 31-year-old patient tells the nurse that he has noticed a progressive loss in his hearing. He says that it does seem to help when people speak louder or if he turns up the volume of a television or radio. The most likely cause of his hearing loss is: a. Otosclerosis. b. Presbycusis. c. Trauma to the bones. d. Frequent ear infections.

A. Otosclerosis. Otosclerosis is a common cause of conductive hearing loss in young adults between the ages of 20 and 40 years. Presbycusis is a type of hearing loss that occurs with aging. Trauma and frequent ear infections are not a likely cause of his hearing loss

The nurse is preparing to do an otoscopic examination on a 2-year-old child. Which one of these reflects the correct procedure? a. Pulling the pinna down b. Pulling the pinna up and back c. Slightly tilting the child's head toward the examiner d. Instructing the child to touch his chin to his chest

A. Pulling the pinna down For an otoscopic examination on an infant or on a child under 3 years of age, the pinna is pulled down. The other responses are not part of the correct procedure.

The nurse is testing a patient's visual accommodation, which refers to which action? a. Pupillary constriction when looking at a near object b. Pupillary dilation when looking at a far object c. Changes in peripheral vision in response to light d. Involuntary blinking in the presence of bright light

A. Pupillary constriction when looking at a near object The muscle fibers of the iris contract the pupil in bright light and accommodate for near vision, which also results in pupil constriction. The other responses are not correct.

The nurse assesses the hearing of a 7-month-old by clapping hands. What is the expected response? The infant: a. Turns his or her head to localize the sound. b. Shows no obvious response to the noise. c. Shows a startle and acoustic blink reflex. d. Stops any movement, and appears to listen for the sound.

A. Turns his or her head to localize the sound. With a loud sudden noise, the nurse should notice the infant turning his or her head to localize the sound and to respond to his or her own name. A startle reflex and acoustic blink reflex is expected in newborns; at age 3 to 4 months, the infant stops any movement and appears to listen.

The nurse is assessing a patient with a history of intravenous drug abuse. In assessing his mouth, the nurse notices a dark red confluent macule on the hard palate. This could be an early sign of: a.Acquired immunodeficiency syndrome (AIDS). b.Measles. c.Leukemia. d.Carcinoma.

A. acquired immunodeficiency syndrome (AIDS)

The nurse is performing an assessment on a 21-year-old patient and notices that his nasal mucosa appears pale, gray, and swollen. What would be the most appropriate question to ask the patient? a."Are you aware of having any allergies?" b."Do you have an elevated temperature?" c."Have you had any symptoms of a cold?" d."Have you been having frequent nosebleeds?"

A. are you aware of having any allergies?

The nurse needs to pull the portion of the ear that consists of movable cartilage and skin down and back when administering eardrops. This portion of the ear is called the: a. Auricle. b. Concha. c. Outer meatus. d. Mastoid process.

A. auricle The external ear is called the auricle or pinna and consists of movable cartilage and skin.

When performing the corneal light reflex assessment, the nurse notes that the light is reflected at 2 o'clock in each eye. The nurse should: a. Consider this a normal finding. b. Refer the individual for further evaluation. c. Document this finding as an asymmetric light reflex. d. Perform the confrontation test to validate the findings.

A. consider this a normal finding Reflection of the light on the corneas should be in exactly the same spot on each eye, or symmetric. If asymmetry is noted, then the nurse should administer the cover test.

During an assessment of a 20-year-old patient with a 3-day history of nausea and vomiting, the nurse notices dry mucosa and deep vertical fissures in the tongue. These findings are reflective of: a.Dehydration. b.Irritation by gastric juices. c.A normal oral assessment. d.Side effects from nausea medication

A. dehydration

A patient's vision is recorded as 20/80 in each eye. The nurse interprets this finding to mean that the patient: a. Has poor vision. b. Has acute vision. c. Has normal vision. d. Is presbyopic.

A. has poor vision Normal visual acuity is 20/20 in each eye; the larger the denominator, the poorer the vision.

The nurse is performing an oral assessment on a 40-year-old Black patient and notices the presence of a 1 cm, nontender, grayish-white lesion on the left buccal mucosa. Which one of these statements is true? This lesion is: a.Leukoedema and is common in dark-pigmented persons. b.The result of hyperpigmentation and is normal. c.Torus palatinus and would normally be found only in smokers. d.Indicative of cancer and should be immediately tested.

A. leukoedema and is common in dark-pigmented persons

The salivary gland that is the largest and located in the cheek in front of the ear is the _________ gland. a.Parotid b.Stensen's c.Sublingual d.Submandibular

A. parotid

When assessing the tongue of an adult, the nurse knows that an abnormal finding would be: a.Smooth glossy dorsal surface. b.Thin white coating over the tongue. c.Raised papillae on the dorsal surface. d.Visible venous patterns on the ventral surface.

A. smooth glossy dorsal surface

The nurse is performing an external eye examination. Which statement regarding the outer layer of the eye is true? a. The outer layer of the eye is very sensitive to touch. b. The outer layer of the eye is darkly pigmented to prevent light from reflecting internally. c. The trigeminal nerve (CN V) and the trochlear nerve (CN IV) are stimulated when the outer surface of the eye is stimulated. d. The visual receptive layer of the eye in which light waves are changed into nerve impulses is located in the outer layer of the eye.

A. the outer layer of the eye is very sensitive to touch The cornea and the sclera make up the outer layer of the eye. The cornea is very sensitive to touch. The middle layer, the choroid, has dark pigmentation to prevent light from reflecting internally. The trigeminal nerve (CN V) and the facial nerve (CN VII) are stimulated when the outer surface of the eye is stimulated. The retina, in the inner layer of the eye, is where light waves are changed into nerve impulses.

A 45-year-old man is in the clinic for a routine physical. During the history the patient states he's been having difficulty sleeping. "I'll be sleeping great and then I wake up and feel like I can't get my breath." The nurse's best response to this would be: a. "When was your last electrocardiogram?" b. "It's probably because it's been so hot at night." c. "Do you have any history of problems with your heart?" d. "Have you had a recent sinus infection or upper respiratory infection?"

ANS: "Do you have any history of problems with your heart?" Paroxysmal nocturnal dyspnea occurs with heart failure. Lying down increases volume of intrathoracic blood, and the weakened heart cannot accommodate the increased load. Classically, the person awakens after 2 hours of sleep, arises, and flings open a window with the perception of needing fresh air.

Which of these statements is most appropriate when the nurse is obtaining a genitourinary history from an elderly man?

ANS: "Do you need to get up at night to urinate?" The elderly male patient should be asked about the presence of nocturia. This may be due to diuretic medication, fluid retention from mild heart failure or varicose veins, or fluid ingestion 3 hours before bedtime, especially coffee and alcohol. The other questions are more appropriate for younger males.

The nurse is assessing an elderly woman and suspects abuse. Which questions are appropriate for screening for abuse? Select all that apply.

ANS: "Has anyone ever physically hurt you?", "Are you alone a lot?", "Are you afraid of anybody at home or anyone who enters your home?", "Has anyone ever failed to help you take care of yourself when you needed help?" Directly asking "Are you being abused?" is not appropriate for a screening question for abuse because the woman could easily say "no" and no further information would be obtained. The other questions are among the questions recommended by the American Medical Association when screening for elder abuse.

The nurse is describing how to perform a testicular self-examination to a patient. Which of these statements is most appropriate?

ANS: "If you notice an enlarged testicle or a painless lump, call your health care provider." If the patient notices a firm painless lump, a hard area, or an overall enlarged testicle, he should call his health care provider for further evaluation. The testicle normally feels rubbery with a smooth surface. A good time to examine the testicles is during the shower or bath, when one's hands are warm and soapy, and the scrotum is warm. It should be performed once a month.

When the nurse is conducting sexual history from a male adolescent, which statement would be most appropriate to use at the beginning of the interview?

ANS: "Often boys your age have questions about sexual activity." Start the interview with a permission statement. This conveys that it is normal and all right to think or feel a certain way. Avoid sounding judgmental.

During an interview, a woman has answered "yes" to two of the Abuse Assessment Screen questions. What should the nurse say next?

ANS: "Tell me about this abuse in your relationship." If a woman answers "yes" to any of the Abuse Assessment Screen questions, then the nurse should ask questions designed to assess how recent and how serious the abuse was. Asking the woman an open-ended question, such as "tell me about this abuse in your relationship" is a good way to start.

The mother of a 10-year-old boy asks the nurse to discuss the recognition of puberty. The nurse should reply by saying:

ANS: "The first sign of puberty is enlargement of the testes." Puberty begins sometime between ages 9 1/2 and 13 1/2 years. The first sign is enlargement of the testes. Next, pubic hair appears and then penis size increases.

Which statement is best for the nurse to use when preparing to administer the Abuse Assessment Screen?

ANS: "We ask the following questions because domestic violence is so common in our society." Such an introduction both alerts the woman that questions about domestic violence are coming and makes sure that the woman knows she is not being singled out for these questions.

The nurse is interviewing a patient who has a hearing impairment. What techniques would be most beneficial in communicating with this patient? 1.Assess the communication method he prefers. 2.Avoid using facial and hand gestures because most hearing-impaired people find this degrading. 3.Request a sign language interpreter before meeting with him to help facilitate the communication. 4.Speak loudly and with exaggerated facial movement when talking with him because this helps with lip reading.

ANS: 1 Ask the deaf person the preferred way to communicate—by signing, lip reading, or writing. If the person prefers lip reading, be sure to face him or her squarely and have good lighting on your face. Do not exaggerate your lip movements because this distorts your words. Similarly, shouting distorts the reception of a hearing aid the person may wear. Speak slowly and supplement your voice with appropriate hand gestures or pantomime.

The nurse is interviewing a recent male immigrant from Mexico. During the course of the interview, he leans forward and then finally moves his chair close enough that his knees are nearly touching. The nurse begins to feel uncomfortable with his proximity. Which statement most closely reflects what the nurse should do next? 1.Try to relax—these behaviors are culturally appropriate for this person. 2.Discreetly move one's chair back until the distance is more comfortable and then continue with the interview. 3.These behaviors are indicative of sexual aggression, and the nurse should confront this person about them. 4.The nurse should laugh but tell him that he or she is uncomfortable with his proximity and ask him to move away.

ANS: 1 Both the patient's and one's own sense of spatial distance are significant throughout the interview and physical examination, with culturally appropriate distance zones varying widely. For example, you may find yourself backing away from people of Mexican-American, Indian, or Middle Eastern origins who invade your personal space with regularity in an attempt to bring you closer into the space that is comfort- able to them.

As the nurse enters a patient's room, the nurse finds her crying. The patient states that she has just found out that the lump in her breast is cancer and says, "I'm so afraid of, um, you know." The nurse's most therapeutic response would be to say, in a gentle manner: 1."You're afraid you might lose your breast?" 2."No, I'm not sure what you are talking about." 3."I'll wait here until you get yourself under control and then we can talk." 4."I can see that you are very upset. Perhaps we should discuss this later."

ANS: 1 Reflection echoes the patient's words, repeating part of what the person just said. Reflection also can help express the feeling behind a person's words.

When taking a history from a newly admitted patient, the nurse notices that he is pausing often and looking at the nurse expectantly. What would be the nurse's best response to this behavior? 1.Be silent and allow him to continue when he is ready. 2.Smile at him and say, "Don't worry about all of this. I'm sure we can find out why you're having these pains." 3.Lean back in the chair and ask, "Mr. J., you are looking at me kind of funny; there isn't anything wrong, is there?" 4.Stand up and say, "I can see that this interview is uncomfortable for you. We can continue it another time."

ANS: 1 Silent attentiveness communicates that the person has time to think, to organize what he or she wishes to say without interruption from you. This "thinking silence" is the one health professionals interrupt most often.

In an interview, the nurse may find it necessary to take notes to aid his or her memory later. Which of the following statements is true regarding note-taking? 1.Taking notes may impede the nurse's observation of the patient's nonverbal behaviors. 2.Taking notes allows the patient to continue at his or her own pace as the nurse records what is said. 3.Taking notes allows the nurse to shift attention away from the patient, resulting in increased comfort level. 4.Taking notes allows the nurse to break eye contact with the patient, which may increase his or her level of comfort.

ANS: 1 Some use of history forms and note-taking may be unavoidable. But be aware that note-taking during the interview has disadvantages. It breaks eye contact too often; it shifts attention away from the patient, diminishing his or her sense of importance. It can interrupt the patient's narrative flow, and it impedes the observation of the patient's nonverbal behavior.

During a follow-up visit, the nurse discovers that a patient has not been taking his insulin on a regular basis. The nurse asks, "Why haven't you taken your insulin?" Which of the following is an appropriate evaluation of this question? 1.It may put the patient on the defensive. 2.It is an innocent search for information. 3.It would have been better to discuss this with his wife. 4.It is the best way to discover the reasons for his behavior.

ANS: 1 The adult's use of "why" questions usually implies blame and condemnation and puts the person on the defensive.

A 75-year-old woman is at the office for a preoperative interview. The nurse is aware that the interview may take longer than interviews with younger persons. What is the reason for this? 1.An aged person has a longer story to tell. 2.An aged person is usually lonely and likes to have someone to talk to. 3.Aged persons lose much of their mental abilities and require greater time to complete an interview. 4.As a person ages, they are unable to hear and thus interviewers usually need to repeat much of what is said.

ANS: 1 The interview usually takes longer with older adults because they have a longer story to tell.

he nurse has just started an assessment of a newborn child of a Vietnamese mother. Considering the mother's cultural background, which statement is true regarding this examination? The mother: 1. will be offended if the infant's fontanels are examined. 2. will be offended if the infant's diaper area is touched during the examination. 3. would prefer to have the results of the examination communicated directly to her husband. 4. would prefer to receive written material about growth and development rather than a verbal explanation.

ANS: 1 Touching children may also have associated meaning cross-culturally. Many Asians believe that one's strength resides in the head and touching the head is considered disrespectful. Thus, palpating the fontanel of an infant from Southeast Asian descent should be approached with sensitivity.

A female patient does not speak English well, and the nurse needs to choose an interpreter. Which of the following would be the most appropriate choice? 1. A trained interpreter 2. A male family member 3. A female family member 4. A volunteer college student from the foreign language studies department.

ANS: 1 Whenever possible, use a trained interpreter, preferably one who knows medical terminology. In general, an older, more mature interpreter is preferred to a younger, less experienced one, and the same sex is preferred when possible.

The nurse is assessing a patient's pulses and notices a difference between the patient's apical pulse and radial pulse. The apical pulse was 118 beats per minute, and the radial pulse was 105 beats per minute. What is the pulse deficit?

ANS: 13 The nurse should count a serial measurement (one after the other) of apical beat and then the radial pulse. Normally every beat heard at the apex should perfuse to the periphery and be palpable. The two counts should be identical. If different, the nurse should subtract the radial rate from the apical and record the remainder as the pulse deficit.

During a prenatal check, a patient begins to cry as the nurse asks her about previous pregnancies. She states that she is remembering her last pregnancy, which ended in miscarriage. The nurse's best response to her crying would be: 1."I'm so sorry for making you cry!" 2."I can see that you are sad remembering this. It is all right to cry." 3."Why don't I step out for a few minutes until you're feeling better?" 4."I can see that you feel sad about this; why don't we talk about something else?"

ANS: 2 A beginning examiner usually feels horrified when the patient starts crying. When you say something that "makes the person cry," do not think you have hurt the person. You have just hit on a topic that is important. Do not go on to a new topic. Just let the person cry and express his or her feelings fully. You can offer a tissue and wait until the crying subsides to talk.

A mother brings her 28-month-old daughter into the clinic for a well-child visit. At the beginning of the visit, the nurse focuses attention away from the toddler, but as the interview progresses, the toddler begins to "warm up" and is smiling shyly at the nurse. The nurse will be most successful in interacting with the toddler if which is done next? 1.Tickle the toddler and get her to laugh. 2.Stoop down to her level and ask her about the toy she is holding. 3.Continue to ignore her until it is time for the physical examination. 4.Ask the mother. to leave during the examination of the toddler because toddlers often fuss less if their parents aren't in view.

ANS: 2 Although most of the communication is with the parent, do not ignore the child completely. Making contact to will assist in easing into the physical examination later. Begin by asking about the toys the child is playing with or about a special doll or teddy bear brought from home: "Does your doll have a name?" or "What can your truck do?" Stoop down to meet the child at his or her eye level.

The nurse is conducting an interview with a woman who has recently learned that she is pregnant and has come to the clinic today to begin prenatal care. The woman states that she and her husband are excited about the pregnancy but have a few questions. She looks nervously at her hands during the interview and sighs loudly. Considering the concept of communication, the nurse knows that which statement is most accurate? The woman: 1. is excited about her pregnancy but nervous about labor. 2. is exhibiting verbal and nonverbal behavior that does not match. 3. is excited about her pregnancy but her husband is not and this is upsetting to her. 4. is not excited about her pregnancy but believes the nurse will respond negatively to her if she states this.

ANS: 2 Communication is all behavior, conscious and unconscious, verbal and nonverbal. All behavior has meaning.

The nurse has used interpretation regarding a patient's statement or actions. After using this technique, it would be best for the nurse to: 1.apologize because this can be demeaning for the patient. 2.allow the patient time to confirm or correct the inference. 3.continue with the interview as though nothing had happened. 4.immediately restate the nurse's conclusion on the basis of the patient's nonverbal response.

ANS: 2 Interpretation is not based on direct observation as is confrontation, but it is based on one's inference or conclusion. You do run a risk of making the wrong inference. If this is the case, the person will correct it. But even if the inference is corrected, interpretation helps to prompt further discussion of the topic.

A pregnant woman states, "I just know labor will be so painful that I won't be able to stand it. I know it sounds awful, but I really dread going into labor." The nurse responds by stating, "Oh, don't worry about labor so much. I have been through it and although it is painful there are many good medications to decrease the pain." Which statement is true regarding this response? 1.It was a therapeutic response. By sharing something personal, the nurse gives hope to this woman. 2.It was a nontherapeutic response. By providing false reassurance, the nurse actually cut off further discussion of the woman's fears. 3.It was a therapeutic response. By providing information about the medications available, the nurse is giving information to the woman. 4.It was a nontherapeutic response. The nurse is essentially giving the message to the woman that labor cannot be tolerated without medication.

ANS: 2 Providing false assurance or reassurance, this "courage builder," relieves one's anxiety and gives you the false sense of having provided comfort. But for the woman, it actually closes off communication. It trivializes her anxiety and effectively denies any further talk of it.

A 16-year-old boy has just been admitted to the unit for overnight observation after being in an automobile accident. What is the nurse's best approach to communicating with him? 1. Use periods of silence to communicate respect for him. 2. Be totally honest with him, even if the information is unpleasant. 3. Tell him that everything that is discussed will be kept totally confidential. 4. Use slang language when possible to help him open up.

ANS: 2 Successful communication (with adolescents) is possible and rewarding. The guidelines are simple. The first consideration is one's attitude, which must be one of respect. Second, communication must be totally honest. Adolescents' intuition is highly tuned and can detect phoniness or withholding of information. Always give them the truth.

Which of the following is appropriate for the nurse to say near the end of the interview? 1."Did we forget something?" 2."Is there anything else you would like to mention?" 3."I need to go on to the next patient. I'll be back." 4."While I'm here, let's talk about your upcoming surgery."

ANS: 2 This question gives the person the final opportunity for self-expression. No new topic should be introduced.

A woman is discussing the problems she is having with her 2-year-old son. "He won't go to sleep at night and during the day he has several fits. I get so upset when that happens." The nurse's best verbal response would be: 1."Go on, I'm listening." 2."Fits? Tell me what you mean by this." 3."Yes, it can be upsetting when a child has a fit." 4."Don't be upset when he has a fit; every 2-year-old has fits."

ANS: 2 Use clarification when the person's word choice is ambiguous or confusing (e.g., "Tell me what you mean by 'tired blood.'") Clarification is also used to summarize the person's words, simplify the words to make them clearer, and then ask if you are on the right track.

During a clinic visit, a patient states, "The doctor just told me he thought I ought to stop smoking. He doesn't understand how hard I've tried. I just don't know the best way to do it. What should I do?" The nurse's most appropriate response in this case would be: 1."I'd quit. The doctor really knows what he is talking about." 2."Would you like some information about the different ways a person can quit smoking?" 3."Stopping your dependence on cigarettes can be very difficult. I understand how you feel." 4."Why are you confused? Didn't the doctor give you the information about the smoking cessation program we offer?"

ANS: 2 Use clarification when the person's word choice is ambiguous or confusing. Clarifi- cation is also used to summarize the person's words, simplify the words to make them clearer, and then ask if you are on the right track.

A 17-year-old single mother is describing how difficult it is to raise a 2-year-old by herself. During the course of the interview she states, "I can't believe my boyfriend left me to do this by myself! What a terrible thing to do to me!" Which of the following responses by the nurse uses empathy? 1."You feel alone?" 2."You can't believe he left you alone." 3."It must be so hard to face this all alone." 4."I would be angry, too; raising a child alone is no picnic."

ANS: 3 An empathetic response recognizes the feeling and puts it into words. It names the feeling and allows the expression of it. It strengthens rapport. Other empathetic responses are, "This must be very hard for you," "I understand," or just placing your hand on the person's arm.

Receiving is a part of the communication process. Which receiver is most likely to misinterpret a message sent by a healthcare professional? 1.A well-adjusted adolescent in for a sports physical 2.A recovering alcoholic in for a basic physical examination 3.A man whose wife has just been diagnosed with lung cancer 4.A hearing-impaired man who uses sign language to communicate and has an interpreter with him

ANS: 3 The receiver attaches meaning determined by his or her past experiences, culture, self-concept, and current physical and emotional states.

During an interview, a woman says, "I have decided that I can no longer allow my children to live with their father's violence. I just can't seem to leave him though." Using interpretation, the nurse's best response would be: 1."You're going to leave him?" 2."If you're afraid for your children, why can't you leave?" 3."It sounds as if you might be afraid of how your husband will respond." 4."It sounds as though you have made your decision. I think it is a good one."

ANS: 3 This statement is not based on one's inference or conclusion. It links events, makes associations, or implies cause. Interpretation also ascribes feelings and helps the person understand his or her own feelings in relation to the verbal message.

Critique the following statement made by the nurse: "I know it may be hard, but you should do what the doctor ordered because he/she is the expert in this field." 1. This statement is inappropriate because it shows the nurse's bias. 2. This statement is appropriate because members of the health care team are experts in the area of patient care. 3. This type of statement promotes dependency and inferiority on the part of the patient and is best avoided in an interview situation. 4. At times, it is necessary to use authority statements when dealing with patients, especially when they are undecided about an issue.

ANS: 3 Using authority responses promotes dependency and inferiority. It is best to avoid using authority. Although the health care provider and patient do not have equal professional knowledge, both have equally worthy roles in the health process.

A nurse is taking complete health histories on all the patients attending a wellness workshop. On the history form, one of the written question asks, "You don't smoke, drink, or take drugs, do you?" This question is an example of: 1.talking too much. 2.using confrontation. 3.using biased or leading questions. 4.using blunt language to deal with distasteful topics.

ANS: 3 Using leading or biased questions. Asking, "You don't smoke, do you?" implies that one answer is "better" than another. If the person wants to please you, he or she is either forced to answer in a way corresponding to your values or is made to feel guilty when admitting the other answer.

A woman, Mrs. H., has just entered the emergency department after being battered by her husband. The nurse will need to get some information from her to begin treatment. What is the best choice for an opening with this patient? 1."Nancy, my name is Mrs. C." 2."Hello, Mrs. H., my name is Mrs. C. It sure is cold today!" 3."Mrs. H., my name is Mrs. C. You sure look like you're in pain, can I get you anything?" 4."Mrs. H., my name is Mrs. C. I'll need to ask you a few questions about what happened."

ANS: 4 Address the person, using his or her surname. Introduce yourself and state your role in the agency. Friendly small talk is not needed to build rapport.

An American Indian woman has come to the clinic for diabetic follow-up teaching. During the interview, the nurse notices that she never makes eye contact and speaks mostly to the floor. Which statement is true regarding this situation? 1. She is nervous and embarrassed. 2. She has something to hide and is ashamed. 3. She is showing inconsistent verbal and nonverbal behaviors. 4. She is showing that she is listening carefully to what the nurse is saying.

ANS: 4 Eye contact is perhaps among the most culturally variable nonverbal behaviors. Asian, American Indian, Indochinese, Arab, and Appalachian people may consider direct eye contact impolite or aggressive, and they may avert their own eyes during the interview. American Indians often stare at the floor during the interview, a culturally appropriate behavior indicating that the listener is paying close attention to the speaker.

A man has been admitted to the observation unit for observation after being treated for a large cut on his forehead. As the nurse works through the interview, one of the standard questions has to do with alcohol, tobacco, and drug use. When the nurse asks him about tobacco use, he states, "I quit smoking after my wife died 7 years ago." However, the nurse notices an open package of cigarettes in his shirt pocket. Using confrontation, the nurse could say: 1."Mr. K., I know that you are lying." 2."Mr. K., come on, tell me how much you smoke." 3."Mr. K., I didn't realize your wife had died. It must be difficult for you at this time. Please tell me more about that." 4."Mr. K., you have said that you don't smoke, but I see that you have an open package of cigarettes in your pocket."

ANS: 4 In the case of confrontation, a certain action, feeling, or statement has been observed, and you now focus the person's attention on it. You give your honest feedback about what you see or feel. This may focus on a discrepancy. Or you may confront the person when you notice parts of the story are inconsistent.

During an interview, a parent of a hospitalized child is sitting in an open position. As the interviewer begins to discuss his son's treatment, however, he suddenly crosses his arms against his chest and crosses his legs. This would suggest that the parent is: 1.just changing positions. 2.more comfortable in this position. 3.tired and needs a break in the interview. 4.uncomfortable talking about his son's treatment.

ANS: 4 Note the patient's position. An open position with the extension of large muscle groups shows relaxation, physical comfort, and a willingness to share information. A closed position with the arms and legs crossed tends to look defensive and anxious. Note any change in posture. If a person in a relaxed position suddenly tenses, it suggests possible discomfort with the new topic.

A man arrives at the clinic for an annual wellness physical. He is experiencing no acute health problems. Which of the following statements by the nurse is most appropriate when beginning the interview? 1."How is your family?" 2."Why are you here today?" 3."Tell me about your hypertension?" 4."How has your health been since your last visit?"

ANS: 4 Open-ended questions are used for narrative information. Use this type of question- ing to begin the interview, to introduce a new section of questions, and whenever the person introduces a new topic.

The interview portion of data collection collects: 1.physical data. 2.historical data. 3.objective data. 4.subjective data.

ANS: 4 The interview is the first, and really the most important part of data collection, collecting subjective data—what the person says about himself or herself.

A female nurse is interviewing a male patient who is near the same age as the nurse. During the interview, the patient makes an overtly sexual comment. The nurse's best reaction would be: 1."Stop that immediately!" 2."Oh, you are too funny. Let's keep going with the interview." 3."Do you really think I'd be interested?" 4."It makes me uncomfortable when you talk that way. Please don't."

ANS: 4 The nurse's response must make it clear that she is a health professional who can best care for the person by maintaining a professional relationship. At the same time, the nurse should communicate that she accepts the person and understands the person's need to be self-assertive but that sexual advances cannot be tolerated.

During an interview, the nurse states: "You mentioned shortness of breath. Tell me more about that." Identify the verbal skill used with this statement. 1.Reflection 2.Facilitation 3.Direct question 4.Open-ended question

ANS: 4 The open-ended question asks for narrative information. It states the topic to be discussed but only in general terms. Use it to begin the interview, to introduce a new section of questions, and whenever the person introduces a new topic.

A patient has finished giving the nurse information about the reason he is seeking care. When reviewing the data, the nurse finds that some information about past hospitalizations is missing. Which statement by the nurse would be most appropriate to gather these data? 1."Mr. Y., at your age, surely you have been hospitalized before!" 2."Mr. Y., I just need permission to get your medical records from County Medical." 3."Mr. Y., you mentioned that you have been hospitalized on several occasions. Would you tell me more about that?" 4."Mr. Y., I just need to get some additional information about your past hospitaliza- tions. When was the last time you were admitted for chest pain?"

ANS: 4 Use direct questions after the person's opening narrative to fill in any details he or she left out. Also use direct questions when specific facts are needed, such as when asking about past health problems or during the review of system

23. During an examination of a 3-year-old child, the nurse will need to take her blood pressure. What might the nurse do to try to gain the child's full cooperation? 1.Tell the child that the blood pressure cuff is going to give her arm a big hug. 2.Tell the child that the blood pressure cuff is asleep and cannot wake up. 3.Give the blood pressure cuff a name and refer to it by this name during the assessment. 4.Tell the child that by using the blood pressure cuff, we can see how strong her muscles are.

ANS: 4 Use short, simple sentences with a concrete explanation. Take time to give a short, simple explanation for any unfamiliar equipment that will be used on the child. Preschoolers are animistic; they imagine inanimate objects can come alive and have human characteristics. Thus a blood pressure cuff can wake up and bite or pinch.

When observing a patient's verbal and nonverbal communication, the nurse notices a discrepancy. Which statement is true regarding this situation? 1.The nurse should ask someone who knows the patient well to help interpret this discrepancy. 2.The nurse should focus on the patient's verbal message and try to ignore the nonverbal behaviors. 3.The nurse should try to integrate the verbal and nonverbal messages and then interpret them as an "average." 4.The nurse should focus on the patient's nonverbal behaviors because these are often more reflective of a patient's true feelings.

ANS: 4 When nonverbal and verbal messages are congruent, the verbal is reinforced. When they are incongruent, the nonverbal message tends to be the true one because it is under less conscious control. Thus it is important to study the nonverbal messages of patients and examiners and to understand their meanings.

In using verbal responses to assist the patient's narrative, some responses focus on the patient's frame of reference and some focus on the health care provider's perspective. An example of a verbal response that focuses on the health care provider's perspective would be: 1.empathy. 2.reflection. 3.facilitation. 4.confrontation.

ANS: 4 When using the response of confrontation, the frame or reference shifts from the patient's perspective to yours, and you start to express your own thoughts and feelings.

1. A woman has come to the clinic to seek help with a substance abuse problem. She admits to using cocaine just before arriving. Which of these assessment findings would the nurse expect to find when examining this woman? a. Dilated pupils, pacing, and psychomotor agitation b. Dilated pupils, unsteady gait, and aggressiveness c. Pupil constriction, lethargy, apathy, and dysphoria d. Constricted pupils, euphoria, and decreased temperature

ANS: A A cocaine users appearance includes pupillary dilation, tachycardia or bradycardia, elevated or lowered blood pressure, sweating, chills, nausea, vomiting, and weight loss. The persons behavior includes euphoria, talkativeness, hypervigilance, pacing, psychomotor agitation, impaired social or occupational functioning, fighting, grandiosity, and visual or tactile hallucinations.

2. The nurse is assessing a patient who has been admitted for cirrhosis of the liver, secondary to chronic alcohol use. During the physical assessment, the nurse looks for cardiac problems that are associated with chronic use of alcohol, such as: a. Hypertension. b. Ventricular fibrillation. c. Bradycardia. d. Mitral valve prolapse.

ANS: A Even moderate drinking leads to hypertension and cardiomyopathy, with an increase in left ventricular mass, dilation of ventricles, and wall thinning. Ventricular fibrillation, bradycardia, and mitral valve prolapse are not associated with chronic heavy use of alcohol.

A 43-year-old woman is at the clinic for a routine examination. She reports that she has had a breast lump in her right breast for years. Recently, it has begun to change in consistency and is becoming harder. She reports that 5 years ago her physician evaluated the lump and determined that it "was nothing to worry about." The examination validates the presence of a mass in the right upper outer quadrant at 1 o'clock, approximately 5 cm from the nipple. It is firm, mobile, nontender, with borders that are not well defined. The nurse's recommendation to her is: A) "Because of the change in consistency of the lump, it should be further evaluated by a physician." B) "The changes could be related to your menstrual cycles. Keep track of changes in the mass each month." C) "This is probably nothing to worry about because it has been present for years and was determined to be noncancerous at that time." D) "Because you are experiencing no pain and the size has not changed, continue to monitor the lump and return to the clinic in 3 months."

ANS: A A lump that has been present for years and is not exhibiting changes may not be serious but still should be explored. Any recent change or new lump should be evaluated. The other responses are not correct.

During an examination, the nurse notes a supernumerary nipple just under the patient's left breast. The patient tells the nurse that she always thought it was a mole. Which statement about this finding is correct? A) It is a normal variation and not a significant finding. B) It is a significant finding and needs further investigation. C) It also contains glandular tissue and may leak milk during pregnancy and lactation. D) The patient is correct—it is actually a mole that happens to be located under the breast.

ANS: A A supernumerary nipple looks like a mole, but close examination reveals a tiny nipple and areola. It is not a significant finding.

19. A nurse notices that a patient has ascites, which indicates the presence of: A) fluid. B) feces. C) flatus. D) fibroid tumors.

ANS: A Ascites is free fluid in the peritoneal cavity, and occurs with heart failure, portal hypertension, cirrhosis, hepatitis, pancreatitis, and cancer.

29. Just before going home, a new mother asks the nurse about the infant's umbilical cord. Which of these statements is correct? A) "It should fall off by 10 to 14 days." B) "It will soften before it falls off." C) "It contains two veins and one artery." D) "Skin will cover the area within 1 week."

ANS: A At birth, the umbilical cord is white and contains two umbilical arteries and one vein inside the Wharton jelly. The umbilical stump dries within a week, hardens, and falls off by 10 to 14 days. Skin will cover the area by 3 to 4 weeks.

During a speculum inspection of the vagina, the nurse would expect to see what at the end of the vaginal canal? A) Cervix B) Uterus C) Ovaries D) Fallopian tubes

ANS: A At the end of the canal, the uterine cervix projects into the vagina

A 14-year-old girl is anxious about not having reached menarche. When taking the history, the nurse should ascertain which of the following? The age: A) she began to develop breasts B) her mother developed breasts C) she began to develop pubic hair D) she began to develop axillary hair.

ANS: A Full development from stage 2 to stage 5 takes an average of 3 years, although the range is 1.5 to 6 years. Pubic hair develops during this time, and axillary hair appears 2 years after the onset of pubic hair. The beginning of breast development precedes menarche by about 2 years. Menarche occurs in breast development stage 3 or 4, usually just after the peak of the adolescent growth spurt, which occurs around age 12 years. See Figure 17-6.

During a health history of a patient who complains of chronic constipation, the patient asks the nurse about high-fiber foods. The nurse relates that an example of a high-fiber food would be: A) broccoli. B) hamburger. C) iceberg lettuce. D) yogurt.

ANS: A High-fiber foods are either soluble type (i.e., beans, prunes, barley, broccoli) and insoluble type (i.e., cereals, wheat germ). The other examples are not considered high-fiber foods.

The mother of a 5-year-old girl tells the nurse that she has noticed her daughter "scratching at her bottom a lot the last few days." During the assessment, the nurse finds redness and raised skin in the anal area. This most likely indicates: A) pinworms. B) chickenpox. C) constipation. D) bacterial infection.

ANS: A In children, pinworms are a common cause of intense itching and irritated anal skin. The other options are not correct.

When assessing a newborn infant's genitalia, the nurse notices that the genitalia are somewhat engorged. The labia majora are swollen, the clitoris looks large, and the hymen is thick. The vaginal opening is difficult to visualize. The infant's mother states that she is worried about the labia being swollen. The nurse should reply: A) "This is a normal finding in newborns and should resolve within a few weeks." B) "This could indicate an abnormality and may need to be evaluated by a physician." C) "We will need to have estrogen levels evaluated to make sure that they are within normal limits." D) "We will need to keep close watch over the next few days to see if the genitalia decrease in size."

ANS: A It is normal for a newborn's genitalia to be somewhat engorged. A sanguineous vaginal discharge or leukorrhea is normal during the first few weeks because of the maternal estrogen effect. During the early weeks, the genital engorgement resolves, and the labia minora atrophy and remain small until puberty.

During an examination, which tests will the nurse collect to screen for cervical cancer? A) Endocervical specimen, cervical scrape, and vaginal pool B) Endocervical specimen, vaginal pool, and acetic acid wash C) Endocervical specimen, KOH preparation, and acetic acid wash D) Cervical scrape, acetic acid wash, saline mount ("wet prep")

ANS: A Laboratories may vary in method, but usually the test consists of three specimens: endocervical specimen, cervical scrape, and vaginal pool. The other tests (acetic acid wash, KOH preparation, and saline mount) are used to test for sexually transmitted infections

24. During an assessment of a newborn infant, the nurse recalls that pyloric stenosis would be manifested by: A) projectile vomiting. B) hypoactive bowel activity. C) palpable olive-sized mass in right lower quadrant. D) pronounced peristaltic waves crossing from right to left.

ANS: A Marked peristalsis together with projectile vomiting in the newborn suggests pyloric stenosis. After feeding, pronounced peristaltic waves cross from left to right, leading to projectile vomiting. One can also palpate an olive-sized mass in the right upper quadrant

A woman is in the clinic for an annual gynecologic examination. The nurse should plan to begin the interview with the: A) menstrual history because it is generally nonthreatening. B) obstetric history because it is the most important information. C) urinary system history because there may be problems in this area as well. D) sexual history because it will build rapport to discuss this first.

ANS: A Menstrual history is usually nonthreatening; thus it is a good place to start. Obstetric, urinary, and sexual histories are also part of the interview but not necessarily the best topics with which to start.

4. What should the nurse assess before entering the patient's room on morning rounds? A. Posted conditions, such as isolation precautions B. The patient's input and output chart from the previous shift C. The patient's general appearance D. The presence of any visitors in the room

ANS: A On the way to the room, the nurse should assess the presence of conditions such as isolation precautions, latex allergies, or fall precautions.

6. During an assessment of a hospitalized patient, the nurse pinches a fold of skin under the clavicle or on the forearm to test: A. Mobility and turgor. B. The patient's response to pain. C. The percentage of the patient's fat-to-muscle ratio. D. The presence of edema.

ANS: A Pinch up a fold of skin under the clavicle or on the forearm to note mobility and turgor.

The nurse is preparing to interview a postmenopausal woman. Which of these statements is true with regard to the history of a postmenopausal woman? A) The nurse should ask a postmenopausal woman if she ever has vaginal bleeding. B) Once a woman reaches menopause, the nurse does not need to ask any further history questions. C) The nurse should screen for monthly breast tenderness. D) Postmenopausal women are not at risk for contracting sexually transmitted infections and thus these questions can be omitted.

ANS: A Postmenopausal bleeding warrants further workup and referral. The other statements are not true.

The nurse is discussing breast self-examination with a postmenopausal woman. The best time for postmenopausal women to perform breast self-examination is: A) the same day every month. B) daily, during the shower or bath. C) 1 week after her menstrual period. D) every year with her annual gynecologic examination.

ANS: A Postmenopausal women are no longer experiencing regular menstrual cycles but need to continue to perform breast self-examination on a monthly basis. Choosing the same day of the month is a helpful reminder to perform breast self-examination.

When performing an external genitalia examination of a 10-year-old girl, the nurse notices that there is no pubic hair, and the mons and the labia are covered with fine vellus hair. These findings are consistent with stage _____ of sexual maturity, according to the Sexual Maturity Rating scale. A) 1 B) 2 C) 3 D) 4

ANS: A Sexual Maturity Rating stage 1 is the preadolescent stage. There is no pubic hair. The mons and labia are covered with fine, vellus hair as on the abdomen. See Table 26-1.

A 70-year-old man is visiting the clinic for difficulty in passing urine. In the history he indicates he has to urinate frequently, especially at night. He has burning when he urinates and has noticed pain in his back. Given this history, what might the nurse expect to find during the physical assessment? A) Asymmetric, hard, fixed prostate gland B) Occult blood and perianal pain to palpation C) Symmetrically enlarged, soft prostate gland D) A soft nodule protruding from rectal mucosa

ANS: A Subjective symptoms of carcinoma of the prostate include frequency, nocturia, hematuria, weak stream, hesitancy, pain or burning on urination, and continuous pain in lower back, pelvis, and thighs. Objective symptoms of carcinoma of the prostate include a malignant neoplasm often starts as a single hard nodule on the posterior surface, producing asymmetry and a change in consistency. As it invades normal tissue, multiple hard nodules appear, or the entire gland feels stone hard and fixed.

An 11-year-old girl is in the clinic for a sports physical. The nurse notices that she has begun to develop breasts, and during the conversation the girl reveals that she is worried about her development. The nurse should use which of these techniques to best assist the young girl in understanding the expected sequence for development? The nurse should: A) use the Tanner's table on the five stages of sexual development. B) describe her development and compare it with that of other girls her age. C) use Jacobsen's table on expected development on the basis of height and weight data. D) reassure her that her development is within normal limits and should tell her not to worry about the next step.

ANS: A Tanner's table on the five stages of pubic hair development is helpful in teaching girls the expected sequence of sexual development (see Table 26-1). The other responses are not appropriate.

A woman has just learned that she is pregnant. What are some things the nurse should teach her about her breasts? A) She can expect her areolae to become larger and darker in color. B) Breasts may begin secreting milk after the fourth month of pregnancy. C) She should inspect her breasts for visible veins and report this immediately. D) During pregnancy, breast changes are fairly uncommon; most of the changes occur after the birth.

ANS: A The areolae become larger and grow a darker brown as pregnancy progresses, and the tubercles become more prominent. (The brown color fades after lactation, but the areolae never return to the original color). A venous pattern is prominent over the skin surface and does not need to be reported as it is an expected finding. After the fourth month, colostrum, a thick, yellow fluid (precursor to milk) may be expressed from the breasts.

A 62-year-old man is experiencing fever, chills, malaise, urinary frequency, and urgency. He also reports urethral discharge and a dull aching pain in the perineal and rectal area. These symptoms are most consistent with which of the following? A) Prostatitis B) A polyp C) Carcinoma of the prostate D) Benign prostatic hypertrophy (BPH)

ANS: A The common presenting symptoms of prostatitis are fever, chills, malaise, and urinary frequency and urgency. The individual may also have dysuria, urethral discharge, and a dull aching pain in the perineal and rectal area. See Table 25-3 for descriptions of carcinoma of the prostate and BPH. These are not the symptoms of a polyp.

The nurse is caring for a newborn infant. Thirty hours after birth, the infant passes a dark green meconium stool. The nurse recognizes that this is important because: A) this stool would indicate anal patency. B) the dark green color could indicate occult blood in the stool. C) meconium stool can be reflective of distress in the newborn. D) the newborn should have passed the first stool within 12 hours after birth.

ANS: A The first stool passed by the newborn is dark green meconium and occurs within 24 to 48 hours of birth, indicating anal patency. The other responses are not correct

The nurse is reviewing statistics regarding breast cancer. Which woman, aged 40 years in the United States, has the highest risk for development of breast cancer? A) African-American B) White C) Asian D) American Indian

ANS: A The incidence of breast cancer varies with different cultural groups. White women have a higher incidence of breast cancer than African-American women starting at age 45 years; but African-American women have a higher incidence before age 45 years. Asian, Hispanic, and American Indian women have a lower risk for development of breast cancer (American Cancer Society, 2009-2010).

During an examination the nurse observes a female patient's vestibule and expects to see the: A) urethral meatus and vaginal orifice. B) vaginal orifice and vestibular (Bartholin) glands. C) urethral meatus and paraurethral (Skene) glands. D) paraurethral (Skene) and vestibular (Bartholin) glands.

ANS: A The labial structures encircle a boat-shaped space, or cleft, termed the vestibule. Within it are numerous openings. The urethral meatus and vaginal orifice are visible. The ducts of the paraurethral (Skene) glands and the vestibular (Bartholin) glands are present but not visible.

1. The nurse is percussing the seventh right intercostal space at the midclavicular line over the liver. Which sound should the nurse expect to hear? A) Dullness B) Tympany C) Resonance D) Hyperresonance

ANS: A The liver is located in the right upper quadrant and would elicit a dull percussion note

During an annual physical exam, a 43-year-old patient states that she doesn't perform monthly breast self-examinations (BSE). She tells the nurse that she believes that mammograms "do a much better job than I ever could to find a lump." The nurse should explain to her that: A) BSEs may detect lumps that appear between mammograms. B) breast self-examination is unnecessary until the age of 50 years. C) she is correct, mammography is a good replacement for breast self-examination. D) she doesn't need to perform breast self-examination as long as a physician checks her breasts yearly.

ANS: A The monthly practice of breast self-examination, along with clinical breast examination and mammograms are complementary screening measures. Mammography can reveal cancers too small to be detected by the woman or by the most experienced examiner. However, interval lumps may become palpable between mammograms.

The nurse is assisting with a self-breast examination clinic. Which of these women reflect abnormal findings during the inspection phase of breast examination? A) Woman whose nipples are in different planes (deviated) B) Woman whose left breast is slightly larger than her right C) Nonpregnant woman whose skin is marked with linear striae D) Pregnant woman whose breasts have a fine blue network of veins visible under the skin

ANS: A The nipples should be symmetrically placed on the same plane on the two breasts. With deviation in pointing, an underlying cancer causes fibrosis in the mammary ducts, which pulls the nipple angle toward it. The other examples are normal findings. See Table 17-3.

The nurse is palpating a female patient's breasts during an examination. Which of these positions is most likely to make significant lumps more distinct during breast palpation? A) Supine with arms raised over her head B) Sitting with arms relaxed at the sides C) Supine with arms relaxed at the sides D) Sitting with arms flexed and fingertips touching shoulders

ANS: A The nurse should help the woman to a supine position, tuck a small pad under the side to be palpated, and help the woman raise her arm over her head. These maneuvers will flatten the breast tissue and displace it medially. Any significant lumps will then feel more distinct

The nurse is preparing to palpate the rectum and should use which of these techniques? A) Flex the finger and insert slowly toward the umbilicus. B) Instruct the patient first that this will be a painful procedure. C) Insert an extended index finger at a right angle to the anus. D) Place the finger directly into the anus to overcome the tight sphincter.

ANS: A The nurse should place the pad of the index finger gently against the anal verge. The nurse will feel the sphincter tighten and then relax. As it relaxes, the nurse should flex the tip of the finger and slowly insert it into the anal canal in a direction toward the umbilicus. The nurse should never approach the anus at right angles with the index finger extended—this would cause pain. The nurse should instruct the patient that palpation is not painful but may feel like needing to move the bowels.

A patient is newly diagnosed with benign breast disease. The nurse recognizes that which statement about benign breast disease is true? The presence of benign breast disease: A) makes it harder to examine the breasts. B) frequently turns into cancer in a woman's later years. C) is easily reduced with hormone replacement therapy. D) is usually diagnosed before a woman reaches childbearing age.

ANS: A The presence of benign breast disease (formerly fibrocystic breast disease) makes it harder to examine the breasts; the general lumpiness of the breast conceals a new lump. The other statements are not true.

30. Which of these percussion findings would the nurse expect to find in a patient with a large amount of ascites? A) Dullness across the abdomen B) Flatness in the right upper quadrant C) Hyperresonance in the left upper quadrant D) Tympany in the right and left lower quadrants

ANS: A The presence of fluid causes a dull sound to percussion. A large amount of ascitic fluid would produce a dull sound to percussion

33. When palpating the abdomen of a 20-year-old patient, the nurse notices the presence of tenderness in the left upper quadrant with deep palpation. Which of these structures is most likely to be involved? A) Spleen B) Sigmoid colon C) Appendix D) Gallbladder

ANS: A The spleen is located in the left upper quadrant of the abdomen. The gallbladder is in the right upper quadrant, the sigmoid colon is in the left lower quadrant, and the appendix is in the right lower quadrant

A new mother calls the clinic to report that part of her left breast is red, swollen, tender, very hot, and hard. She has a fever of 101° F. She has also had symptoms of the flu, such as chills, sweating, and feeling tired. The nurse notices that she has been breastfeeding for 1 month. From her description, what condition does the nurse suspect? A) Mastitis B) Paget's disease C) Plugged milk duct D) Mammary duct ectasia

ANS: A The symptoms describe mastitis, which stems from infection or stasis caused by a plugged duct. A plugged duct does not have infection present. (See Table 17-7.) Refer to Table 17-6 for descriptions of Paget's disease and mammary duct ectasia.

36. During report, the student nurse hears that a patient has "hepatomegaly" and recognizes that this term refers to: A) an enlarged liver. B) an enlarged spleen. C) distended bowel. D) excessive diarrhea.

ANS: A The term hepatomegaly refers to an enlarged liver. The term splenomegaly refers to an enlarged spleen. The other responses are not correct.

The nurse is reviewing the changes that occur with menopause. Which of these are changes associated with menopause? A) Uterine and ovarian atrophy along with thinning vaginal epithelium B) Ovarian atrophy, increased vaginal secretions, and increasing clitoral size C) Cervical hypertrophy, ovarian atrophy, and increased acidity of vaginal secretions D) Vaginal mucosa fragility, increased acidity of vaginal secretions, and uterine hypertrophy

ANS: A The uterus shrinks because of its decreased myometrium. The ovaries atrophy to 1 to 2 cm and are not palpable after menopause. The sacral ligaments relax, and the pelvic musculature weakens, so the uterus droops. The cervix shrinks and looks paler with a thick glistening epithelium. The vaginal epithelium atrophies, becoming thinner, drier, and itchy. The vaginal pH becomes more alkaline, and secretions are decreased. This results in a fragile mucosal surface that is at risk for bleeding and vaginitis.

During a vaginal examination of a 38-year-old woman, the nurse notices that the vulva and vagina are erythematous and edematous with thick, white, curdlike discharge adhering to the vaginal walls. The woman reports intense pruritus and thick white discharge from her vagina. The nurse knows that these history and physical examination findings are most consistent with which of these conditions? A) Candidiasis B) Trichomoniasis C) Atrophic vaginitis D) Bacterial vaginosis

ANS: A The woman with candidiasis often reports intense pruritus and thick white discharge. The vulva and vagina are erythematous and edematous. The discharge is usually thick, white, and curdlike. Infection with trichomoniasis causes a profuse, watery, gray-green, and frothy discharge. Bacterial vaginosis causes a profuse discharge that has a "foul, fishy, rotten" odor. Atrophic vaginitis may have a mucoid discharge. See Table 26-5 for complete descriptions of each option.

The nurse is assessing a patient who is admitted with possible delirium. Which of these are manifestations of delirium? Select all that apply. a. Develops over a short period. b. Person is experiencing apraxia. c. Person is exhibiting memory impairment or deficits. d. Occurs as a result of a medical condition, such as systemic infection. e. Person is experiencing agnosia.

ANS: A (Develops over a short period), C (Person is exhibiting memory impairment or deficits), D (Occurs as a result of a medical condition such as systematic infection) Delirium is a disturbance of consciousness that develops over a short period and may be attributable to a medical condition. Memory deficits may also occur. Apraxia and agnosia occur with dementia.

11. The nurse is assessing the intravenous (IV) infusion at the beginning of the shift. Which of these should be included in the assessment of the infusion? Select all that apply. A. Proper IV solution is infusing according to physician's orders. B. IV solution is infusing at the proper rate according to physician's orders. C. The infusion is proper according to the nurse's assessment of the patient's needs. D. Capillary refill in the fingers E. IV site date F. Whether the patient is voiding sufficiently

ANS: A, B, C, E The nurse should verify that the proper IV solution is hanging and is flowing at the proper rate according to the physician's orders and the nurse's own assessment of the patient's needs. In addition, the nurse should note the date of the IV site and surrounding skin condition. Checking capillary refill is part of the cardiovascular assessment; checking the patient's voiding is part of the genitourinary assessment.

The nurse is assessing the breasts of a 68-year-old woman and discovers a mass in the upper outer quadrant of the left breast. When assessing this mass, the nurse keeps in mind that characteristics of a cancerous mass include which of the following? Select all that apply. A) Nontender mass B) Dull, heavy pain on palpation C) Rubbery texture and mobile D) Hard, dense, and immobile E) Regular border F) Irregular, poorly delineated border

ANS: A, D, F Cancerous breast masses are solitary, unilateral, nontender, masses. They are solid, hard, dense, and fixed to underlying tissues or skin as cancer becomes invasive. Their borders are irregular and poorly delineated. They are often painless, although the person may have pain. They are most common in upper outer quadrant. A dull, heavy pain on palpation and a mass with a rubbery texture and a regular border are characteristics of benign breast disease.

The electrical stimulus of the cardiac cycle follows which sequence? a. AV node SA node bundle of His b. Bundle of His AV node SA node c. SA node AV node bundle of His bundle branches d. AV node SA node bundle of His bundle branches

ANS: AV node SA node bundle of His bundle branches Specialized cells in the SA node near the superior vena cava initiate an electrical impulse. The current flows in an orderly sequence, first across the atria to the AV node low in the atrial septum. There it is delayed slightly so that the atria have time to contract before the ventricles are stimulated. Then the impulse travels to the bundle of His, the right and left bundle branches, and then through the ventricles.

A 30-year-old female patient is describing feelings of hopelessness and depression. She has attempted self-mutilation and has a history of suicide attempts. She describes difficulty sleeping at night and has lost 10 pounds in the past month. Which of these statements or questions is the nurses best response in this situation? a. Do you have a weapon? b. How do other people treat you? c. Are you feeling so hopeless that you feel like hurting yourself now? d. People often feel hopeless, but the feelings resolve within a few weeks.

ANS: Are you feeling so hopeless that you feel like hurting yourself now? When the person expresses feelings of hopelessness, despair, or grief, assessing the risk of physical harm to him or herself is important. This process begins with more general questions. If the answers are affirmative, then the assessment continues with more specific questions.

A 26-year-old woman was robbed and beaten a month ago. She is returning to the clinic today for a follow-up assessment. The nurse will want to ask her which one of these questions? a. How are things going with the trial? b. How are things going with your job? c. Tell me about your recent engagement! d. Are you having any disturbing dreams?

ANS: Are you having any disturbing dreams? In posttraumatic stress disorder, the person has been exposed to a traumatic event. The traumatic event is persistently re-experienced by recurrent and intrusive, distressing recollections of the event, including images, thoughts, or perceptions; recurrent distressing dreams of the event; and acting or feeling as if the traumatic event were recurring.

A 2-month-old uncircumcised infant has been brought to the clinic for a well-baby checkup. How would the nurse proceed with the genital examination?

ANS: Avoid retracting the foreskin until the infant is 3 months old. If uncircumcised, then the foreskin is normally tight during the first 3 months and should not be retracted because of the risk of tearing the membrane attaching the foreskin to the shaft. The other options are not correct.

6. When reviewing the use of alcohol by older adults, the nurse notes that older adults have several characteristics that can increase the risk of alcohol use. Which would increase the bioavailability of alcohol in the blood for longer periods in the older adult? a. Increased muscle mass b. Decreased liver and kidney functioning c. Decreased blood pressure d. Increased cardiac output

ANS: B Decreased liver and kidney functioning increases the bioavailability of alcohol in the blood for longer periods. Aging people experience decreased muscle mass (not increased), which also increases the alcohol concentration in the blood because the alcohol is distributed to less tissue over time. Blood pressure and cardiac output are not factors regarding bioavailability.

9. The nurse has completed an assessment on a patient who came to the clinic for a leg injury. As a result of the assessment, the nurse has determined that the patient has at-risk alcohol use. Which action by the nurse is most appropriate at this time? a. Record the results of the assessment, and notify the physician on call. b. State, You are drinking more than is medically safe. I strongly recommend that you quit drinking, and Im willing to help you. c. State, It appears that you may have a drinking problem. Here is the telephone number of our local Alcoholics Anonymous chapter. d. Give the patient information about a local rehabilitation clinic.

ANS: B If an assessment has determined that the patient has at-risk drinking behavior, then the nurse should give a short but clear statement of assistance and concern. Simply giving out a telephone number or referral to agencies may not be enough.

11. The nurse is reviewing aspects of substance abuse in preparation for a seminar. Which of these statements illustrates the concept of tolerance to an illicit substance? The person: a. Has a physiologic dependence on a substance. b. Requires an increased amount of the substance to produce the same effect. c. Requires daily use of the substance to function and is unable to stop using it. d. Experiences a syndrome of physiologic symptoms if the substance is not used.

ANS: B The concept of tolerance to a substance indicates that the person requires an increased amount of the substance to produce the same effect. Abuse indicates that the person needs to use the substance daily to function, and the person is unable to stop using it. Dependence is an actual physiologic dependence on the substance. Withdrawal occurs when cessation of the substance produces a syndrome of physiologic symptoms.

10. A patient is brought to the emergency department. He is restless, has dilated pupils, is sweating, has a runny nose and tearing eyes, and complains of muscle and joint pains. His girlfriend thinks he has influenza, but she became concerned when his temperature went up to 39.4 C. She admits that he has been a heavy drug user, but he has been trying to stop on his own. The nurse suspects that the patient is experiencing withdrawal symptoms from which substance? a. Alcohol b. Heroin c. Crack cocaine d. Sedatives

ANS: B Withdrawal symptoms of opiates, such as heroin, are similar to the clinical picture of influenza and include symptoms such as dilated pupils, lacrimation, runny nose, tachycardia, fever, restlessness, muscle and joint pains, and other symptoms. (Withdrawal symptoms from alcohol, cocaine, and sedatives are described in Table 6-7.

A woman who is 22 weeks pregnant has a vaginal infection. She tells the nurse that she is afraid that this infection will hurt the fetus. The nurse knows that which of these statements is true? A) If intercourse is avoided, then the risk for infection is minimal. B) A thick mucus plug forms that protects the fetus from infection. C) The acidic pH of vaginal secretions promotes the growth of pathogenic bacteria. D) The mucus plug that forms in the cervical canal is a good medium for bacterial growth.

ANS: B A clot of thick, tenacious mucus forms in the spaces of the cervical canal (the mucus plug), which protects the fetus from infection. Cervical and vaginal secretions increase during pregnancy and are thick, white, and more acidic. The acidic pH keeps pathogenic bacteria from multiplying in the vagina, but the increase in glycogen increases the risk of candidiasis (commonly called a yeast infection) during pregnancy.

27. During an abdominal assessment, the nurse is unable to hear bowel sounds in a patient's abdomen. Before reporting this finding as "silent bowel sounds" the nurse should listen for at least: A) 1 minute. B) 5 minutes. C) 10 minutes. D) 2 minutes in each quadrant.

ANS: B Absent bowel sounds are rare. The nurse must listen for 5 minutes before deciding bowel sounds are completely absent.

A 40-year-old black man is in the office for his annual physical. Which statement regarding the prostate-specific antigen (PSA) blood test is true, according to the American Cancer Society? The PSA: A) should be done with this visit. B) should be done at age 45 years. C) should be done at age 50 years. D) is only necessary if there is a family history of prostate cancer.

ANS: B According to the American Cancer Society (2006) the PSA blood test should be done annually for black men beginning at age 45 years, and annually for all other men over age 50 years.

11. The nurse is watching a new graduate nurse perform auscultation of a patient's abdomen. Which statement by the new graduate shows a correct understanding of the reason auscultation precedes percussion and palpation of the abdomen? A) "We need to determine areas of tenderness before using percussion and palpation." B) "It prevents distortion of bowel sounds that might occur after percussion and palpation." C) "It allows the patient more time to relax and therefore be more comfortable with the physical examination." D) "This prevents distortion of vascular sounds such as bruits and hums that might occur after percussion and palpation."

ANS: B Auscultation is performed first (after inspection) because percussion and palpation can increase peristalsis, which would give a false interpretation of bowel sounds.

12. The nurse is listening to bowel sounds. Which of these statements is true of bowel sounds? A) They are usually loud, high-pitched, rushing, tinkling sounds. B) They are usually high-pitched, gurgling, irregular sounds. C) They sound like two pieces of leather being rubbed together. D) They originate from the movement of air and fluid through the large intestine.

ANS: B Bowel sounds are high-pitched, gurgling, cascading sounds that occur irregularly from 5 to 30 times per minute. They originate from the movement of air and fluid through the small intestine.

During a breast health interview, a patient states that she has noticed pain in her left breast. The nurse's most appropriate response to this would be: A) "Don't worry about the pain; breast cancer is not painful." B) "I would like some more information about the pain in your left breast." C) "Oh, I had pain like that after my son was born; it turned out to be a blocked milk duct." D) "Breast pain is almost always the result of benign breast disease."

ANS: B Breast pain occurs with trauma, inflammation, infection, or benign breast disease. The nurse will need to gather more information about the patient's pain rather than make statements that ignore the patient's concerns.

A 54-year-old woman who has just completed menopause is in the clinic today for a yearly physical examination. Which of these statements should the nurse include in patient education? "A postmenopausal woman: A) is not at any greater risk for heart disease than a younger woman is." B) should be aware that she is at increased risk for dyspareunia because of decreased vaginal secretions." C) has only stopped menstruating; there really are no other significant changes with which she should be concerned." D) is likely to have difficulty with sexual pleasure as a result of drastic changes in the female sexual response cycle."

ANS: B Decreased vaginal secretions leave the vagina dry and at risk for irritation and pain with intercourse (dyspareunia). The other statements are incorrect.

After completing an assessment of a 60-year-old man with a family history of colon cancer, the nurse discusses with him early detection measures for colon cancer. The nurse should mention the need for a(n): A) annual proctoscopy. B) colonoscopy every 10 years. C) fecal test for blood every 6 months. D) digital rectal examinations every 2 years.

ANS: B Early detection measures for colon cancer include a digital rectal examination performed annually after age 50 years, a fecal occult blood test annually after age 50 years, sigmoidoscopy every 5 years or colonoscopy every 10 years after age 50 years; and a PSA blood test annually for men over 50 years old, except black men beginning at age 45 years (American Cancer Society, 2006).

During an interview, a patient reveals that she is pregnant. She states that she is not sure whether she will breastfeed her baby and asks for some information about this. Which of these statements by the nurse is accurate with regard to breastfeeding? A) "Breastfed babies tend to be more colicky." B) "Breastfeeding provides the perfect food and antibodies for your baby." C) "Breastfed babies eat more often than infants on formula." D) "Breastfeeding is second nature and every woman can do it."

ANS: B Exclusively breastfeeding for 6 months provides the perfect food and antibodies for the baby, decreases the risk of ear infections, promotes bonding, and provides relaxation

The nurse is preparing to examine the external genitalia of a school-age girl. Which of these positions would be most appropriate in this situation? A) In the parent's lap B) In a frog-leg position on the examining table C) In the lithotomy position with the feet in stirrups D) Lying flat on the examining table with legs extended

ANS: B For school-age children it is best to place them on the examining table in a frog-leg position. With toddlers and preschoolers, it is best to have the child on the parent's lap in a frog-leg position.

17. An older patient has been diagnosed with pernicious anemia. The nurse knows that this condition could be related to: A) increased gastric acid secretion. B) decreased gastric acid secretion. C) delayed gastrointestinal emptying time. D) increased gastrointestinal emptying time.

ANS: B Gastric acid secretion decreases with aging, and this may cause pernicious anemia (because it interferes with vitamin B12 absorption), iron deficiency anemia, and malabsorption of calcium.

A 30-year-old woman is visiting the clinic because of "pain in my bottom when I have a bowel movement." The nurse should assess for which problem? A) Pinworms B) Hemorrhoids C) Colon cancer D) Fecal incontinence

ANS: B Having painful bowel movements, known as dyschezia, may be due to a local condition (hemorrhoid or fissure) or constipation. The other responses are not correct.

5. The nurse has administered a pain medication to a patient by an intravenous infusion. The nurse should reassess the patient's response to the pain medication within _____ minutes. A. 5 B. 15 C. 30 D. 60

ANS: B If pain medication is given, then the nurse should reassess the patient's response in 15 minutes for IV administration or 1 hour for oral administration.

A patient states during the interview that she noticed a new lump in the shower a few days ago. It was on her left breast near her axilla. The nurse should plan to: A) palpate the lump first. B) palpate the unaffected breast first. C) avoid palpating the lump because it could be a cyst, which might rupture. D) palpate the breast with the lump first but plan to palpate the axilla last.

ANS: B If the woman mentions a breast lump she has discovered herself, the nurse should examine the unaffected breast first to learn a baseline of normal consistency for this individual.

9. When assessing a patient in the hospital setting, the nurse knows that which statement is true? A. The patient will need a brief assessment at least every 4 hours. B. The patient will need a consistent, specialized examination every 8 hours that focuses on certain parameters. C. The patient will need a complete head-to-toe physical examination every 24 hours. D. Most patients require a minimal examination each shift unless they are in critical condition.

ANS: B In a hospital setting, the patient does not require a complete head-to-toe physical examination during every 24-hour stay. The patient does require a consistent specialized examination every 8 hours that focuses on certain parameters

The structure that secretes a thin, milky alkaline fluid to enhance the viability of sperm is the: A) Cowper's gland. B) prostate gland. C) median sulcus. D) bulbourethral gland.

ANS: B In men, the prostate gland secretes a thin milky alkaline fluid that enhances sperm viability. The Cowper's glands (also known as bulbourethral glands) secrete a clear, viscid mucus. The median sulcus is a groove dividing the lobes of the prostate gland and does not secrete fluid.

A nurse is assessing a patient's risk of contracting a sexually transmitted infection (STI). An appropriate question to ask would be: A) "You know that it's important to use condoms for protection, right?" B) "Do you use a condom with each episode of sexual intercourse?" C) "Do you have a sexually transmitted infection?" D) "You are aware of the dangers of unprotected sex, aren't you?"

ANS: B In reviewing a patient's risk for sexually transmitted infections, the nurse should ask, in a nonconfrontational manner, whether condoms are used at each episode of sexual intercourse. Asking a person whether he or she has an infection does not address the risk.

23. The nurse is assessing the abdomen of an aging adult. Which of these statements regarding the aging adult and abdominal assessment is true? A) The abdominal tone is increased. B) The abdominal musculature is thinner. C) Abdominal rigidity with acute abdominal conditions is more common. D) The aging person complains of more pain with an acute abdominal condition than a younger person would.

ANS: B In the aging person, the abdominal musculature is thinner and has less tone than that of the younger adult, and abdominal rigidity with acute abdominal conditions is less common in aging. The aging person often complains less of pain than a younger person would with an acute abdominal condition.

A married couple has come to the clinic seeking advice on pregnancy. They have been trying to conceive for 4 months and have not been successful. What should the nurse do first? A) Ascertain whether either of them has been using broad-spectrum antibiotics. B) Explain that couples are considered infertile after 1 year of unprotected intercourse. C) Immediately refer the woman to an expert in pelvic inflammatory disease—the most common cause of infertility. D) Explain that couples are considered infertile after 3 months of engaging in unprotected intercourse and that they will need a referral to a fertility expert.

ANS: B Infertility is considered after 1 year of engaging in unprotected sexual intercourse without conceiving. The other actions are not appropriate

While inspecting a patient's breasts, the nurse finds that the left breast is slightly larger than the right with the presence of Montgomery's glands bilaterally. The nurse should: A) palpate over the Montgomery's glands, checking for drainage. B) consider these normal findings and proceed with the examination. C) ask extensive history questions regarding the woman's breast asymmetry. D) continue with examination and then refer the patient for further evaluation of the Montgomery's glands.

ANS: B Normal findings of the breast include one breast (most often the left) slightly larger than the other and the presence of Montgomery's glands across the areola.

The nurse is palpating a female patient's adnexa. The findings include a firm, smooth uterine wall; the ovaries are palpable and feel smooth and firm. The fallopian tube is firm and pulsating. The nurse's most appropriate course of action would be to: A) tell the patient that her examination was normal. B) give her an immediate referral to a gynecologist. C) suggest that she return in a month for a recheck to verify the findings. D) tell the patient that she may have an ovarian cyst that should be evaluated further.

ANS: B Normally the uterine wall feels firm and smooth, with the contour of the fundus rounded. Ovaries are not often palpable, but when they are, they normally feel smooth, firm, and almond shaped and are highly movable, sliding through the fingers. The fallopian tube is not palpable normally. No other mass or pulsation should be felt. Pulsation or palpable fallopian tube suggests ectopic pregnancy, which warrants immediate referral

28. A patient is suspected of having inflammation of the gallbladder, or cholecystitis. The nurse should conduct which of these techniques to assess for this condition? A) Obturator test B) Test for Murphy's sign C) Assess for rebound tenderness D) Iliopsoas muscle test

ANS: B Normally, palpating the liver causes no pain. In a person with inflammation of the gallbladder, orcholecystitis, pain occurs as the descending liver pushes the inflamed gallbladder onto the examining hand during inspiration (Murphy's test). The person feels sharp pain and abruptly stops inspiration midway.

15. The nurse is assessing the abdomen of a pregnant woman who is complaining of having "acid indigestion" all the time. The nurse knows that esophageal reflux during pregnancy can cause: A) diarrhea. B) pyrosis. C) dysphagia. D) constipation.

ANS: B Pyrosis, or heartburn (not constipation), is caused by esophageal reflux during pregnancy. The other options are not correct

During an examination of a 7-year-old girl, the nurse notices that the girl is showing breast budding. What should the nurse do next? A) Ask her if her periods have started. B) Assess the girl's weight and body mass index (BMI). C) Ask the girl's mother at what age she started to develop breasts. D) Nothing; this is a normal finding.

ANS: B Research has shown that girls with overweight or obese BMI levels have a higher occurrence of early onset of breast budding (before age 8 years for African-American girls and age 10 years for white girls) and early menarche.

A 50-year-old woman calls the clinic because she has noticed some changes in her body and breasts and wonders if they could be due to the hormone replacement therapy (HRT) she started 3 months ago. The nurse should tell her: A) "Hormone replacement therapy is at such a low dose that side effects are very unusual." B) "Hormone replacement therapy has several side effects, including fluid retention, breast tenderness, and vaginal bleeding." C) "It would be very unusual to have vaginal bleeding with hormone replacement therapy, and I suggest you come in to the clinic immediately to have this evaluated." D) "It sounds as if your dose of estrogen is too high; I think you may need to decrease the amount you are taking and then call back in a week."

ANS: B Side effects of hormone replacement therapy include fluid retention, breast pain, and vaginal bleeding. The other responses are not correct.

When the nurse is discussing sexuality and sexual issues with adolescents, a permission statement helps to convey that it is normal to think or feel a certain way. Which of these is the best example of a permission statement? A) "It is okay that you have become sexually active." B) "Often girls your age have questions about sexual activity. Have you any questions?" C) "If it is okay with you, I'd like to ask you some questions about your sexual history." D) "Often girls your age engage in sexual activity. It is okay to tell me if you have had intercourse."

ANS: B Start with a permission statement, "Often girls your age experience . . . ." This conveys that it is normal to think or feel a certain way, and it is important to relay that the topic is normal and unexceptional.

A 52-year-old patient states that when she sneezes or coughs she "wets herself a little." She is very concerned that something may be wrong with her. The nurse suspects that the problem is: A) dysuria. B) stress incontinence. C) hematuria. D) urge incontinence.

ANS: B Stress incontinence is involuntary urine loss with physical strain, sneezing, or coughing. Dysuria is pain or burning with urination. Hematuria is bleeding with urination. Urge incontinence is involuntary urine loss but it occurs due to an overactive detrusor muscle in the bladder that contracts and causes an urgent need to void

The nurse is performing an examination of the anus and rectum. Which of these statements is correct and important to remember during this examination? A) The rectum is about 8 cm long. B) The anorectal junction cannot be palpated. C) Above the anal canal, the rectum turns anteriorly. D) There are no sensory nerves in the anal canal or rectum.

ANS: B The anal columns are folds of mucosa that extend vertically down from the rectum and end in the anorectal junction. This junction is not palpable, but it is visible on proctoscopy. The rectum is 12 cm long, and just above the anal canal, the rectum dilates and turns posteriorly.

If a patient reports a recent breast infection, then the nurse should expect to find _____ node enlargement. A) nonspecific B) ipsilateral axillary C) contralateral axillary D) inguinal and cervical

ANS: B The breast has extensive lymphatic drainage. Most of the lymph, more than 75%, drains into the ipsilateral, or same side, axillary nodes.

Which of the following statements is true regarding the internal structures of the breast? The breast is: A) mainly muscle, with very little fibrous tissue. B) composed of fibrous, glandular, and adipose tissue. C) composed mostly of milk ducts, known as lactiferous ducts. D) composed of glandular tissue, which supports the breast by attaching to the chest wall.

ANS: B The breast is composed of glandular tissue, fibrous tissue (including the suspensory ligaments), and adipose tissue.

During an examination, the nurse would expect the cervical os of a woman who has never had children to appear: A) stellate. B) small and round. C) as a horizontal irregular slit. D) everted.

ANS: B The cervical os in a nulliparous woman is small and round. In the parous woman, it is a horizontal, irregular slit that also may show healed lacerations on the sides. See Figure 26-13.

During an internal examination, the nurse notices that the cervix bulges outside the introitus when the patient is asked to strain. The nurse will document this as: A) uterine prolapse, graded first degree. B) uterine prolapse, graded second degree. C) uterine prolapse, graded third degree. D) a normal finding.

ANS: B The cervix should not be found to bulge into the vagina. Uterine prolapse is graded as follows: first degree—cervix appears at introitus with straining; second degree—cervix bulges outside introitus with straining; and third degree—whole uterus protrudes, even without straining (essentially, uterus is inside out).

Which of these statements about the sphincters is correct? A) The internal sphincter is under voluntary control. B) The external sphincter is under voluntary control. C) Both sphincters remain slightly relaxed at all times. D) The internal sphincter surrounds the external sphincter.

ANS: B The external sphincter surrounds the internal sphincter but also has a small section overriding the tip of the internal sphincter at the opening. The external sphincter is under voluntary control. Except for the passing of feces and gas, the sphincters keep the anal canal tightly closed.

The nurse is palpating the prostate gland through the rectum and notices an abnormal finding if which of these is present? A) Palpable central groove B) Tenderness to palpation C) Heart shape D) Elastic and rubbery consistency

ANS: B The normal prostate gland should feel smooth, elastic, and rubbery; should be slightly movable; should be heart-shaped with a palpable central groove; and should not be tender to palpation.

During the examination portion of a patient's visit, she will be in lithotomy position. Which statement below reflects some things that the nurse can do to make this more comfortable for her? A) Ask her to place her hands and arms behind her head. B) Elevate her head and shoulders to maintain eye contact. C) Allow her to choose to have her feet in the stirrups or have them resting side by side on the edge of the table. D) Allow her to keep her buttocks about 6 inches from the edge of the table to prevent her from feeling as if she will fall off.

ANS: B The nurse should elevate her head and shoulders to maintain eye contact. The patient's arms should be placed at her sides or across the chest, not behind the head, because this position only tightens the abdominal muscles. The feet should be placed into the stirrups, knees apart, and buttocks at the edge of the examining table. Place the stirrups so the legs are not abducted too far

31. A 40-year-old man states that his physician told him that he has a hernia. He asks the nurse to explain what a hernia is. Which response by the nurse is appropriate? A) "No need to worry. Most men your age develop hernias." B) "A hernia is a loop of bowel protruding through a weak spot in the abdominal muscles." C) "This hernia is a result of prenatal growth abnormalities that are just now causing problems." D) "I'll have to have your physician explain this to you."

ANS: B The nurse should explain that a hernia is a protrusion of the abdominal viscera through an abnormal opening in the muscle wall

39. The nurse is preparing to examine a patient who has been complaining of right lower quadrant pain. Which technique is correct during the assessment? The nurse should: A) examine the tender area first. B) examine the tender area last. C) avoid palpating the tender area. D) palpate the tender area first and then auscultate for bowel sounds.

ANS: B The nurse should save the examination of any identified tender areas until last. This method avoids pain and the resulting muscle rigidity that would obscure deep palpation later in the examination. Auscultation is done before percussion and palpation because percussion and palpation can increase peristalsis, which would give a false interpretation of bowel sounds

During a breast examination on a female patient, the nurse notices that the nipple is flat, broad, and fixed. The patient states it "started doing that a few months ago." This finding suggests: A) dimpling. B) a retracted nipple. C) nipple inversion. D) deviation in nipple pointing.

ANS: B The retracted nipple looks flatter and broader, like an underlying crater. A recent retraction suggests cancer, which causes fibrosis of the whole duct system and pulls in the nipple. It also may occur with benign lesions such as ectasia of the ducts. The nurse should not confuse retraction with the normal long-standing type of nipple inversion, which has no broadening and is not fixed.

In performing a breast examination, the nurse knows that it is especially important to examine the upper outer quadrant of the breast. The reason for this is that the upper outer quadrant is: A) the largest quadrant of the breast. B) the location of most breast tumors. C) where most of the suspensory ligaments attach. D) more prone to injury and calcifications than other locations in the breast.

ANS: B The upper outer quadrant is the site of most breast tumors. In the upper outer quadrant, the nurse should notice the axillary tail of Spence, the cone-shaped breast tissue that projects up into the axilla, close to the pectoral group of axillary lymph nodes.

The nurse has just completed an inspection of a nulliparous woman's external genitalia. Which of these would be a description of a finding within normal limits? A) Redness of the labia majora B) Multiple nontender sebaceous cysts C) Discharge that is sticky and yellow-green D) Gaping and slightly shriveled labia majora

ANS: B There should be no lesions, except for occasional sebaceous cysts. These are yellowish 1-cm nodules that are firm, nontender, and often multiple. The labia majora are dark pink, moist, and symmetrical; redness indicates inflammation or lesions. Discharge that is sticky and yellow-green may indicate infection. In the nulliparous woman, the labia majora meet in the midline, are symmetric and plump.

14. During an abdominal assessment, the nurse would consider which of these findings as normal? A) The presence of a bruit in the femoral area B) A tympanic percussion note in the umbilical region C) A palpable spleen between the ninth and eleventh ribs in the left midaxillary line D) A dull percussion note in the left upper quadrant at the midclavicular line

ANS: B Tympany should predominate in all four quadrants of the abdomen because air in the intestines rises to the surface when the person is supine. Vascular bruits are not usually present. Normally the spleen is not palpable. Dullness would not be found in the area of lung resonance (left upper quadrant at the midclavicular line)

A patient calls the clinic for instructions before having a Papanicolaou (Pap) smear. The most appropriate instructions from the nurse are: A) "If you are menstruating, please use pads to avoid placing anything into the vagina." B) "Avoid intercourse, inserting anything into the vagina, or douching within 24 hours of your appointment." C) "If you suspect that you have a vaginal infection, please gather a sample of the discharge to bring with you." D) "We would like you to use a mild saline douche before your examination. You may pick this up in our office."

ANS: B When instructing a patient before a Papanicolaou (Pap) smear is obtained, the nurse should follow these guidelines: Do not obtain during the woman's menses or if a heavy infectious discharge is present. Instruct the woman not to douche, have intercourse, or put anything into the vagina within 24 hours before collecting the specimens. Any specimens will be obtained during the visit, not beforehand.

20. The nurse knows that during an abdominal assessment, deep palpation is used to determine: A) bowel motility. B) enlarged organs. C) superficial tenderness. D) overall impression of skin surface and superficial musculature.

ANS: B With deep palpation, the nurse should notice the location, size, consistency, and mobility of any palpable organs and the presence of any abnormal enlargement, tenderness, or masses.

1. A patient with a known history of heavy alcohol use has been admitted to the ICU after he was found unconscious outside a bar. The nurse closely monitors him for symptoms of withdrawal. Which of these symptoms may occur during this time? Select all that apply. a. Bradycardia b. Coarse tremor of the hands c. Transient hallucinations d. Somnolence e. Sweating

ANS: B, C, E Symptoms of uncomplicated alcohol withdrawal start shortly after the cessation of drinking, peak at the second day, and improve by the fourth or fifth day. Symptoms include coarse tremors of the hands, tongue, and eyelids; anorexia; nausea and vomiting; autonomic hyperactivity (e.g., tachycardia, sweating, elevated blood pressure); and transient hallucinations, among other symptoms (see Table 6-7).

The nurse is presenting a class on risk factors for cardiovascular disease. Which of these are considered modifiable risk factors for myocardial infarction (MI)? Select all that apply. a. Ethnicity b. Abnormal lipids c. Smoking d. Gender e. Hypertension f. Diabetes g. Family history

ANS: B. Abnormal lipids C, Smoking E. Hypertension F. Diabetes Nine modifiable risk factors for MI, as identified by a recent study, include abnormal lipids, smoking, hypertension, diabetes, abdominal obesity, psychosocial factors, consumption of fruits and vegetables, alcohol use, and regular physical activity.

A 52-year-old woman has a papule on her nose that has rounded, pearly borders and a central red ulcer. She said she first noticed it several months ago and that it has slowly grown larger. The nurse suspects which condition? a. Acne b. Basal cell carcinoma c. Melanoma d. Squamous cell carcinoma

ANS: Basal cell carcinoma Basal cell carcinoma usually starts as a skin-colored papule that develops rounded, pearly borders with a central red ulcer. It is the most common form of skin cancer, and it grows slowly. This description does not fit acne lesions. See Table 12-11 for descriptions of malignant melanoma and squamous cell carcinoma.

The nurse is preparing for a class on risk factors for hypertension, and reviews recent statistics. Which racial group has the highest prevalence of hypertension in the world? a. Blacks b. Whites c. American Indians d. Hispanics

ANS: Blacks According to the American Heart Association, the prevalence of hypertension is higher among African-Americans than in other racial groups.

A 55-year-old man is in the clinic for a yearly check-up. He is worried because his father died of prostate cancer. The nurse knows that which tests should be done at this time? Select all that apply.

ANS: Blood test for prostate-specific antigen Digital rectal examination Prostate cancer is typically detected by testing the blood for prostate-specific antigen (PSA) or by a digital rectal exam (DRE). It is recommended that both PSA and DRE be offered to men yearly, beginning at age 50 years. If the PSA is elevated, then further lab work or a transrectal ultrasound (TRUS) and biopsy may be recommended.

The nurse is examining a 3-year-old child who was brought to the emergency room after a fall. Which bruise, if found, would be of most concern?

ANS: Bruising on the abdomen Studies have shown that children who were walking often had bruises over the bony prominences of the front of their bodies. Other studies have found that bruising in "atypical" places such as the buttocks, hands, feet and abdomen was exceedingly rare and should arouse concern.

1. At the beginning of rounds, when the nurse enters the room, what should the nurse do first? A. Check the intravenous infusion site for swelling or redness. B. Check the infusion pump settings for accuracy. C. Make eye contact with the patient and introduce himself or herself as the patient's nurse. D. Offer the patient something to drink

ANS: C

4. During a session on substance abuse, the nurse is reviewing statistics with the class. For persons aged 12 years and older, which illicit substance was most commonly used? a. Crack cocaine b. Heroin c. Marijuana d. Hallucinogens

ANS: C In persons age 12 years and older who reported using during the past month, marijuana (hashish) was the most commonly used illicit drug reported.

The nurse is preparing for a class in early detection of breast cancer. Which statement is true with regard to breast cancer in African-American women in the United States? A) Breast cancer is not a threat to African-American women. B) African-American women have a lower incidence of regional or distant breast cancer than white women. C) African-American women are more likely to die of breast cancer at any age. D) Breast cancer incidence in African-American women is higher than that of white women after age 45.

ANS: C African-American women have a higher incidence of breast cancer before age 45 years than white women, and are more likely to die of their disease. In addition, African-American women are significantly more likely to be diagnosed with regional or distant breast cancer than are white women. This racial difference in mortality rates may be related to insufficient use of screening measures and lack of access to health care.

During the interview a patient reveals that she has some vaginal discharge. She is worried that it may be a sexually transmitted infection. The nurse's most appropriate response to this would be: A) "Oh, don't worry. Some cyclic vaginal discharge is normal." B) "Have you been engaging in unprotected sexual intercourse?" C) "I'd like some information about the discharge. What color is it?" D) "Have you had any urinary incontinence associated with the discharge?"

ANS: C Ask questions that help the patient reveal more information about her symptoms in a nonthreatening manner. Assess vaginal discharge further by asking about the amount, color, and odor. Normal vaginal discharge is small, clear or cloudy, and always nonirritating.

8. When assessing a patient's general appearance, the nurse should include which of these questions? A. Is the patient's muscle strength equal in both arms? B. Is ptosis or facial droop present? C. Does the patient respond appropriately to questions? D. Are the pupils equal in reaction and size?

ANS: C Assessing whether the patient responds appropriately to questions is a component of assessment of the patient's general appearance. The other answers reflect components of the neurologic examination.

7. When assessing the neurologic system of a hospitalized patient during morning rounds, the nurse should include which of these during the assessment? A. Blood pressure B. The patient's rating of pain on a 1 to 10 scale C. The patient's ability to communicate D. The patient's personal hygiene level

ANS: C Assessment of a patient's ability to communicate is part of the neurologic assessment. Blood pressure and pain rating are measurements, and personal hygiene is assessed under general appearance.

The changes normally associated with menopause occur generally because the cells in the reproductive tract are: A) aging. B) becoming fibrous. C) estrogen dependent. D) able to respond to estrogen.

ANS: C Because cells in the reproductive tract are estrogen dependent, decreased estrogen levels during menopause bring dramatic physical changes. The other options are not correct.

21. The nurse notices that a patient has had a black, tarry stool and recalls that a possible cause would be: A) gallbladder disease. B) overuse of laxatives. C) gastrointestinal bleeding. D) localized bleeding around the anus.

ANS: C Black stools may be tarry as a result of occult blood (melena) from gastrointestinal bleeding. Redblood in stools occurs with localized bleeding around the anus

A patient who is visiting the clinic complains of having "stomach pains for 2 weeks" and describes his stools as being "soft and black" for about the last 10 days. He denies taking any medications. The nurse is aware that these symptoms are most indicative of: A) excessive fat caused by malabsorption. B) increased iron intake resulting from a change in diet. C) occult blood resulting from gastrointestinal bleeding. D) absent bile pigment from liver problems.

ANS: C Black stools may be tarry due to occult blood (melena) from gastrointestinal bleeding or nontarry from ingestion of iron medications (not diet). Excessive fat causes the stool to become frothy; absence of bile pigment causes clay-colored stools.

A woman is in the family planning clinic seeking birth control information. She states that her breasts "change all month long" and that she is worried that this is unusual. What is the nurse's best response? A) Tell her that it is unusual. The breasts of nonpregnant females usually stay pretty much the same all month long. B) Tell her that it is very common for breasts to change in response to stress and that she should assess her life for stressful events. C) Tell her that, because of the changing hormones during the monthly menstrual cycle, cyclic breast changes are common. D) Tell her that breast changes normally occur only during pregnancy and that a pregnancy test is needed at this time.

ANS: C Breasts of the nonpregnant woman change with the ebb and flow of hormones during the monthly menstrual cycle. During the 3 to 4 days before menstruation, the breasts feel full, tight, heavy, and occasionally sore. The breast volume is smallest on days 4 to 7 of the menstrual cycle.

3. A patient is having difficulty in swallowing medications and food. The nurse would document that this patient has: A) aphasia. B) dysphasia. C) dysphagia. D) anorexia.

ANS: C Dysphagia is a condition that occurs with disorders of the throat or esophagus and results in difficulty swallowing. Aphasia and dysphasia are speech disorders. Anorexia is a loss of appetite

A patient has had three pregnancies and two live births. The nurse would record this information as gravida _____, para _____, AB _____. A) 2; 2; 1 B) 3; 2; 0 C) 3; 2; 1 D) 3; 3; 1

ANS: C Gravida is number of pregnancies. Para is number of births. Abortions are interrupted pregnancies, including elective abortions and spontaneous miscarriages.

In examining a 70-year-old male patient, the nurse notices that he has bilateral gynecomastia. Which of the following describes the nurse's best course of action? A) Recommend that he make an appointment with his physician for a mammogram. B) Ignore it; it is not unusual for men to have benign breast enlargement. C) Explain that this condition may be the result of hormonal changes and recommend that he see his physician. D) Tell him that gynecomastia in men is usually associated with prostate enlargement and recommend that he be screened thoroughly.

ANS: C Gynecomastia may reappear in the aging male and may be due to testosterone deficiency.

During an external genitalia examination of a woman, the nurse notices several lesions around the vulva. The lesions are pink, moist, soft, and pointed papules. The patient states that she is not aware of any problems in that area. The nurse recognizes that these lesions may be: A) syphilitic chancre. B) herpes simplex virus type 2 (herpes genitalis). C) human papillomavirus (HPV), or genital warts. D) pediculosis pubis (crab lice).

ANS: C HPV lesions are painless, warty growths that the woman may not notice. Lesions are pink or flesh colored, soft, pointed, moist, warty papules that occur in single or multiple cauliflower-like patches around the vulva, introitus, anus, vagina, or cervix. Herpetic lesions are painful clusters of small, shallow vesicles with surrounding erythema. Syphilitic chancres begin as a solitary silvery papule that erodes to a red, round or oval, superficial ulcer with a yellowish discharge. Pediculosis pubis causes severe perineal itching and excoriations and erythematous areas. See Table 26-2.

During the interview with a female patient, the nurse gathers data that indicate that the patient is perimenopausal. Which of these statements made by this patient leads to this conclusion? A) "I have noticed that my muscles ache at night when I go to bed." B) "I will be very happy when I can stop worrying about having a period." C) "I have been noticing that I sweat a lot more than I used to, especially at night." D) "I have only been pregnant twice, but both times I had breast tenderness as my first symptom."

ANS: C Hormone shifts occur during the perimenopausal period, and associated symptoms of menopause may occur, such as hot flashes, night sweats, numbness and tingling, headache, palpitations, drenching sweats, mood swings, vaginal dryness, and itching. The other responses are not correct.

The nurse has palpated a lump in a female patient's right breast. The nurse documents this as a small, round, firm, distinct, lump located at 2 o'clock, 2 cm from the nipple. It is nontender and fixed. There is no associated retraction of skin or nipple, no erythema, and no axillary lymphadenopathy. Which of these statements reveals the information that is missing from the documentation? It is missing information about: A) the shape of the lump. B) the lump's consistency. C) the size of the lump. D) whether the lump is solitary or multiple.

ANS: C If the nurse feels a lump or mass, he or she should note these characteristics: (1) location, (2) size—judge in centimeters in three dimensions: width × length × thickness, (3) shape, (4) consistency, (5) motility, (6) distinctness, (7) nipple, (8) the skin over the lump, (9) tenderness, and (10) lymphadenopathy.

3. During a morning assessment, the nurse notices that a patient's urine output is below the expected amount. What should the nurse do next? A. Obtain an order for a Foley catheter. B. Obtain an order for a straight catheter. C. Perform a bladder scan test. D. Refer the patient to a urologist.

ANS: C If urine output is below the expected value, then the nurse should perform a bladder scan according to institutional policy to check for retention.

During a physical examination, a 45-year-old woman states that she has had a crusty, itchy rash on her breast for about 2 weeks. In trying to find the cause of the rash, which of these would be important for the nurse to determine? A) Is the rash raised and red? B) Does it appear to be cyclic? C) Where did it first appear—on the nipple, the areola, or the surrounding skin? D) What was she doing when she first noticed the rash, and do her actions make it worse?

ANS: C It is important for the nurse to determine where the rash first appeared. Paget's disease starts with a small crust on the nipple apex and then spreads to the areola. Eczema or other dermatitis rarely starts at nipple unless it results from breastfeeding. It usually starts on the areola or surrounding skin and then spreads to the nipple. See Table 17-6.

26. The nurse is reviewing the assessment of an aortic aneurysm. Which of these statements is true regarding an aortic aneurysm? A) A bruit is absent. B) Femoral pulses are increased. C) A pulsating mass is usually present. D) Most are located below the umbilicus.

ANS: C Most aortic aneurysms are palpable during routine examination and feel like a pulsating mass. A bruit will be audible, and femoral pulses are present but decreased. Such aneurysms are located in the upper abdomen just to the left of midline

The nurse is examining a 35-year-old female patient. During the history, the nurse notices that she has had two term pregnancies, and both babies were delivered vaginally. During the internal examination the nurse observes that the cervical os is a horizontal slit with some healed lacerations and that the cervix has some nabothian cysts that are small, smooth, and yellow. In addition, the nurse notices that the cervical surface is granular and red, especially around the os. Finally, the nurse notices the presence of stringy, opaque, odorless secretions. Which of these findings are abnormal? A) Nabothian cysts are present. B) The cervical os is a horizontal slit. C) The cervical surface is granular and red. D) Stringy and opaque secretions are present.

ANS: C Normal findings: Nabothian cysts may be present on the cervix after childbirth. The cervical os is a horizontal, irregular slit in the parous woman. Secretions vary according to the day of the menstrual cycle, and may be clear and thin or thick, opaque, and stringy. The surface is normally smooth, but cervical eversion, or ectropion may occur where the endocervical canal is "rolled out." Abnormal finding: The cervical surface should not be reddened or granular, which may indicate a lesion.

When performing the bimanual examination, the nurse notices that the cervix feels smooth and firm, is round, and is fixed in place (does not move). When cervical palpation is performed, the patient complains of some pain. The nurse's interpretation of these results should be which of these? A) These findings are all within normal limits. B) The cervical consistency should be soft and velvety—not firm. C) The cervix should move when palpated; an immobile cervix may indicate malignancy. D) Pain may occur during palpation of the cervix.

ANS: C Normally the cervix feels smooth and firm, as the consistency of the tip of the nose. It softens and feels velvety at 5 to 6 weeks of pregnancy (Goodell's sign). The cervix should be evenly rounded. With a finger on either side, the examiner should be able to move the cervix gently from side to side, and doing so should produce no pain for the patient. Hardness of the cervix may occur with malignancy. Immobility may occur with malignancy, and pain may occur with inflammation or ectopic pregnancy.

9. While examining a patient, the nurse observes abdominal pulsations between the xiphoid and umbilicus. The nurse would suspect that these are: A) pulsations of the renal arteries. B) pulsations of the inferior vena cava. C) normal abdominal aortic pulsations. D) increased peristalsis from a bowel obstruction.

ANS: C Normally, one may see the pulsations from the aorta beneath the skin in the epigastric area, particularly in thin persons with good muscle wall relaxation.

A patient contacts the office and tells the nurse that she is worried about her 10-year-old daughter having breast cancer. She describes a unilateral enlargement of the right breast with associated tenderness. She is worried because the left breast is not enlarged. What would be the nurse's best response? A) Tell the mother that breast development is usually fairly symmetric and she should be examined right away. B) Tell the mother that she should bring her daughter in right away because breast cancer is fairly common in preadolescent girls. C) Tell the mother that, although an examination of her daughter would rule out a problem, it is most likely normal breast development. D) Tell the mother that it is unusual for breasts that are first developing to feel tender because they haven't developed much fibrous tissue.

ANS: C Occasionally one breast may grow faster than the other, producing a temporary asymmetry. This may cause some distress; reassurance is necessary. Tenderness is common also.

A 54-year-old man comes to the clinic with a "horrible problem." He tells the nurse that he has just discovered a lump on his breast and is fearful of cancer. The nurse knows that which statement about breast cancer in males is true? A) Breast masses in men are difficult to detect because of minimal breast tissue. B) Breast cancer in men rarely spreads to the lymph nodes. C) One percent of all breast cancer occurs in men. D) Most breast masses in men are diagnosed as gynecomastia.

ANS: C One percent of all breast cancer occurs in men. Early spread to axillary lymph nodes occurs due to minimal breast tissue.

During a bimanual examination, the nurse detects a solid tumor on the ovary that is heavy and fixed, with a poorly defined mass. This finding is suggestive of: A) an ovarian cyst. B) endometriosis. C) ovarian cancer. D) an ectopic pregnancy.

ANS: C Ovarian tumors that are solid, heavy, and fixed, with poorly defined mass are suggestive of malignancy. Benign masses may feel mobile and solid. An ovarian cyst may feel smooth, round, fluctuant, mobile, and nontender. With an ectopic pregnancy, the examiner may feel a palpable, tender pelvic mass that is solid, mobile, and unilateral. Endometriosis may have masses (in various locations in the pelvic area) that are small, firm, nodular and tender to palpation, with enlarged ovaries.

40. During a health history, the patient tells the nurse, "I have pain all the time in my stomach. It's worse two hours after I eat, but it gets better if I eat again!" The nurse suspects that the patient has which condition, based on these symptoms? A) Appendicitis B) Gastric ulcer C) Duodenal ulcer D) Cholecystitis

ANS: C Pain associated with duodenal ulcers occurs 2 to 3 hours after a meal, yet it is relieved by more food. Chronic pain associated with gastric ulcers occurs usually on an empty stomach. Severe, acute pain would occur with appendicitis and cholecystitis

16. The nurse is performing percussion during an abdominal assessment. Percussion notes heard during the abdominal assessment may include: A) flatness, resonance, and dullness. B) resonance, dullness, and tympany. C) tympany, hyperresonance, and dullness. D) resonance, hyperresonance, and flatness.

ANS: C Percussion notes normally heard during the abdominal assessment may include tympany, which should predominate because air in the intestines rises to the surface when the person is supine; hyperresonance, which may be present with gaseous distention; and dullness, which may be found over a distended bladder, adipose tissue, fluid, or a mass.

A woman states that 2 weeks ago she had a urinary tract infection that was treated with an antibiotic. As a part of the interview, the nurse should ask, "Have you noticed: A) a change in your urination patterns?" B) any excessive vaginal bleeding?" C) any unusual vaginal discharge or itching?" D) any changes in your desire for intercourse?"

ANS: C Several medications may increase the risk of vaginitis. Broad-spectrum antibiotics alter the balance of normal flora, which may lead to the development of vaginitis. The other questions are not correct

18. A patient is complaining of a sharp pain along the costovertebral angles. The nurse knows that this symptom is most often indicative of: A) ovary infection. B) liver enlargement. C) kidney inflammation. D) spleen enlargement.

ANS: C Sharp pain along the costovertebral angles occurs with inflammation of the kidney or paranephric area. The other options are not correct

The nurse notices that a patient has had a pale, yellow, greasy stool, or steatorrhea, and recalls that this is caused by: A) occult bleeding. B) absent bile pigment. C) increased fat content. D) ingestion of bismuth preparations.

ANS: C Steatorrhea (pale, yellow, greasy stool) is caused by increased fat content in the stools, as in malabsorption syndrome. Occult bleeding and ingestion of bismuth products cause black stool, and absent bile pigment causes gray, tan stool.

22. During an abdominal assessment, the nurse elicits tenderness on light palpation in the right lower quadrant. The nurse interprets that this finding could indicate a disorder of which of these structures? A) Spleen B) Sigmoid C) Appendix D) Gallbladder

ANS: C The appendix is located in the right lower quadrant, and when the iliopsoas muscle is inflamed (which occurs with an inflamed or perforated appendix), pain is felt in the right lower quadrant

During a discussion about breast self-examination with a 30-year-old woman, which of these statements by the nurse is most appropriate? A) "The best time to examine your breasts is during ovulation." B) "Examine your breasts every month on the same day of the month." C) "Examine your breasts shortly after your menstrual period each month." D) "The best time to examine your breasts is immediately before menstruation."

ANS: C The best time to conduct breast self-examination is shortly after the menstrual period when the breasts are the smallest and least congested.

In performing an assessment of a woman's axillary lymph system, the nurse should assess which of these nodes? A) Central, axillary, lateral, and sternal nodes B) Pectoral, lateral, anterior, and sternal nodes C) Central, lateral, pectoral, and subscapular nodes D) Lateral, pectoral, axillary, and suprascapular nodes

ANS: C The breast has extensive lymphatic drainage. Four groups of axillary nodes are present: (1) central, (2) pectoral (anterior), (3) subscapular (posterior), and (4) lateral.

During an internal examination of a woman's genitalia, the nurse will use which technique for proper insertion of the speculum? A) Instruct the woman to bear down, open the speculum blades, and apply in a swift, upward movement. B) Insert the blades of the speculum on a horizontal plane, turning them to a 30-degree angle while continuing to insert them. Ask the woman to bear down after the speculum is inserted. C) Instruct the woman to bear down, turn the width of the blades horizontally, and insert the speculum at a 45-degree angle downward toward the small of the woman's back. D) Lock the blades open by turning the thumbscrew. Once the blades are open, apply pressure to the introitus and insert the blades at a 45-degree angle downward to bring the cervix into view.

ANS: C The examiner should instruct the woman to bear down, turn the width of the blades horizontally, and insert the speculum at a 45-degree angle downward toward the small of the woman's back. See the text under "Speculum Examination" for more detail.

A 55-year-old postmenopausal woman is being seen in the clinic for a yearly examination. She is concerned about changes in her breasts that she has noticed over the past 5 years. She states that her breasts have decreased in size and that the elasticity has changed so that her breasts seem "flat and flabby." The nurse's best reply would be: A) "This change occurs most often because of long-term use of bras that do not provide enough support to the breast tissues." B) "This is a normal change that occurs as women get older. It is due to the increased levels of progesterone during the aging process." C) "Decreases in hormones after menopause causes atrophy of the glandular tissue in the breast. This is a normal process of aging." D) "Postural changes in the spine make it appear that your breasts have changed in shape. Exercises to strengthen the muscles of the upper back and chest wall will help to prevent the changes in elasticity and size."

ANS: C The hormonal changes of menopause cause the breast glandular tissue to atrophy, making the breasts more pendulous, flattened, and sagging.

Which statement would be most appropriate when the nurse is introducing the topic of sexual relationships during an interview? A) "Now it is time to talk about your sexual history. When did you first have intercourse?" B) "Women often feel dissatisfied with their sexual relationships. Would it be okay to discuss this now?" C) "Often women have questions about their sexual relationship and how it affects their health. Do you have any questions?" D) "Most women your age have had more than one sexual partner. How many would you say you have had?"

ANS: C The nurse should begin with an open-ended question to assess individual needs. The nurse should include appropriate questions as a routine part of the history, because doing so communicates that the nurse accepts the individual's sexual activity and believes it is important. The nurse's comfort with discussion prompts the patient's interest and possibly relief that the topic has been introduced. This establishes a database for comparison with any future sexual activities and provides an opportunity to screen sexual problems.

During the physical examination, the nurse notices that a female patient has an inverted left nipple. Which statement regarding this is most accurate? A) Normal nipple inversion is usually bilateral. B) A unilateral inversion of a nipple is always a serious sign. C) It should be determined whether the inversion is a recent change. D) Nipple inversion is not significant unless accompanied by an underlying palpable mass.

ANS: C The nurse should distinguish a recently retracted nipple from one that has been inverted for many years or since puberty. Normal nipple inversion may be unilateral or bilateral and usually can be pulled out (i.e., it is not fixed). Recent nipple retraction signifies acquired disease. See Table 17-3.

The nurse is conducting a class about breast self-examination (BSE). Which of these statements indicates proper BSE technique? A) The best time to perform BSE is in the middle of the menstrual cycle. B) The woman needs to do BSE only bimonthly unless she has fibrocystic breast tissue. C) The best time to perform BSE is 4 to 7 days after the first day of the menstrual period. D) If she suspects that she is pregnant, the woman should not perform a BSE until her baby is born.

ANS: C The nurse should help each woman establish a regular schedule of self-care. The best time to conduct breast self-examination is right after the menstrual period, or the fourth through seventh day of the menstrual cycle, when the breasts are the smallest and least congested. Advise the pregnant or menopausal woman who is not having menstrual periods to select a familiar date to examine her breasts each month, for example, her birth date or the day the rent is due.

During an examination of a woman, the nurse notices that her left breast is slightly larger than her right breast. Which of these statements is true about this finding? A) Breasts should always be symmetric. B) This finding is probably due to breastfeeding and is nothing to worry about. C) This finding is not unusual, but the nurse should verify that this change is not new. D) This finding is very unusual and means she may have an inflammation or growth.

ANS: C The nurse should notice symmetry of size and shape. It is common to have a slight asymmetry in size; often the left breast is slightly larger than the right. A sudden increase in the size of one breast signifies inflammation or new growth.

The nurse is examining only the rectal area of a woman and should place the woman in what position? A) Lithotomy position B) Prone position C) Left lateral decubitus position D) Bending over the table while standing

ANS: C The nurse should place the female patient in lithotomy position if examining genitalia as well; use the left lateral decubitus position for the rectal area alone.

The nurse is preparing to teach a woman about breast self-examination (BSE). Which statement by the nurse is correct? A) "BSE is more important than ever for you because you have never had any children." B) "BSE is so important because one out of nine women will develop breast cancer in her lifetime." C) "BSE on a monthly basis will help you feel familiar with your own breasts and their normal variations." D) "BSE will save your life because you are likely to find a cancerous lump between mammograms."

ANS: C The nurse should stress that a regular monthly self-examination will familiarize her with her own breasts and their normal variations. This is a positive step that will reassure her of her healthy state. While teaching, the nurse should focus on the positive aspects of breast self-examination and should avoid citing frightening mortality statistics about breast cancer. This may generate excessive fear and denial that actually obstructs a woman's self-care action.

While assessing a hospitalized, bedridden patient, the nurse notices that the patient has been incontinent of stool. The stool is loose and gray-tan in color. The nurse recognizes that this finding indicates which of the following? A) Occult blood B) Inflammation C) Absent bile pigment D) Ingestion of iron preparations

ANS: C The presence of gray, tan stool indicates absent bile pigment, which can occur with obstructive jaundice. Ingestion of iron preparations and presence of occult blood would turn the stools to a black color. Jelly-like mucus shreds mixed in the stool would indicate inflammation.

A 46-year-old man requires assessment of his sigmoid colon. The nurse is aware that which of these is most appropriate for this examination? A) Proctoscope B) Ultrasound C) Colonoscope D) Rectal exam with an examining finger

ANS: C The sigmoid colon is 40 cm long and is accessible to examination only with the colonoscope. The other responses are not appropriate for examination of the entire sigmoid colon.

37. During an assessment the nurse notices that a patient's umbilicus is enlarged and everted. It is midline, and there is no change in skin color. The nurse recognizes that the patient may have which condition? A) Intra-abdominal bleeding B) Constipation C) Umbilical hernia D) An abdominal tumor

ANS: C The umbilicus is normally midline and inverted, with no signs of discoloration. With an umbilical hernia, the mass is enlarged and everted. The other responses are incorrect.

During a history interview, a female patient states that she has noticed a few drops of clear discharge from her right nipple. What should the nurse do next? A) Contact the physician immediately to report the discharge. B) Ask her if she is possibly pregnant. C) Ask her some additional questions about the medications she is taking. D) Immediately obtain a sample for culture and sensitivity testing.

ANS: C The use of some medications, such as oral contraceptives, phenothiazines, diuretics, digitalis, steroids, methyldopa, and calcium channel blockers, may cause clear nipple discharge. Bloody or blood-tinged discharge from the nipple, not clear, is significant, especially if a lump is also present. In the pregnant female, colostrum would be a thick, yellowish liquid, and it would be expressed after the fourth month of pregnancy.

While examining a 75-year-old woman, the nurse notices that the skin over her right breast is thickened and the hair follicles are exaggerated. This condition is known as: A) dimpling. B) retraction. C) peau d'orange. D) benign breast disease.

ANS: C This condition is known as peau d'orange. Lymphatic obstruction produces edema, which thickens the skin and exaggerates the hair follicles. The skin has a pig-skin or orange-peel look, and this condition suggests cancer.

During an assessment of the newborn, the nurse expects to see which finding when the anal area is slightly stroked? A) A jerking of the legs B) Flexion of the knees C) A quick contraction of the sphincter D) Relaxation of the external sphincter

ANS: C To assess sphincter tone, the nurse should check the anal reflex by gently stroking the anal area and noticing a quick contraction of the sphincter. The other responses are not correct.

When the nurse is interviewing a preadolescent girl, which opening statement would be least threatening? A) "Do you have any questions about growing up?" B) "What has your mother told you about growing up?" C) "When did you notice that your body was changing?" D) "I remember being very scared when I got my period. How do you think you'll feel?"

ANS: C Try the open-ended, "When did you ... ?" rather than "Do you ... ?" This is less threatening because it implies that the topic is normal and unexceptional.

An 18-year-old patient is having her first pelvic examination. Which action by the nurse is appropriate? A) Invite her mother to be present during the examination. B) Avoid the lithotomy position this first time because it can be uncomfortable and embarrassing. C) Raise the head of the examination table and give her a mirror so that she can view the exam. D) Drape her fully, leaving the drape between her legs elevated to avoid embarrassing her with eye contact.

ANS: C Use the techniques of the educational or mirror pelvic examination. This is a routine examination with some modifications in attitude, position, and communication. First, the woman is considered an active participant, one who is interested in learning and in sharing decisions about her own health care. The woman props herself up on one elbow, or the head of the table is raised. Her other hand holds a mirror between her legs, above the examiner's hands. The woman can see all that the examiner is doing and has a full view of her genitalia. The mirror works well for teaching normal anatomy and its relationship to sexual behavior. You can ask her if she would like to have a family member, friend, or chaperone present for the examination. The drape should be pushed down between the woman's legs so that the nurse can see her face.

When a breastfeeding mother is diagnosed with a breast abscess, which of these instructions from the nurse is correct? The mother needs to: A) continue to nurse on both sides to encourage milk flow. B) discontinue nursing immediately to allow for healing. C) temporarily discontinue nursing on affected breast and manually express milk and discard it. D) temporarily discontinue nursing on affected breast but can manually express milk and give it to the baby.

ANS: C With a breast abscess, the patient must temporarily discontinue nursing on the affected breast, manually express the milk, and discard it. Nursing can continue on the unaffected side.

A 46-year-old woman is in the clinic for her annual gynecologic examination. She voices a concern about ovarian cancer because her mother and sister died of it. The nurse knows that which of these statements is correct regarding ovarian cancer? A) Ovarian cancer rarely has any symptoms. B) The Pap smear detects the presence of ovarian cancer. C) Women at high risk for ovarian cancer should have annual transvaginal ultrasonography for screening. D) Women over age 40 years should have a thorough pelvic examination every 3 years.

ANS: C With ovarian cancer, the patient may have abdominal pain, pelvic pain, increased abdominal size, bloating, and nonspecific gastrointestinal symptoms, or she may be asymptomatic. The Pap smear does not detect the presence of ovarian cancer. Annual transvaginal ultrasonography may detect ovarian cancer at an earlier stage in women who are at high risk for it.

A patient repeatedly seems to have difficulty coming up with a word. He says, I was on my way to work, and when I got there, the thing that you step into that goes up in the air was so full that I decided to take the stairs. The nurse will note on his chart that he is using or experiencing: a. Blocking b. Neologism c.Circumlocution d. Circumstantiality

ANS: Circumlocution Circumlocution is a roundabout expression, substituting a phrase when one cannot think of the name of the object.

A patient repeats, I feel hot. Hot, cot, rot, tot, got. I'm a spot. The nurse documents this as an illustration of: a. Blocking b. Clanging c. Echolalia d. Neologism

ANS: Clanging Clanging is word choice based on sound, not meaning, and includes nonsense rhymes and puns. (See Table 5-6 for the definitions of the other terms.)

The nurse is administering a Mini-Cog test to an older adult woman. When asked to draw a clock showing the time of 10:45, the patient drew a clock with the numbers out of order and with an incorrect time. This result indicates which finding? a. Cognitive impairment b. Amnesia c. Delirium d. Attention-deficit disorder

ANS: Cognitive impairment The Mini-Cog is a newer instrument that screens for cognitive impairment, often found with dementia. The result of an abnormal drawing of a clock and time indicates a cognitive impairment.

During an interview, the nurse notes that the patient gets up several times to wash her hands even though they are not dirty. This behavior is an example of: a. Social phobia b. Compulsive disorder c. Generalized anxiety disorder d. Posttraumatic stress disorder

ANS: Compulsive disorder Repetitive behaviors, such as hand washing, are behaviors that the person feels driven to perform in response to an obsession. The behaviors are aimed at preventing or reducing distress or preventing some dreaded event or situation.

7. During an assessment, the nurse asks a female patient, How many alcoholic drinks do you have a week? Which answer by the patient would indicate at-risk drinking? a. I may have one or two drinks a week. b. I usually have three or four drinks a week. c. Ill have a glass or two of wine every now and then. d. I have seven or eight drinks a week, but I never get drunk.

ANS: D For women, having seven or more drinks a week or three or more drinks per occasion is considered at-risk drinking, according to the National Institute on Alcohol Abuse and Alcoholism.

5. A woman who has just discovered that she is pregnant is in the clinic for her first obstetric visit. She asks the nurse, How many drinks a day is safe for my baby? The nurses best response is: a. You should limit your drinking to once or twice a week. b. Its okay to have up to two glasses of wine a day. c. As long as you avoid getting drunk, you should be safe. d. No amount of alcohol has been determined to be safe during pregnancy.

ANS: D No amount of alcohol has been determined to be safe for pregnant women. The potential adverse effects of alcohol use on the fetus are well known; women who are pregnant should be screened for alcohol use, and abstinence should be recommended.

During an assessment, the nurse notes that the patient's apical impulse is displaced laterally, and it is palpable over a wide area. This indicates: a. Systemic hypertension. b. Pulmonic hypertension. c. Pressure overload, as in aortic stenosis. d. Volume overload, as in heart failure.

ANS: D With volume overload, as in heart failure and cardiomyopathy, cardiac enlargement laterally displaces the apical impulse and is palpable over a wider area when left ventricular hypertrophy and dilation are present.

While performing a rectal examination, the nurse notices a firm, irregularly shaped mass. What should the nurse do next? A) Continue with the examination and document the finding in the chart. B) Instruct patient to return for repeat assessment in 1 month. C) Tell the patient that a mass was felt but it is nothing to worry about. D) Report the finding and refer the patient to a specialist for further examination.

ANS: D A firm or hard mass with irregular shape or rolled edges may signify carcinoma. Promptly report any mass that is discovered for further examination. The other responses are not correct

During an assessment of a 20-year-old man, the nurse finds a small palpable lesion with a tuft of hair located directly over the coccyx. The nurse knows that this lesion would most likely be a: A) polyp. B) pruritus ani. C) carcinoma. D) pilonidal cyst.

ANS: D A pilonidal cyst or sinus is a hair-containing cyst or sinus located in the midline over the coccyx or lower sacrum. It often opens as a dimple with a visible tuft of hair and, possibly, an erythematous halo. See Table 25-1 for more information, and also for description of pruritus ani. See Table 25-2 for descriptions of rectal polyps and carcinoma.

7. A patient's abdomen is bulging and stretched in appearance. The nurse should describe this finding as: A) obese. B) herniated. C) scaphoid. D) protuberant.

ANS: D A protuberant abdomen is rounded, bulging, and stretched. See Figure 21-7. A scaphoid abdomencaves inward

A 9-year-old girl is in the clinic for a sports physical. After some initial shyness she finally asks, "Am I normal? I don't seem to need a bra yet, but I have some friends who do. What if I never get breasts?" The nurse's best response would be: A) "Don't worry, you still have plenty of time to develop." B) "I know just how you feel, I was a late bloomer myself. Just be patient and they will grow." C) "You will probably get your periods before you notice any significant growth in your breasts." D) "I understand that it is hard to feel different from your friends. Breasts usually develop between 8 and 10 years of age."

ANS: D Adolescent breast development usually begins between 8 and 10 years of age. The nurse should not belittle the girl's feelings by using statements like "don't worry" or by sharing personal experiences. The beginning of breast development precedes menarche by about 2 years.

A 65-year-old patient remarks that she just can't believe that her breasts sag so much. She states it must be from lack of exercise. What explanation should the nurse offer her? A) After menopause, only women with large breasts experience sagging. B) After menopause, sagging is usually due to decreased muscle mass within the breast. C) After menopause, a diet that is high in protein will help maintain muscle mass, which keeps the breasts from sagging. D) After menopause, the glandular and fat tissue atrophies, causing breast size and elasticity to diminish, resulting in breasts that sag.

ANS: D After menopause, the glandular tissue atrophies and is replaced with connective tissue. The fat envelope atrophies also, beginning in the middle years and becoming marked in the eighth and ninth decades. These changes decrease breast size and elasticity, so the breasts droop and sag, looking flattened and flabby

The nurse is teaching a pregnant woman about breast milk. Which statement by the nurse is correct? A) "Your breast milk is present immediately after delivery of the baby." B) "Breast milk is rich in protein and sugars (lactose) but has very little fat." C) "The colostrum, which is present right after birth, does not contain the same nutrition as breast milk does." D) "You may notice a thick, yellow fluid expressed from your breasts as early as the fourth month of pregnancy."

ANS: D After the fourth month, colostrum may be expressed. This thick yellow fluid is the precursor of milk, and it contains the same amount of protein and lactose but practically no fat. The breasts produce colostrum for the first few days after delivery. It is rich with antibodies that protect the newborn against infection, so breastfeeding is importat

13. The physician comments that a patient has abdominal borborygmi. The nurse knows that this term refers to: A) a loud continuous hum. B) a peritoneal friction rub. C) hypoactive bowel sounds. D) hyperactive bowel sounds.

ANS: D Borborygmi is the term used for hyperperistalsis when the person actually feels his or her stomach growling.

A 35-year-old woman is at the clinic for a gynecologic examination. During the examination, she asks the nurse, "How often do I need to have this Pap test done?" Which reply by the nurse is correct? A) "It depends. Do you smoke?" B) "This will need to be done annually until you are 65." C) "If you have 2 consecutive normal Pap tests, then you can wait 5 years between tests." D) "After age 30, if you have 3 consecutive normal Pap tests, then you may be screened every 2 to 3 years."

ANS: D Cervical cancer screening with the Pap test continues annually until age 30. After age 30, if the woman has 3 consecutive normal Pap tests, then women may be screened every 2 to 3 years.

8. The nurse is describing a scaphoid abdomen. To the horizontal plane, a scaphoid contour of the abdomen depicts a _____ profile. A) flat B) convex C) bulging D) concave

ANS: D Contour describes the profile of the abdomen from the rib margin to the pubic bone; a scaphoid contour is one that is concave from a horizontal plane. See Figure 21-7.

25. To detect diastasis recti, the nurse should have the patient perform which of these maneuvers? A) Relax in the supine position. B) Raise the arms in the left lateral position. C) Raise the arms over the head while supine. D) Raise the head while remaining supine.

ANS: D Diastasis recti is a separation of the abdominal rectus muscles, which can occur congenitally, as a result of pregnancy, or from marked obesity. This is assessed by having the patient raise the head while remaining supine.

The nurse is performing a breast examination. Which of these statements best describes the correct procedure to use when screening for nipple and skin retraction during a breast examination? Have the woman: A) bend over and touch her toes. B) lie down on her left side and notice any retraction. C) shift from a supine position to a standing position; notice any lag or retraction. D) slowly lift her arms above her head and note any retraction or lag in movement.

ANS: D Direct the woman to change position while checking the breasts for skin retraction signs. First ask her to lift her arms slowly over her head. Both breasts should move up symmetrically. Retraction signs are due to fibrosis in the breast tissue, usually caused by growing neoplasms. The nurse should notice if there is a lag in movement of one breast.

4. The nurse suspects that a patient has a distended bladder. How should the nurse assess for this condition? A) Percuss and palpate in the lumbar region. B) Inspect and palpate in the epigastric region. C) Auscultate and percuss in the inguinal region. D) Percuss and palpate the midline area above the suprapubic bone.

ANS: D Dull percussion sounds would be elicited over a distended bladder, and the hypogastric area would seem firm to palpation.

10. The nurse is giving report to the next shift and is using the SBAR framework for communication. Which of these statements reflects the Background portion of the report? A. "I'm worried that his gastrointestinal bleeding is getting worse." B. "We need an order for oxygen." C. "My name is Ms. Smith and I'm giving report on Mrs. X in room 1104." D. "He is four days postoperative and his incision is open to air."

ANS: D During the Background portion, the nurse should state data pertinent to the moment's problem such as the condition of the patient's incision. During the Situation portion, the nurse provides his or her name and the patient's name. During the Assessment portion, the nurse states what her or she thinks is happening (i.e., the gastrointestinal bleeding). During the Recommendation portion, the nurse should offer probable solutions or orders that may be implemented.

5. The nurse is aware that one change that may occur in the gastrointestinal system of an aging adult is: A) increased salivation. B) increased liver size. C) increased esophageal emptying. D) decreased gastric acid secretion.

ANS: D Gastric acid secretion decreases with aging. As one ages, salivation decreases, esophageal emptying is delayed, and liver size decreases

A 22-year-old woman is being seen at the clinic for problems with vulvar pain, dysuria, and fever. On physical examination, the nurse notices clusters of small, shallow vesicles with surrounding erythema on the labia. There is also inguinal lymphadenopathy present. The most likely cause of these lesions is: A) pediculosis pubis. B) contact dermatitis. C) human papillomavirus. D) herpes simplex virus type 2.

ANS: D Herpes simplex virus type 2 presents with clusters of small, shallow vesicles with surrounding erythema that erupt on the genital areas. There is also the presence of inguinal lymphadenopathy. The individual reports local pain, dysuria, and fever. See Table 26-2 for more information and descriptions of the other conditions.

6. A 22-year-old man comes to the clinic for an examination after falling off his motorcycle and landing on his left side on the handlebars. The nurse suspects that he may have injured his spleen. Which of these statements is true regarding assessment of the spleen in this situation? A) The spleen can be enlarged as a result of trauma. B) The spleen is normally felt upon routine palpation. C) If an enlarged spleen is noticed, then the nurse should palpate thoroughly to determine size. D) An enlarged spleen should not be palpated because it can rupture easily.

ANS: D If an enlarged spleen is felt, then the nurse should refer the person but should not continue to palpate it. An enlarged spleen is friable and can rupture easily with overpalpation

38. During an abdominal assessment, the nurse tests for a fluid wave. A positive fluid wave test occurs with: A) splenomegaly. B) distended bladder. C) constipation. D) ascites.

ANS: D If ascites (fluid in the abdomen) is present, then the examiner will feel a fluid wave when assessing the abdomen. A fluid wave is not present with splenomegaly, a distended bladder, or constipation

Which of these clinical situations would the nurse consider to be outside normal limits? A) A patient has had one pregnancy. She states that she believes she may be entering menopause. Her breast examination reveals breasts that are soft and sag slightly. B) A patient has never been pregnant. Her breast examination reveals large pendulous breasts that have a firm, transverse ridge along the lower quadrant in both breasts. C) A patient has never been pregnant. She reports that she should begin her period tomorrow. Her breast examination reveals breast tissue that is nodular and somewhat engorged. She states that the examination was slightly painful. D) A patient has had two pregnancies and she breastfed both of her children. Her youngest child is now 10 years old. Her breast examination reveals breast tissue that is somewhat soft and she has a small amount of thick yellow discharge from both nipples.

ANS: D In nulliparous women, normal breast tissue feels firm, smooth, and elastic; after pregnancy, the tissue feels softer and looser. If any discharge appears, the nurse should note its color and consistency. Except in pregnancy and lactation, discharge is abnormal. Premenstrual engorgement is normal, and consists of a slight enlargement, tenderness to palpation, and a generalized nodularity. A firm, transverse ridge of compressed tissue in the lower quadrants, known as the inframammary ridge, is especially noticeable in large breasts.

During an examination, the nurse asks the patient to perform the Valsalva maneuver and notices that the patient has a moist, red, doughnut-shaped protrusion from the anus. The nurse knows that this would be consistent with: A) a rectal polyp. B) hemorrhoids. C) a rectal fissure. D) rectal prolapse.

ANS: D In rectal prolapse, the rectal mucous membrane protrudes through the anus, appearing as a moist red doughnut with radiating lines. It occurs following a Valsalva maneuver, such as straining at stool, or with exercise. See Table 25-1. For a description of rectal polyps, see Table 25-2. See Table 25-1 for descriptions of rectal fissure and hemorrhoids.

A 65-year-old woman is in the office for routine gynecologic care. She had a complete hysterectomy 3 months ago after cervical cancer was detected. The nurse knows that which of these statements is true with regard to this visit? A) Her cervical mucosa will be red and dry looking. B) She will not need to have a Pap smear done. C) The nurse can expect to find that her uterus will be somewhat enlarged and her ovaries small and hard. D) The nurse should plan to lubricate the instruments and the examining hand well to avoid a painful examination.

ANS: D In the aging adult woman, natural lubrication is decreased; to avoid a painful examination, the nurse should take care to lubricate instruments and the examining hand adequately. Menopause, with the resulting decrease in estrogen production, shows numerous physical changes. The cervix shrinks and looks pale and glistening. With the bimanual examination, the uterus feels smaller and firmer and the ovaries are not palpable normally. Women should continue cervical cancer screening up to age 70 years if they have an intact cervix and are in good health. Women who have had a total hysterectomy for benign gynecologic disease do not need cervical cancer screening, but if the hysterectomy was done for cervical cancer, then Pap tests should continue until the patient has a 10-year history of no abnormal results.

A 13-year-old girl is visiting the clinic for a sports physical. The nurse should remember to include which of these tests in the examination? A) Test for occult blood B) The Valsalva maneuver C) Internal palpation of the anus D) Inspection of the perianal area

ANS: D Inspect the perianal region of the school-aged child and adolescent during examination of the genitalia. Internal palpation is not performed routinely at this age. Testing for occult blood and doing the Valsalva maneuver are also not necessary.

A 22-year-old woman has been considering using oral contraceptives. As a part of her history, the nurse should ask: A) "Do you have a history of heart murmurs?" B) "Will you be in a monogamous relationship?" C) "Have you thought this choice through carefully?" D) "If you smoke, how many cigarettes do you smoke per day?"

ANS: D Oral contraceptives, together with cigarette smoking, increase the risk for cardiovascular side effects. If cigarettes are used, then the nurse should assess smoking history. The other questions are not appropriate.

35. The nurse is assessing a patient for possible peptic ulcer disease and knows that which condition often causes this problem? A) Hypertension B) Streptococcus infections C) History of constipation and frequent laxative use D) Frequent use of nonsteroidal antiinflammatory drugs

ANS: D Peptic ulcer disease occurs with frequent use of nonsteroidal antiinflammatory drugs, alcohol use, smoking, and Helicobacter pylori infection.

The nurse is reviewing risk factors for breast cancer. Which of these women have risk factors that place them at a higher risk for breast cancer? A) 37 year old who is slightly overweight B) 42 year old who has had ovarian cancer C) 45 year old who has never been pregnant D) 65 year old whose mother had breast cancer

ANS: D Risk factors for breast cancer include having a first-degree relative with breast cancer (mother, sister, or daughter) and being older than 50 years. Refer to Table 17- 2 for other risk factors.

A woman who is 8 weeks pregnant is in the clinic for a checkup. The nurse reads on her chart that her cervix is softened and looks cyanotic. The nurse knows that the woman is exhibiting _____ sign and _____ sign. A) Tanner's; Hegar's B) Hegar's; Goodell's C) Chadwick's; Hegar's D) Goodell's; Chadwick's

ANS: D Shortly after the first missed menstrual period, the female genitalia show signs of the growing fetus. The cervix softens (Goodell's sign) at 4 to 6 weeks, and the vaginal mucosa and cervix look cyanotic (Chadwick's sign) at 8 to 12 weeks. These changes occur because of increased vascularity and edema of the cervix and hypertrophy and hyperplasia of the cervical glands. Hegar's sign occurs when the isthmus of the uterus softens at 6 to 8 weeks. Tanner's sign is not a correct response.

During a health history, a 22-year old woman asks, "Can I get that vaccine for HPV? I have genital warts and I'd like them to go away!" What is the nurse's best response? A) "The HPV vaccine is for girls and women ages 9 to 26, so we can start that today." B) "This vaccine is only for girls who have not started to have intercourse yet." C) "Let's check with the physician to see if you are a candidate for this vaccine." D) "The vaccine cannot protect you if you already have an HPV infection."

ANS: D The HPV (human papillomavirus) vaccine is appropriate for girls and women age 9 to 26 and is given to prevent cervical cancer by preventing HPV infections before girls become sexually active. However, it cannot protect the woman if an HPV infection is already present.

Which of these statements about the anal canal is true? The anal canal: A) is about 2 cm long in the adult. B) slants backward toward the sacrum. C) contains hair and sebaceous glands. D) is the outlet for the gastrointestinal tract.

ANS: D The anal canal is the outlet for the gastrointestinal tract, and it is about 3.8 cm long in the adult. It is lined with a modified skin that does not contain hair or sebaceous glands, and it slants forward toward the umbilicus.

While doing an assessment of the perianal area of a patient, the nurse notices that the pigmentation of anus is darker than surrounding skin, the anal opening is closed, and there is a skin sac that is shiny and blue. The patient mentioned that he has had pain with bowel movements and has noted some spots of blood occasionally. What would this assessment and history be most likely to indicate? A) Anal fistula B) Pilonidal cyst C) Rectal prolapse D) Thrombosed hemorrhoid

ANS: D The anus normally looks moist and hairless, with coarse folded skin that is more pigmented than the perianal skin. The anal opening is tightly closed. The shiny blue skin sac indicates a thrombosed hemorrhoid.

32. A 45-year-old man is in the clinic for a physical examination. During the abdominal assessment, the nurse percusses the abdomen and notices an area of dullness above the right costal margin of about 10 cm. The nurse should: A) document the presence of hepatomegaly. B) ask additional history questions regarding his alcohol intake. C) describe this as an enlarged liver and refer him to a physician. D) consider this a normal finding and proceed with the examination.

ANS: D The average liver span in the midclavicular line is 6 to 12 cm. Men and taller individuals are at the upper end of this range. Women and shorter individuals are at the lower end of this range. A liver span of 10 cm is within normal limits for this individual

During the assessment of an 18-month-old child, the mother expresses concern to the nurse about the infant's inability to toilet train. What would be the nurse's best response? A) "Some children are just more difficult to train, so I wouldn't worry about it yet." B) "Have you considered reading any of the books on toilet training? They can be very helpful." C) "This could mean there is a problem in your baby's development. We'll watch her closely for the next few months." D) "The nerves that will allow your baby to have control over the passing of stools are not developed until at least 18 to 24 months of age."

ANS: D The infant passes stools by reflex. Voluntary control of the external anal sphincter cannot occur until the nerves supplying the area have become fully myelinated, usually around 1 1/2 to 2 years of age. Toilet training usually starts after the age of 2.

2. During an assessment, the nurse is unable to palpate pulses in the left lower leg. What should the nurse do next? A. Document that the pulses are not palpable. B. Reassess the pulses in 1 hour. C. Have the patient turn to the side and then palpate for the pulses again. D. Use a Doppler device to assess the pulses

ANS: D The nurse should be prepared to assess pulses in the lower extremities by Doppler measurement if they cannot be detected by palpation.

A 60-year-old man has just been told he has benign prostatic hypertrophy. He has a friend who just died from cancer of the prostate, and he is concerned this will happen to him. How should the nurse respond? A) "The swelling in your prostate is only temporary and will go away." B) "We will treat you with chemotherapy so we can control the cancer." C) "It would be very unusual for a man your age to have cancer of the prostate." D) "The enlargement of your prostate is caused by hormone changes and not cancer."

ANS: D The prostate gland commonly starts to enlarge during the middle adult years. This benign prostatic hypertrophy (BPH) is present in 1 out of 10 males at the age of 40 years and increases with age. It is thought that the hypertrophy is caused by hormonal imbalance that leads to the proliferation of benign adenomas. The other responses are not appropriate.

2. Which structure is located in the left lower quadrant of the abdomen? A) Liver B) Duodenum C) Gallbladder D) Sigmoid colon

ANS: D The sigmoid colon is located in the left lower quadrant of the abdomen.

A 25-year-old woman comes to the emergency department with a sudden fever of 101° F(38.3C) and abdominal pain. Upon examination, the nurse notices that she has rigid, boardlike lower abdominal musculature. When the nurse tries to perform a vaginal examination, the patient has severe pain when the uterus and cervix are moved. The nurse knows that these signs and symptoms are suggestive of: A) endometriosis. B) uterine fibroids. C) ectopic pregnancy. D) pelvic inflammatory disease.

ANS: D These signs and symptoms are suggestive of acute pelvic inflammatory disease, also known as acute salpingitis. See Table 26-7. For description of endometriosis and uterine fibroids, see Table 26-6; for description of ectopic pregnancy, see Table 26-7.

During a history, the patient states, "It really hurts back there, and sometimes it itches, too. I have even seen blood on the tissue when I have a bowel movement. Is there something there?" The nurse should expect to see which of these upon examination of the anus? A) Rectal prolapse B) Internal hemorrhoid C) External hemorrhoid that has resolved D) External hemorrhoid that is thrombosed

ANS: D These symptoms are consistent with an external hemorrhoid. An external hemorrhoid, when thrombosed, contains clotted blood and becomes a painful, swollen, shiny blue mass that itches and bleeds with defecation. When the external hemorrhoid resolves, it leaves a flabby, painless skin sac around the anal orifice. An internal hemorrhoid is not palpable, but may appear as a red mucosal mass when the person performs a Valsalva maneuver. A rectal prolapse appears as a moist, red doughnut with radiating lines

When testing stool for occult blood, the nurse is aware that a false-positive result may occur with: A) absent bile pigment. B) increased fat content. C) increased ingestion of iron medication. D) a large amount of red meat within the last 3 days.

ANS: D When testing for occult blood, a false-positive finding may occur if the person has ingested significant amounts of red meat within 3 days of the test. Absent bile pigment causes the stools to be gray or tan in color. Increased fat content causes the stool to be pale, yellow, and greasy. Increased ingestion of iron medication causes the stool to be black in color.

During morning rounds, the nurse asks a patient, How are you today? The patient responds, You today, you today, you today! and mumbles the words. This speech pattern is an example of: a. Echolalia b. Clanging c. Word salad d. Perseveration

ANS: Echolalia Echolalia occurs when a person imitates or repeats another's words or phrases, often with a mumbling, mocking, or a mechanical tone.

A 55-year-old man is experiencing severe pain of sudden onset in the scrotal area. It is somewhat relieved by elevation. On examination the nurse notices an enlarged, red scrotum that is very tender to palpation. It is difficult to distinguish the epididymis from the testis, and the scrotal skin is thick and edematous. This description is consistent with which of these?

ANS: Epididymitis Epididymitis presents as severe pain of sudden onset in the scrotum that is somewhat relieved by elevation. On examination, the scrotum is enlarged, reddened, and exquisitely tender. The epididymis is enlarged and indurated and may be hard to distinguish from the testis. The overlying scrotal skin may be thick and edematous. See Table 24-6 for more information and for descriptions of the other terms.

During an assessment of a healthy adult, where would the nurse expect to palpate the apical impulse? a. Third left intercostal space at the midclavicular line b. Fourth left intercostal space at the sternal border c. Fourth left intercostal space at the anterior axillary line d. Fifth left intercostal space at the midclavicular line

ANS: Fifth left intercostal space at the midclavicular line The apical impulse should occupy only one intercostal space, the fourth or fifth, and it should be at or medial to the midclavicular line.

The nurse is preparing to conduct a mental status examination. Which statement is true regarding the mental status examination? a. A patients family is the best resource for information about the patients coping skills. b. Gathering mental status information during the health history interview is usually sufficient. c. Integrating the mental status examination into the health history interview takes an enormous amount of extra time. d. To get a good idea of the patients level of functioning, performing a complete mental status examination is usually necessary.

ANS: Gathering mental status information during the health history interview is usually sufficient. The full mental status examination is a systematic check of emotional and cognitive functioning. The steps described, however, rarely need to be taken in their entirety. Usually, one can assess mental status through the context of the health history interview.

The nurse is planning to assess new memory with a patient. The best way for the nurse to do this would be to: a. Administer the FACT test. b. Ask him to describe his first job. c. Give him the Four Unrelated Words Test. d. Ask him to describe what television show he was watching before coming to the clinic.

ANS: Give him the Four Unrelated Words Test. Ask questions that can be corroborated, which screens for the occasional person who confabulates or makes up answers to fill in the gaps of memory loss. The Four Unrelated Words Test tests the persons ability to lay down new memories and is a highly sensitive and valid memory test.

The nurse discovers speech problems in a patient during an assessment. The patient has spontaneous speech, but it is mostly absent or is reduced to a few stereotypical words or sounds. This finding reflects which type of aphasia? a. Global b. Brocas c. Dysphonic d. Wernickes

ANS: Global Global aphasia is the most common and severe form of aphasia. Spontaneous speech is absent or reduced to a few stereotyped words or sounds, and prognosis for language recovery is poor. (Brocas aphasia and Wernickes aphasia are described in Table 5-4.) Dysphonic aphasia is not a valid condition.

A patient describes feeling an unreasonable, irrational fear of snakes. His fear is so persistent that he can no longer comfortably look at even pictures of snakes and has made an effort to identify all the places he might encounter a snake and avoids them. The nurse recognizes that he: a. Has a snake phobia. b. Is a hypochondriac; snakes are usually harmless. c. Has an obsession with snakes. d. Has a delusion that snakes are harmful, which must stem from an early traumatic incident involving snakes.

ANS: Has a snake phobia. A phobia is a strong, persistent, irrational fear of an object or situation; the person feels driven to avoid it. (See Table 5-7 for the definitions of the other terms.)

When the nurse is performing a genital examination on a male patient, which of these actions is correct?

ANS: Have the patient shift his weight onto the left (unexamined) leg when palpating for a hernia on the right side. When palpating for the presence of a hernia on the right side, ask the male patient to shift his weight onto the left (unexamined) leg. It is not appropriate to auscultate for a bruit over the scrotum. When palpating for lymph notes, palpate the horizontal chain. The inguinal canal should be palpated whether or not a bulge is present.

During a cardiac assessment on a 38 year-old patient in the hospital for "chest pain," the nurse finds the following: jugular vein pulsations 4 cm above sternal angle when he is elevated at 45 degrees, blood pressure 98/60 mm Hg, heart rate 130 beats per minute, ankle edema, difficulty in breathing when supine, and an S3 on auscultation. Which of these conditions best explains the cause of these findings? a. Fluid overload b. Atrial septal defect c. MI d. Heart failure

ANS: Heart failure Heart failure causes decreased cardiac output when the heart fails as a pump and the circulation becomes backed up and congested. Signs and symptoms include dyspnea, orthopnea, paroxysmal nocturnal dyspnea, decreased blood pressure, dependent and pitting edema; anxiety; confusion; jugular vein distention; and fatigue. The S3 is associated with heart failure and is always abnormal after age 35. The S3 may be the earliest sign of heart failure.

The nurse is performing a mental status assessment on a 5-year-old girl. Her parents are undergoing a bitter divorce and are worried about the effect it is having on their daughter. Which action or statement might lead the nurse to be concerned about the girls mental status? a. She clings to her mother whenever the nurse is in the room. b. She appears angry and will not make eye contact with the nurse. c. Her mother states that she has begun to ride a tricycle around their yard. d. Her mother states that her daughter prefers to play with toddlers instead of kids her own age while in daycare.

ANS: Her mother states that her daughter prefers to play with toddlers instead of kids her own age while in daycare. The mental status assessment of infants and children covers behavioral, cognitive, and psycho-social development and examines how the child is coping with his or her environment. Essentially, the nurse should follow the same Association for Behavioral and Cognitive Therapies (ABCT) guidelines as those for the adult, with special consideration for developmental milestones. The best examination technique arises from a thorough knowledge of the developmental milestones (described in Chapter 2). Abnormalities are often problems of omission (e.g., the child does not achieve a milestone as expected).

During a mental status examination, the nurse wants to assess a patients affect. The nurse should ask the patient which question? a. How do you feel today? b. Would you please repeat the following words? c. Have these medications had any effect on your pain? d. Has this pain affected your ability to get dressed by yourself?

ANS: How do you feel today? Judge mood and affect by body language and facial expression and by directly asking, How do you feel today? or How do you usually feel? The mood should be appropriate to the persons place and condition and should appropriately change with the topics.

A patient states, I feel so sad all of the time. I cant feel happy even doing things I used to like to do. He also states that he is tired, sleeps poorly, and has no energy. To differentiate between a dysthymic disorder and a major depressive disorder, the nurse should ask which question? a. Have you had any weight changes? b. Are you having any thoughts of suicide? c. How long have you been feeling this way? d. Are you having feelings of worthlessness?

ANS: How long have you been feeling this way? Major depressive disorder is characterized by one or more major depressive episodes, that is, at least 2 weeks of depressed mood or loss of interest accompanied by at least four additional symptoms of depression. Dysthymic disorder is characterized by at least 2 years of depressed mood for more days than not, accompanied by additional depressive symptoms.

The nurse is assessing orientation in a 79-year-old patient. Which of these responses would lead the nurse to conclude that this patient is oriented? a. I know my name is John. I couldn't tell you where I am. I think it is 2010, though. b. I know my name is John, but to tell you the truth, I get kind of confused about the date. c. I know my name is John; I guess I'm at the hospital in Spokane. No, I don't know the date. d. I know my name is John. I am at the hospital in Spokane. I couldn't tell you what date it is, but I know that it is February of a new year 2010.

ANS: I know my name is John. I am at the hospital in Spokane. I couldn't tell you what date it is, but I know that it is February of a new year 2010. Many aging persons experience social isolation, loss of structure without a job, a change in residence, or some short-term memory loss. These factors affect orientation, and the person may not provide the precise date or complete name of the agency. You may consider aging persons oriented if they generally know where they are and the present period. They should be considered oriented to time if the year and month are correctly stated. Orientation to place is accepted with the correct identification of the type of setting (e.g., hospital) and the name of the town.

The nurse is conducting a patient interview. Which statement made by the patient should the nurse more fully explore during the interview? a. I sleep like a baby. b. I have no health problems. c. I never did too good in school. d. I am not currently taking any medications.

ANS: I never did too good in school. In every mental status examination, the following factors from the health history that could affect the findings should be noted: any known illnesses or health problems, such as alcoholism or chronic renal disease; current medications, the side effects of which may cause confusion or depression; the usual educational and behavioral level, noting this level as the patients normal baseline and not expecting a level of performance on the mental status examination to exceed it; and responses to personal history questions, indicating current stress, social interaction patterns, and sleep habits.

A patient has been diagnosed with schizophrenia. During a recent interview, he shows the nurse a picture of a man holding a decapitated head. He describes this picture as horrifying but then laughs loudly at the content. This behavior is a display of: a. Confusion b. Ambivalence c. Depersonalization d. Inappropriate affect

ANS: Inappropriate affect An inappropriate affect is an affect clearly discordant with the content of the persons speech. (See Table 5-5 for the definitions of the other terms.)

In assessing a 70-year-old man, the nurse finds the following: blood pressure 140/100 mm Hg; heart rate 104 and slightly irregular; split S2. Which of these findings can be explained by expected hemodynamic changes related to age? a. Increase in resting heart rate b. Increase in systolic blood pressure c. Decrease in diastolic blood pressure d. Increase in diastolic blood pressure

ANS: Increase in systolic blood pressure With aging, there is an increase in systolic blood pressure. No significant change in diastolic pressure occurs with age. No change in resting heart rate occurs with aging. Cardiac output at rest is not changed with aging.

The nurse is performing the Denver II screening test on a 12-month-old infant during a routine well-child visit. The nurse should tell the infants parents that the Denver II: a. Tests three areas of development: cognitive, physical, and psychological b. Will indicate whether the child has a speech disorder so that treatment can begin. c. Is a screening instrument designed to detect children who are slow in development. d. Is a test to determine intellectual ability and may indicate whether problems will develop later in school.

ANS: Is a screening instrument designed to detect children who are slow in development. The Denver II is a screening instrument designed to detect developmental delays in infants and preschoolers. It tests four functions: gross motor, language, fine motor-adaptive, and personal-social. The Denver II is not an intelligence test; it does not predict current or future intellectual ability. It is not diagnostic; it does not suggest treatment regimens.

The nurse is assessing the skin of a patient who has AIDS and notices multiple patch-like lesions on the temple and beard area that are faint pink in color. The nurse recognizes these lesions as: a. Measles (rubeola). b. Kaposis sarcoma. c. Angiomas. d. Herpes zoster.

ANS: Kaposi's sarcoma. Kaposi's sarcoma is a vascular tumor that, in early stages, appears as multiple, patch-like, faint pink lesions over the patient's temple and beard areas. Measles is characterized by a red-purple maculopapular blotchy rash which appears on third or fourth day of illness. Rash appears first behind ears and spreads over face, then over neck, trunk, arms, and legs. Cherry (senile) angiomas are small (1 to 5 mm), smooth, slightly raised bright red dots that commonly appear on the trunk in all adults over 30 years old. Herpes zoster causes vesicles that are elevated with a cavity containing clear fluid, up to 1 cm in size.

Which term refers to a wound produced by the tearing or splitting of body tissue, usually from blunt impact over a bony surface?

ANS: Laceration The term laceration fits this definition. An abrasion is caused by rubbing of the skin or mucous membrane. A contusion is injury to tissues without breakage of skin, and a hematoma is a localized collection of extravasated blood.

A patient drifts off to sleep when she is not being stimulated. The nurse can easily arouse her by calling her name, but the patient remains drowsy during the conversation. The best description of this patients level of consciousness would be: a. Lethargic b. Obtunded c. Stuporous d. Semialert

ANS: Lethargic Lethargic (or somnolent) is when the person is not fully alert, drifts off to sleep when not stimulated, and can be aroused when called by name in a normal voice but looks drowsy. He or she appropriately responds to questions or commands, but thinking seems slow and fuzzy. He or she is inattentive and loses the train of thought. Spontaneous movements are decreased. (See Table 5-3 for the definitions of the other terms.)

A 20-year-old construction worker has been brought into the emergency department with heat stroke. He has delirium as a result of a fluid and electrolyte imbalance. For the mental status examination, the nurse should first assess the patients: a. Affect and mood b. Memory and affect c. Language abilities d. Level of consciousness and cognitive abilities

ANS: Level of consciousness and cognitive abilities Delirium is a disturbance of consciousness (i.e., reduced clarity of awareness of the environment) with reduced ability to focus, sustain, or shift attention. Delirium is not an alteration in mood, affect, or language abilities.

When the nurse is auscultating the carotid artery for bruits, which of these statements reflects correct technique? a. While listening with the bell of the stethoscope, the patient is asked to take a deep breath and hold it. b. While auscultating one side with the bell of the stethoscope, the carotid artery is palpated on the other side to check pulsations. c. While lightly applying the bell of the stethoscope over the carotid artery and listening, the patient is asked to take a breath, exhale, and briefly hold it. d. While firmly placing the bell of the stethoscope over the carotid artery and listening, the patient is asked to take a breath, exhale, and briefly hold it.

ANS: Lightly apply the bell of the stethoscope over the carotid artery, and while listening, have the patient take a breath, exhale, and hold it briefly. The correct technique for auscultating the carotid artery for bruits involves the nurse lightly applying the bell of the stethoscope over the carotid artery at three levels. While listening, the nurse asks the patient take a breath, exhale, and briefly hold it. Holding the breath on inhalation will also tense the levator scapulae muscles, which makes it hard to hear the carotid arteries. Examining only one carotid artery at a time will avoid compromising arterial blood flow to the brain. Pressure over the carotid sinus, which may lead to decreased heart rate, decreased blood pressure, and cerebral ischemia with syncope, should be avoided.

The nurse is preparing to auscultate for heart sounds. Which technique is correct? a. Listening to the sounds at the aortic, tricuspid, pulmonic, and mitral areas b. Listening by inching the stethoscope in a rough Z pattern, from the base of the heart across and down, then over to the apex c. Listening to the sounds only at the site where the apical pulse is felt to be the strongest d. Listening for all possible sounds at a time at each specified area

ANS: Listen by inching the stethoscope in a rough Z pattern, from the base of the heart across and down, then over to the apex. Do not limit auscultation of breath sounds to only four locations. Sounds produced by the valves may be heard all over the precordium. Inch the stethoscope in a rough Z pattern from the base of the heart across and down, then over to the apex. Or, start at the apex and work your way up. See Figure 19-22. Listen selectively to one sound at a time.

A few days after a summer hiking trip, a 25-year-old man comes to the clinic with a rash. On examination, the nurse notes that the rash is red, macular, with a bull's eye pattern across his midriff and behind his knees. The nurse suspects: a. Rubeola. b. Lyme disease. c. Allergy to mosquito bites. d. Rocky Mountain spotted fever.

ANS: Lyme disease. Lyme disease occurs in people who spend time outdoors in May through September. The first state has the distinctive bull's eye, a red macular or papular rash that radiates from the site of the tick bite with some central clearing, 5 cm or larger, usually in the axilla, midriff, inguina, or behind the knee, with regional lymphadenopathy.

During reporting, the nurse hears that a patient is experiencing hallucinations. Which is an example of a hallucination? a. Man believes that his dead wife is talking to him. b. Woman hears the doorbell ring and goes to answer it, but no one is there. c. Child sees a man standing in his closet. When the lights are turned on, it is only a dry cleaning bag. d. Man believes that the dog has curled up on the bed, but when he gets closer he sees that it is a blanket.

ANS: Man believes that his dead wife is talking to him Hallucinations are sensory perceptions for which no external stimuli exist. They may strike any sense: visual, auditory, tactile, olfactory, or gustatory.

The nurse is assessing a 75-year-old man. As the nurse begins the mental status portion of the assessment, the nurse expects that this patient: a. Will have no decrease in any of his abilities, including response time. b. Will have difficulty on tests of remote memory because this ability typically decreases with age. c. May take a little longer to respond, but his general knowledge and abilities should not have declined. d. Will exhibit had a decrease in his response time because of the loss of language and a decrease in general knowledge.

ANS: May take a little longer to respond, but his general knowledge and abilities should not have declined. The aging process leaves the parameters of mental status mostly intact. General knowledge does not decrease, and little or no loss in vocabulary occurs. Response time is slower than in a youth. It takes a little longer for the brain to process information and to react to it. Recent memory, which requires some processing, is somewhat decreased with aging, but remote memory is not affected.

The nurse is aware that which of these statements is true regarding the incidence of testicular cancer?

ANS: Men with a history of cryptorchidism are at greatest risk for development of testicular cancer. Men with undescended testicles (cryptorchidism) are at greatest risk for development of testicular cancer. The overall incidence of testicular cancer is rare. Testicular cancer has no early symptoms. When detected early and treated before metastasis, the cure rate is almost 100%.

The nurse is performing a mental status examination. Which statement is true regarding the assessment of mental status? a. Mental status assessment diagnoses specific psychiatric disorders. b. Mental disorders occur in response to everyday life stressors. c. Mental status functioning is inferred through the assessment of an individuals behaviors. d. Mental status can be directly assessed, similar to other systems of the body (e.g., heart sounds, breath sounds).

ANS: Mental status functioning is inferred through the assessment of an individuals behaviors. Mental status functioning is inferred through the assessment of an individuals behaviors. It cannot be directly assessed like the characteristics of the skin or heart sounds.

A 19-year-old woman comes to the clinic at the insistence of her brother. She is wearing black combat boots and a black lace nightgown over the top of her other clothes. Her hair is dyed pink with black streaks throughout. She has several pierced holes in her nares and ears and is wearing an earring through her eyebrow and heavy black makeup. The nurse concludes that: a. She probably does not have any problems. b. She is only trying to shock people and that her dress should be ignored. c. She has a manic syndrome because of her abnormal dress and grooming. d. More information should be gathered to decide whether her dress is appropriate.

ANS: More information should be gathered to decide whether her dress is appropriate. Grooming and hygiene should be noted the person is clean and well groomed, hair is neat and clean, women have moderate or no makeup, and men are shaved or their beards or mustaches are well groomed. Care should be taken when interpreting clothing that is disheveled, bizarre, or in poor repair because these sometimes reflect the persons economic status or a deliberate fashion trend.

Which of these findings would the nurse expect to notice during a cardiac assessment on a 4-year-old child? a. S3 when sitting up b. Persistent tachycardia above 150 beats per minute c. Murmur at the second left intercostal space when supine d. Palpable apical impulse in the fifth left intercostal space lateral to midclavicular line

ANS: Murmur at second left intercostal space when supine Some murmurs are common in healthy children or adolescents and are termed innocent or functional. The innocent murmur is heard at the second or third left intercostal space and disappears with sitting, and the young person has no associated signs of cardiac dysfunction.

While counting the apical pulse on a 16-year-old patient, the nurse notices an irregular rhythm. His rate speeds up on inspiration and slows on expiration. What would be the nurse's response? a. Talk with the patient about his intake of caffeine. b. Perform an electrocardiogram after the examination. c. No further response is needed because sinus arrhythmia can occur normally. d. Refer the patient to a cardiologist for further testing.

ANS: No further response is needed because this is normal. The rhythm should be regular, although sinus arrhythmia occurs normally in young adults and children. With sinus arrhythmia, the rhythm varies with the person's breathing, increasing at the peak of inspiration, and slowing with expiration.

Which of these individuals would the nurse consider at highest risk for a suicide attempt? a. Man who jokes about death b. Woman who, during a past episode of major depression, attempted suicide c. Adolescent who just broke up with her boyfriend and states that she would like to kill herself d. Older adult man who tells the nurse that he is going to join his wife in heaven tomorrow and plans to use a gun

ANS: Older adult man who tells the nurse that he is going to join his wife in heaven tomorrow and plans to use a gun When the person expresses feelings of sadness, hopelessness, despair, or grief, assessing any possible risk of physical harm to him or herself is important. The interview should begin with more general questions. If the nurse hears affirmative answers, then he or she should continue with more specific questions. A precise suicide plan to take place in the next 24 to 48 hours with use of a lethal method constitutes high risk.

The nurse is assessing for inflammation in a dark-skinned person. Which is the best technique? a. Assessing the skin for cyanosis and swelling b. Assessing the oral mucosa for generalized erythema c. Palpating the skin for edema and increased warmth d. Palpating for tenderness and local areas of ecchymosis

ANS: Palpate the skin for edema and increased warmth. Because you cannot see inflammation in dark-skinned persons, it is often necessary to palpate the skin for increased warmth, taut or tightly pulled surfaces that may be indicative of edema, and hardening of deep tissues or blood vessels.

A patient has been admitted to a hospital after the staff in the nursing home noticed a pressure ulcer in his sacral area. The nurse examines the pressure ulcer and determines that it is a stage II ulcer. Which of these findings are characteristic of a stage II pressure ulcer? Select all that apply. a. Intact skin appears red but is not broken. b. Partial thickness skin erosion is observed with a loss of epidermis or dermis. c. Ulcer extends into the subcutaneous tissue. d. Localized redness in light skin will blanch with fingertip pressure. e. Open blister areas have a red-pink wound bed. f. Patches of eschar cover parts of the wound.

ANS: Partial thickness skin erosion with loss of epidermis or dermis. Open blister areas have a red-pink wound bed. Stage I pressure ulcers have intact skin that appears red but not broken, and localized redness in intact skin will blanche with fingertip pressure. Stage II pressure ulcers have partial thickness skin erosion with loss of epidermis or also the dermis, and open blisters have a red-pink wound bed. Stage II pressure ulcers are full thickness, extending into the subcutaneous tissue; subcutaneous fat may be seen but not muscle, bone, or tendon. Stage IV pressure ulcers involve all skin layers and extend into supporting tissue, exposing muscle, bone, and tendon. Slough (stringy matter attached to the wound bed) or eschar (black or brown necrotic tissue) may be present.

A woman brings her husband to the clinic for an examination. She is particularly worried because after a recent fall, he seems to have lost a great deal of his memory of recent events. Which statement reflects the nurses best course of action? a. Perform a complete mental status examination. b. Refer him to a psychometrician. c. Plan to integrate the mental status examination into the history and physical examination. d. Reassure his wife that memory loss after a physical shock is normal and will soon subside.

ANS: Perform a complete mental status examination. Performing a complete mental status examination is necessary when any abnormality in affect or behavior is discovered or when family members are concerned about a persons behavioral changes (e.g., memory loss, inappropriate social interaction) or after trauma, such as a head injury.

A 23-year-old patient in the clinic appears anxious. Her speech is rapid, and she is fidgety and in constant motion. Which of these questions or statements would be most appropriate for the nurse to use in this situation to assess attention span? a. How do you usually feel? Is this normal behavior for you? b. I am going to say four words. In a few minutes, I will ask you to recall them. c. Describe the meaning of the phrase, Looking through rose-colored glasses. d. Pick up the pencil in your left hand, move it to your right hand, and place it on the table.

ANS: Pick up the pencil in your left hand, move it to your right hand, and place it on the table. Attention span is evaluated by assessing the individuals ability to concentrate and complete a thought or task without wandering. Giving a series of directions to follow is one method used to assess attention span.

A patient has been admitted for severe psoriasis. The nurse can expect to see what finding in the patient's fingernails? a. Splinter hemorrhages b. Paronychia c. Pitting d. Beau lines

ANS: Pitting Pitting nails are characterized by sharply defined pitting and crumbling of the nails with distal detachment, and they are associated with psoriasis. See Table 12-13 for descriptions of the other terms.

A patient is admitted to the unit after an automobile accident. The nurse begins the mental status examination and finds that the patient has dysarthric speech and is lethargic. The nurses best approach regarding this examination is to: a. Plan to defer the rest of the mental status examination. b. Skip the language portion of the examination, and proceed onto assessing mood and affect. c. Conduct an in-depth speech evaluation, and defer the mental status examination to another time. d. Proceed with the examination, and assess the patient for suicidal thoughts because dysarthria is often accompanied by severe depression.

ANS: Plan to defer the rest of the mental status examination. In the mental status examination, the sequence of steps forms a hierarchy in which the most basic functions (consciousness, language) are assessed first. The first steps must be accurately assessed to ensure validity of the steps that follow. For example, if consciousness is clouded, then the person cannot be expected to have full attention and to cooperate with new learning. If language is impaired, then a subsequent assessment of new learning or abstract reasoning (anything that requires language functioning) can give erroneous conclusions.

The nurse is planning health teaching for a 65-year-old woman who has had a cerebrovascular accident (stroke) and has aphasia. Which of these questions is most important to use when assessing mental status in this patient? a. Please count backward from 100 by seven. b. I will name three items and ask you to repeat them in a few minutes. c. Please point to articles in the room and parts of the body as I name them. d. What would you do if you found a stamped, addressed envelope on the sidewalk?

ANS: Please point to articles in the room and parts of the body as I name them. Additional tests for persons with aphasia include word comprehension (asking the individual to point to articles in the room or parts of the body), reading (asking the person to read available print), and writing (asking the person to make up and write a sentence).

The mother of a 3-month-old infant states that her baby has not been gaining weight. With further questioning, the nurse finds that the infant falls asleep after nursing and wakes up after a short amount of time, hungry again. What other information would the nurse want to have? a. Infant's sleeping position b. Sibling history of eating disorders c. Amount of background noise when eating d. Presence of dyspnea or diaphoresis when sucking

ANS: Presence of dyspnea or diaphoresis when sucking To screen for heart disease in an infant, focus on feeding. Note fatigue during feeding. An infant with heart failure takes fewer ounces each feeding, becomes dyspneic with sucking, may be diaphoretic and then falls into exhausted sleep and awakens after a short time hungry again.

A 45-year-old woman is at the clinic for a mental status assessment. In giving her the Four Unrelated Words Test, the nurse would be concerned if she could not ____ four unrelated words ____. a. Invent; within 5 minutes b. Invent; within 30 seconds c. Recall; after a 30-minute delay d. Recall; after a 60-minute delay

ANS: Recall; after a 30-minute delay The Four Unrelated Words Test tests the persons ability to lay down new memories. It is a highly sensitive and valid memory test. It requires more effort than the recall of personal or historic events. To the person say, I am going to say four words. I want you to remember them. In a few minutes I will ask you to recall them. After 5 minutes, ask for the four words. The normal response for persons under 60 years is an accurate three- or four-word recall after a 5-, 10-, and 30-minute delay.

The nurse is assessing bruising on an injured patient. Which color indicates a new bruise that is less than 2 hours old?

ANS: Red A new bruise is usually red and will often develop a purple or purple-blue appearance 12 to 36 hours after blunt-force trauma. The color of bruises (and ecchymoses) generally progresses from purple-blue to bluish-green to greenish-brown to brownish-yellow before fading away.

The direction of blood flow through the heart is best described by which of these? a. Vena cava right atrium right ventricle lungs pulmonary artery left atrium left ventricle b. Right atrium right ventricle pulmonary artery lungs pulmonary vein left atrium left ventricle c. Aorta right atrium right ventricle lungs pulmonary vein left atrium left ventricle vena cava d. Right atrium right ventricle pulmonary vein lungs pulmonary artery left atrium left ventricle

ANS: Right atrium right ventricle pulmonary artery lungs pulmonary vein left atrium left ventricle Returning blood from the body empties into the right atrium and flows into the right ventricle and then goes to the lungs through the pulmonary artery. The lungs oxygenate the blood and it is then returned to the left atrium by the pulmonary vein. It goes from there to the left ventricle and then out to the body through the aorta.

When assessing aging adults, the nurse knows that one of the first things that should be assessed before making judgments about their mental status is: a. Presence of phobias b. General intelligence c. Presence of irrational thinking patterns d. Sensory-perceptive abilities

ANS: Sensory-perceptive abilities Age-related changes in sensory perception can affect mental status. For example, vision loss (as detailed in Chapter 14) may result in apathy, social isolation, and depression. Hearing changes are common in older adults, which produces frustration, suspicion, and social isolation and makes the person appear confused.

The nurse has discovered decreased skin turgor in a patient and knows that this is an expected finding in which of these conditions? a. Severe obesity b. Childhood growth spurts c. Severe dehydration d. Connective tissue disorders such as scleroderma

ANS: Severe dehydration Decreased skin turgor is associated with severe dehydration or extreme weight loss.

The nurse is examining the glans and knows that which of these is a normal finding for this area?

ANS: Smegma may be present under the foreskin of an uncircumcised male. The glans looks smooth and without lesions and does not have hair. The meatus should not have any discharge when the glans is compressed. Some cheesy smegma may have collected under the foreskin of an uncircumcised male.

In assessing a patient's major risk factors for heart disease, which would the nurse want to include when taking a history? a. Family history, hypertension, stress, and age b. Personality type, high cholesterol, diabetes, and smoking c. Smoking, hypertension, obesity, diabetes, and high cholesterol d. Alcohol consumption, obesity, diabetes, stress, and high cholesterol

ANS: Smoking, hypertension, obesity, diabetes, high cholestero For major risk factors for coronary artery disease, collect data regarding elevated serum cholesterol, elevated blood pressure, blood glucose levels above 130 mg/dL or known diabetes mellitus, obesity, cigarette smoking, low activity level.

The nurse is examining a patient who has possible cardiac enlargement. Which statement about percussion of the heart is true? a. Percussion is a useful tool for outlining the heart's borders. b. Percussion is easier in patients who are obese. c. Studies show that percussed cardiac borders do not correlate well with the true cardiac border. d. Only expert health care providers should attempt percussion of the heart.

ANS: Studies show that percussed cardiac borders do not correlate well with the true cardiac border. Numerous comparison studies have shown that the percussed cardiac border correlates "only moderately" with the true cardiac border. Percussion is of limited usefulness with the female breast tissue or in an obese person, or a person with a muscular chest wall. Chest x-rays or echocardiogram examinations are much more accurate in detecting heart enlargement.

As a mandatory reporter of elder abuse, which of these must be present before a nurse notifies the authorities?

ANS: Suspicion of elder abuse and/or neglect Many health care workers are under the erroneous assumption that proof is required before notification of suspected abuse can occur. Only a suspicion of elder abuse or neglect is necessary.

A 16-year-old boy is brought to the clinic for a problem that he refused to let his mother see. The nurse examines him, and finds that he has scrotal swelling on the left side. He had the mumps the previous week, and the nurse suspects that he has orchitis. Which of the assessment findings below support this diagnosis? Select all that apply.

ANS: Swollen testis Mass does not transilluminate Tender upon palpation Scrotal skin is reddened With orchitis, the testis is swollen, with a feeling of weight, and is tender or painful. The mass does not transilluminate, and the scrotal skin is reddened. Transillumination of a mass occurs with a hydrocele, not orchitis.

During an examination, the nurse notes that a patient is exhibiting flight of ideas. Which statement by the patient is an example of flight of ideas? a. My stomach hurts. Hurts, spurts, burts. b. Kiss, wood, reading, ducks, onto, maybe. c. Take this pill? The pill is red. I see red. Red velvet is soft, soft as a baby's bottom. d. I wash my hands, wash them, wash them. I usually go to the sink and wash my hands.

ANS: Take this pill? The pill is red. I see red. Red velvet is soft, soft as a baby's bottom. Flight of ideas is demonstrated by an abrupt change, rapid skipping from topic to topic, and practically continuous flow of accelerated speech. Topics usually have recognizable associations or are plays on words.

During a mental status assessment, which question by the nurse would best assess a persons judgment? a. Do you feel that you are being watched, followed, or controlled? b. Tell me what you plan to do once you are discharged from the hospital. c. What does the statement, People in glass houses shouldn't throw stones, mean to you? d. What would you do if you found a stamped, addressed envelope lying on the sidewalk?

ANS: Tell me what you plan to do once you are discharged from the hospital. A person exercises judgment when he or she can compare and evaluate the alternatives in a situation and reach an appropriate course of action. Rather than testing the persons response to a hypothetical situation (as illustrated in the option with the envelope), the nurse should be more interested in the persons judgment about daily or long-term goals, the likelihood of acting in response to delusions or hallucinations, and the capacity for violent or suicidal behavior.

When the nurse is performing a testicular examination on a 25-year-old man, which of these findings is considered normal?

ANS: Testes that feel oval and movable and are slightly sensitive to compression Testes normally feel oval, firm and rubbery, smooth, and equal bilaterally and are freely movable and slightly tender to moderate pressure. The scrotal skin should not be dry, scaly, or nodular or contain subcutaneous plaques. Any mass would be an abnormal finding.

The mother of a 10-month-old infant tells the nurse that she has noticed that her son becomes blue when he is crying and that the frequency of this is increasing. He is also not crawling yet. During the examination the nurse palpates a thrill at the left lower sternal border and auscultates a loud systolic murmur in the same area. What would be the most likely cause of these findings? a. Tetralogy of Fallot b. Atrial septal defect c. Patent ductus arteriosus d. Ventricular septal defect

ANS: Tetralogy of Fallot Tetralogy of Fallot subjective findings include (1) severe cyanosis, not in the first months of life but developing as the infant grows, and right ventricle outflow (i.e., pulmonic) stenosis gets worse; (2) cyanosis with crying and exertion at first, then at rest; and (3) slowed development. Objective findings include (1) thrill palpable at left lower sternal border; (2) S1 normal, S2 has A2 loud and P2 diminished or absent; and (3) murmur is systolic, loud, crescendo-decrescendo.

The nurse is reviewing anatomy and physiology of the heart. Which statement best describes what is meant by atrial kick? a. The atria contract during systole and attempt to push against closed valves. b. Contraction of the atria at the beginning of diastole can be felt as a palpitation. c. Atrial kick is the pressure exerted against the atria as the ventricles contract during systole. d. The atria contract toward the end of diastole and push the remaining blood into the ventricles.

ANS: The atria contract toward the end of diastole and push the remaining blood into the ventricles. Toward the end of diastole, the atria contract and push the last amount of blood (about 25% of stroke volume) into the ventricles. This active filling phase is called presystole, or atrial systole, or sometimes the "atrial kick."

Which of these statements is true regarding the penis?

ANS: The corpus spongiosum expands into a cone of erectile tissue called the glans. At the distal end of the shaft, the corpus spongiosum expands into a cone of erectile tissue, the glans. The penis is composed of three cylindrical columns of erectile tissue. The prepuce is skin that covers the glans of the penis. The urethral meatus forms at the tip of the glans

Which of these statements about the testes is true?

ANS: The cremaster muscle contracts in response to cold and draws the testicles closer to the body. When it is cold, the cremaster muscle contracts, which raises the scrotal sac and brings the testes closer to the body to absorb heat necessary for sperm viability. The lymphatics of the testes drain into the inguinal lymph nodes. The vas deferens is located along the upper portion of each testis. The left testis is lower than the right because the left spermatic cord is longer.

The nurse is using the danger assessment (DA) tool to evaluate the risk of homicide. Which of these statements best describes its use?

ANS: The higher the number of yes answers, the more serious the danger of the woman's situation. There are no predetermined cutoff scores on the DA. The more yes answers there are, the more serious the danger of the woman's situation. The use of this tool is not limited to law enforcement personnel, and not required in every case of suspected abuse.

The nurse is assessing a patient with possible cardiomyopathy and assesses the hepatojugular reflux. If heart failure is present, then the nurse should see which finding while pushing on the right upper quadrant of the patient's abdomen, just below the rib cage? a. The jugular veins will rise for a few seconds and then recede back to the previous level if the heart is properly working. b. The jugular veins will remain elevated as long as pressure on the abdomen is maintained. c. An impulse will be visible at the fourth or fifth intercostal space at or inside the midclavicular line. d. The jugular veins will not be detected during this maneuver.

ANS: The jugular veins will remain elevated as long as pressure on the abdomen is maintained. When performing hepatojugular reflux, the jugular veins will rise for a few seconds and then recede back to the previous level if the heart is able to pump the additional volume created by the pushing; however, with heart failure, the jugular veins remain elevated as long as pressure on the abdomen is maintained.

The nurse is inspecting the scrotum and testes of a 43-year-old man. Which finding would require additional follow-up and evaluation?

ANS: The skin on the scrotum is taut. Scrotal swelling may cause the skin to be taut and to display pitting edema. Normal scrotal skin is rugae, and asymmetry is normal with the left scrotal half usually lower than the right. The testes may move closer to the body in response to cold temperatures.

Which of these statements describes the closure of the valves in a normal cardiac cycle? a. The aortic valve closes slightly before the tricuspid valve. b. The pulmonic valve closes slightly before the aortic valve. c. The tricuspid valve closes slightly later than the mitral valve. d. Both the tricuspid and pulmonic valves close at the same time.

ANS: The tricuspid valve closes slightly later than the mitral valve. Events occur just slightly later in the right side of the heart because of the route of myocardial depolarization. As a result, two distinct components to each of the heart sounds exist, and sometimes they can be heard separately. In the first heart sound, the mitral component (M1) closes just before the tricuspid component (T1).

When assessing a newborn infant who is 5 minutes old, the nurse knows that which of these statements would be true? a. The left ventricle is larger and weighs more than the right ventricle. b. The circulation of a newborn is identical to that of an adult. c. Blood can flow into the left side of the heart through an opening in the atrial septum. d. The foramen ovale closes just minutes before birth, and the ductus arteriosus closes immediately after.

ANS: There is an opening in the atrial septum where blood can flow into the left side of the heart. First, about two thirds of the blood is shunted through an opening in the atrial septum, the foramen ovale into the left side of the heart, where it is pumped out through the aorta. The foramen ovale closes within the first hour because the pressure in the right side of the heart is now lower than in the left side.

While auscultating heart sounds on a 7-year-old child for a routine physical, the nurse hears an S3, a soft murmur at left midsternal border, and a venous hum when the child is standing. Which of these would be a correct interpretation of these findings? a. S3 is indicative of heart disease in children. b. These findings can all be normal in a child. c. These findings are indicative of congenital problems. d. The venous hum most likely indicates an aneurysm.

ANS: These can all be normal findings in a child. Physiologic S3 is common in children. A venous hum, caused by turbulence of blood flow in the jugular venous system, is common in healthy children and has no pathologic significance. Heart murmurs that are innocent (or functional) in origin are very common through childhood.

The nurse is providing instructions to newly hired graduates for the minimental state examination (MMSE). Which statement best describes this examination? a. Scores below 30 indicate cognitive impairment. b. The MMSE is a good tool to evaluate mood and thought processes. c. This examination is a good tool to detect delirium and dementia and to differentiate these from psychiatric mental illness. d. The MMSE is useful tool for an initial evaluation of mental status. Additional tools are needed to evaluate cognition changes over time.

ANS: This examination is a good tool to detect delirium and dementia and to differentiate these from psychiatric mental illness. The MMSE is a quick, easy test of 11 questions and is used for initial and serial evaluations and can demonstrate a worsening or an improvement of cognition over time and with treatment. It evaluates cognitive functioning, not mood or thought processes. MMSE is a good screening tool to detect dementia and delirium and to differentiate these from psychiatric mental illness.

A 25-year-old woman in her fifth month of pregnancy has a blood pressure of 100/70 mm Hg. In reviewing her previous exam, the nurse notes that her blood pressure in her second month was 124/80 mm Hg. In evaluating this change, what does the nurse know to be true? a. This decline in blood pressure is the result of peripheral vasodilatation and is an expected change. b. Because of increased cardiac output, the blood pressure should be higher at this time. c. This change in blood pressure is not an expected finding because it means a decrease in cardiac output. d. This decline in blood pressure means a decrease in circulating blood volume, which is dangerous for the fetus.

ANS: This is the result of peripheral vasodilatation and is an expected change. Despite the increased cardiac output, arterial blood pressure decreases in pregnancy because of peripheral vasodilatation. The blood pressure drops to its lowest point during the second trimester and then rises after that.

During the cardiac auscultation the nurse hears a sound occurring immediately after S2 at the second left intercostal space. To further assess this sound, what should the nurse do? a. Have the patient turn to the left side while the nurse listens with the bell of the stethoscope. b. Ask the patient to hold his or her breath while the nurse listens again. c. No further assessment is needed because the nurse knows this sound is an S3. d. Watch the patient's respirations while listening for the effect on the sound.

ANS: Watch the patient's respirations while listening for effect on the sound. A split S2 is a normal phenomenon that occurs toward the end of inspiration in some people. A split S2 is heard only in the pulmonic valve area, the second left interspace. When the split S2 is first heard, the nurse should not be tempted to ask the person to hold his or her breath so that the nurse can concentrate on the sounds. Breath holding will only equalize ejection times in the right and left sides of the heart and cause the split to go away. Instead, the nurse should concentrate on the split while watching the person's chest rise up and down with breathing.

A patient has had a cerebrovascular accident (stroke). He is trying very hard to communicate. He seems driven to speak and says, I buy obie get spirding and take my train. What is the best description of this patients problem? a. Global aphasia b. Brocas aphasia c. Echolalia d. Wernickes aphasia

ANS: Wernickes aphasia This type of communication illustrates Wernickes or receptive aphasia. The person can hear sounds and words but cannot relate them to previous experiences. Speech is fluent, effortless, and well articulated, but it has many paraphasias (word substitutions that are malformed or wrong) and neologisms (made-up words) and often lacks substantive words. Speech can be totally incomprehensible. Often, a great urge to speak is present. Repetition, reading, and writing also are impaired. Echolalia is an imitation or the repetition of another persons words or phrases. (See Table 5-4 for the definitions of the other disorders.)

During an examination, the nurse notices a patterned injury on a patient's back. Which of these would cause such an injury?

ANS: Whipping from an extension cord A patterned injury is an injury caused by an object that leaves a distinct pattern on the skin or organ. The other actions do not cause a patterned injury.

A patient has been in the intensive care unit for 10 days. He has just been moved to the medical-surgical unit, and the admitting nurse is planning to perform a mental status examination. During the tests of cognitive function, the nurse would expect that he: a. May display some disruption in thought content. b. Will state, I am so relieved to be out of intensive care. c. Will be oriented to place and person, but the patient may not be certain of the date. d. May show evidence of some clouding of his level of consciousness.

ANS: Will be oriented to place and person, but the patient may not be certain of the date. The nurse can discern the orientation of cognitive function through the course of the interview or can directly and tactfully ask, Some people have trouble keeping up with the dates while in the hospital. Do you know today's date? Many hospitalized people have trouble with the exact date but are fully oriented on the remaining items.

The nurse is assessing a patient who has liver disease for jaundice. Which of these assessment findings is indicative of true jaundice? a. Yellow patches in the outer sclera b. Yellow color of the sclera that extends up to the iris c. Skin that appears yellow when examined under low light d. Yellow deposits on the palms and soles of the feet where jaundice first appears

ANS: Yellow color of the sclera that extends up to the iris The yellow sclera of jaundice extends up to the edge of the iris. Calluses on the palms and soles of the feet often look yellow but are not classified as jaundice. Do not confuse scleral jaundice with the normal yellow subconjunctival fatty deposits that are common in the outer sclera of dark-skinned persons.

During an examination of an aging male, the nurse recognizes that normal changes to expect would be:

ANS: a decrease in the size of the penis. When assessing the genitals of an older man, the nurse may notice thinner, graying pubic hair and a decrease in the size of the penis. The size of the testes may be decreased, they may feel less firm, and the scrotal sac is pendulous with less rugae. There is no change in scrotal color.

During an examination, the nurse notices that a male patient has a red, round, superficial ulcer with a yellowish serous discharge on his penis. On palpation, the nurse finds a nontender base that feels like a small button between the thumb and fingers. At this point the nurse suspects that this patient has:

ANS: a syphilitic chancre. This lesion indicates syphilitic chancre, which begins within 2 to 4 weeks of infection. See Table 24-4 for descriptions of the other options.

During a cardiovascular assessment, the nurse knows that a "thrill" is: a. Vibration that is palpable. b. Palpated in the right epigastric area. c. Associated with ventricular hypertrophy. d. Murmur auscultated at the third intercostal space.

ANS: a vibration that is palpable. A thrill is a palpable vibration. It signifies turbulent blood flow and accompanies loud murmurs. The absence of a thrill does not rule out the presence of a murmur.

A father brings in his 2-month-old infant to the clinic because the infant has had diarrhea for the last 24 hours. He says that his baby has not been able to keep any formula down and that the diarrhea has been at least every 2 hours. The nurse suspects dehydration. The nurse should test skin mobility and turgor in this infant over the: a. Sternum. b. Forehead. c. Forearms. d. Abdomen.

ANS: abdomen. Test mobility and turgor over the abdomen in an infant. Poor turgor, or "tenting," indicates dehydration or malnutrition. The other areas are not appropriate sites for checking skin turgor in an infant.

A 2-year-old boy has been diagnosed with "physiologic cryptorchidism." Given this diagnosis, during assessment the nurse will most likely observe:

ANS: an absence of the testis in the scrotum, but the testis can be milked down. Migratory testes (physiologic cryptorchidism) are common because of the strength of the cremasteric reflex and the small mass of the prepubertal testes. The affected side has a normally developed scrotum and the testis can be milked down. The other responses are not correct.

The nurse is assessing for clubbing of the fingernails and would expect to find: a. Nail bases that are firm and slightly tender. b. Curved nails with a convex profile and ridges across the nails. c. Nail bases that feel spongy with an angle of the nail base of 150 degrees. d. Nail bases with an angle of 180 degrees or greater and nail bases that feel spongy.

ANS: an angle of the nail base of 180 degrees or greater with a nail base that feels spongy. The normal nail is firm at its base and has an angle of 160 degrees. In clubbing, the angle straightens to 180 degrees or greater and the nail base feels spongy.

When listening to heart sounds, the nurse knows that the valve closures that can be heard best at the base of the heart are: a. Mitral and tricuspid. b. Tricuspid and aortic. c. Aortic and pulmonic. d. Mitral and pulmonic.

ANS: aortic and pulmonic. The second heart sound (S2) occurs with closure of the semilunar (aortic and pulmonic) valves and signals the end of systole. Although it is heard over all the precordium, S2 is loudest at the base of the heart.

A 70-year-old patient with a history of hypertension has a blood pressure of 180/100 mm Hg and a heart rate of 90 beats per minute. The nurse hears an extra heart sound at the apex immediately before S1. The sound is heard only with the bell while the patient is in the left lateral position. With these findings and the patient's history, the nurse knows that this extra heart sound is most likely a(n): a. Split S1. b. Atrial gallop. c. Diastolic murmur. d. Summation sound.

ANS: atrial gallop. A pathologic S4 is termed an atrial gallop or an S4 gallop. It occurs with decreased compliance of the ventricle and with systolic overload (afterload), including outflow obstruction to the ventricle (aortic stenosis) and systemic hypertension. A left-sided S4 occurs with these conditions. It is heard best at the apex with the patient in the left lateral position.

In assessing for an S4 heart sound with a stethoscope, the nurse would listen with the: a. Bell of the stethoscope at the base with the patient leaning forward. b. Bell of the stethoscope at the apex with the patient in the left lateral position. c. Diaphragm of the stethoscope in the aortic area with the patient sitting. d. Diaphragm of the stethoscope in the pulmonic area with the patient supine.

ANS: bell at the apex with the patient in the left lateral position. The S4 is a ventricular filling sound. It occurs when atria contract late in diastole. It is heard immediately before S1. This is a very soft sound with a very low pitch. The nurse needs a good bell and must listen for it. It is heard best at the apex, with the person in the left lateral position.

During an assessment of a 68-year-old man with a recent onset of right-sided weakness, the nurse hears a blowing, swishing sound with the bell of the stethoscope over the left carotid artery. This finding would indicate: a. Valvular disorder. b. Blood flow turbulence. c. Fluid volume overload. d. Ventricular hypertrophy.

ANS: blood flow turbulence. A bruit is a blowing, swishing sound indicating blood flow turbulence; normally none is present.

When performing a genitourinary assessment, the nurse notices that the urethral meatus is positioned ventrally. This finding is:

ANS: called hypospadias. Normally the urethral meatus is positioned just about centrally. Hypospadias is the ventral location of the urethral meatus. The position of the meatus does not change with aging. Phimosis is the inability to retract the foreskin. A stricture is a narrow opening of the meatus.

A semiconscious woman is brought to the emergency department after she was found on the floor in her kitchen. Her face, nail beds, lips, and oral mucosa are a bright cherry-red color. The nurse suspects that this coloring is due to: a. Polycythemia. b. Carbon monoxide poisoning. c. Carotenemia. d. Uremia.

ANS: carbon monoxide poisoning. A bright cherry-red coloring in the face, upper torso, nail beds, lips, and oral mucosa appears in cases of carbon monoxide poisoning.

When listening to heart sounds, the nurse knows that S1: a. Is louder than the S2 at the base of the heart. b. Indicates the beginning of diastole. c. Coincides with the carotid artery pulse. d. Is caused by the closure of the semilunar valves.

ANS: coincides with the carotid artery pulse. S1 coincides with the carotid artery pulse. S1 is the start of systole and is louder than S2 at the apex of the heart; S2 is louder than S1 at the base. The nurse should feel the carotid artery pulse gently while auscultating at the apex; the sound heard as each pulse is felt is S1.

The nurse is performing a genital examination on a male patient and notices urethral drainage. When collecting urethral discharge for microscopic examination and culture, the nurse should:

ANS: compress the glans between the examiner's thumb and forefinger and collect any discharge. If urethral discharge is noticed, then the examiner should collect a smear for microscopic examination and culture by compressing the glans anteroposteriorly between the thumb and forefinger. The other options are not correct actions.

When performing a genital examination on a 25-year-old man, the nurse notices deeply pigmented, wrinkled scrotal skin with large sebaceous follicles. On the basis of this information the nurse would:

ANS: consider this a normal finding and proceed with the examination. After adolescence, the scrotal skin is deeply pigmented and has large sebaceous follicles. The scrotal skin looks corrugated

A newborn baby boy is about to have a circumcision. The nurse knows that indications for circumcision include:

ANS: cultural and religious beliefs. Indications for circumcision include cultural and religious beliefs, prevention of phimosis and inflammation of the glans penis and foreskin, decreasing the incidence of cancer of the penis, and decreasing the incidence of urinary tract infections in infancy.

During an examination of an aging male, the nurse recognizes that normal changes to expect would be:

ANS: decreased penis size. In the aging male the amount of pubic hair decreases, the penis size decreases, and there is a decrease in the rugae over the scrotal sac. The scrotal sac does not enlarge.

The nurse is performing a genitourinary assessment on a 50-year-old obese male laborer. On examination the nurse notices a painless round swelling close to the pubis in the area of the internal inguinal ring that is easily reduced when the individual is supine. These findings are most consistent with a(n) _____ hernia.

ANS: direct inguinal Direct inguinal hernias occur most often in men over the age of 40 years. It is an acquired weakness brought on by heavy lifting, obesity, chronic cough, or ascites. The direct inguinal hernia is usually a painless, round swelling close to the pubis in the area of the internal inguinal ring that is easily reduced when the individual is supine. See Table 24-6 for a description of scrotal hernia. See Table 24-7 for descriptions of femoral hernias and indirect inguinal hernias.

During the precordial assessment on an patient who is 8 months pregnant, the nurse palpates the apical impulse at the fourth left intercostal space lateral to the midclavicular line. This finding would indicate: a. Right ventricular hypertrophy. b. Increased volume and size of the heart as a result of pregnancy. c. Displacement of the heart from elevation of the diaphragm. d. Increased blood flow through the internal mammary artery.

ANS: displacement of the heart from elevation of the diaphragm. Palpation of the apical impulse is higher and more lateral compared with the normal position because the enlarging uterus elevates the diaphragm and displaces the heart up and to the left and rotates it on its long axis.

A 59-year-old patient has been diagnosed with prostatitis and is being seen at the clinic for complaints of burning and pain during urination. He is experiencing:

ANS: dysuria. Dysuria or burning with urination is common with acute cystitis, prostatitis, and urethritis. Nocturia is voiding during the night. Polyuria is voiding in excessive quantities. Hematuria is voiding with blood in the urine.

The findings from an assessment of a 70-year-old patient with swelling in his ankles include jugular venous pulsations 5 cm above the sternal angle when the head of his bed is elevated 45 degrees. The nurse knows that this finding indicates: a. Decreased fluid volume. b. Increased cardiac output. c. Narrowing of jugular veins. d. Elevated pressure related to heart failure.

ANS: elevated pressure related to heart failure. Because no cardiac valve exists to separate the superior vena cava from the right atrium, the jugular veins give information about activity on the right side of the heart. They reflect filling pressures and volume changes. Normal jugular venous pulsation is 2 cm or less above the sternal angle. Elevated pressure is more than 3 cm above the sternal angle at 45 degrees and occurs with heart failure.

During inspection of the precordium of an adult patient, the nurse notices the chest moving in a forceful manner along the sternal border. This finding most likely suggests: a. Normal heart. b. Systolic murmur. c. Enlargement of the left ventricle. d. Enlargement of the right ventricle.

ANS: enlargement of the right ventricle. Normally, the examiner may or may not see an apical impulse; when visible, it occupies the fourth or fifth intercostal space at or inside the midclavicular line. A heave or lift is a sustained forceful thrusting of the ventricle during systole. It occurs with ventricular hypertrophy as a result of increased workload. A right ventricular heave is seen at the sternal border; a left ventricular heave is seen at the apex.

A 62-year-old man states that his doctor told him that he has an "inguinal hernia." He asks the nurse to explain what a hernia is. The nurse should:

ANS: explain that a hernia is a loop of bowel protruding through a weak spot in the abdominal muscles. A hernia is a loop of bowel protruding through a weak spot in the musculature. The other options are not correct responses to the patient's question.

During a home visit, the nurse notices that an elderly woman has very little food in her cabinets or refrigerator and that most of her prescription bottles are empty. She says that she has enough money, but her nephew has her checkbook and he "takes care of everything." She says, "Oh, my nephew will get around to getting groceries and my medicine when he can. He's very busy." This is an example of:

ANS: financial neglect. Financial neglect is failure to use the assets of the elderly person to provide services needed by the elderly person. Financial abuse is intentional misuse of the elderly person's financial or material resources without the informed consent of the person; there is no evidence of intent in this example.

During a genital examination, the nurse notices that a male patient has clusters of small vesicles on the glans, surrounded by erythema. The nurse recognizes that these lesions are:

ANS: genital herpes. Genital herpes, or HSV-2, infections are indicated with clusters of small vesicles with surrounding erythema, which are often painful and erupt on the glans or foreskin. See Table 24-4 for descriptions of the other options.

When performing a genital assessment on a middle-aged man, the nurse notices multiple soft, moist, painless papules in the shape of cauliflower-like patches scattered across the shaft of the penis. These lesions are characteristic of:

ANS: genital warts. The lesions of genital warts are soft, pointed, moist, fleshy, painless papules that may be single or multiple in a cauliflower-like patch. They occur on the shaft of the penis, behind the corona, or around the anus, where they may grow into large grape-like clusters. See Table 24-4 for more information and for descriptions of the other options.

The nurse is caring for a black child who has been diagnosed with marasmus. The nurse would expect to find the:

ANS: hair to be less kinky and to be a copper-red color. The hair of black children with severe malnutrition (e.g., marasmus) frequently changes not only in texture but in color—the child's hair becomes less kinky and assumes a copper-red color. The other findings are not present with marasmus.

During a cardiovascular assessment, the nurse knows that an S4 heart sound is: a. Heard at the onset of atrial diastole. b. Usually a normal finding in the older adult. c. Heard at the end of ventricular diastole. d. Heard best over the second left intercostal space with the individual sitting upright.

ANS: heard at the end of ventricular diastole. An S4 heart sound is heard at the end of diastole when the atria contract (atrial systole) and when the ventricles are resistant to filling. The S4 occurs just before the S1.

While performing an assessment of a 65-year-old man with a history of hypertension and coronary artery disease, the nurse notices the presence of pitting edema in the lower legs bilaterally. The skin is puffy and tight but of normal color. There is no increased redness or tenderness over his lower legs, and the peripheral pulses are equal and strong. In this situation, the nurse suspects that the likely cause of the edema would be: a. Heart failure b. Venous thrombosis c. Local inflammation d. Blockage of lymphatic drainage

ANS: heart failure. Bilateral edema or edema that is generalized over the entire body is caused by a central problem such as heart failure or kidney failure. Unilateral edema usually has a local or peripheral cause.

During a health history, a patient tells the nurse that he has trouble in starting his urine stream. This problem is known as:

ANS: hesitancy. Hesitancy is trouble in starting the urine stream. Urgency is the feeling that one cannot wait to urinate. Dribbling is the lost of urine before or after the main act of urination. Frequency is urinating more often than usual.

A patient's mother has noticed that her son, who has been to a new babysitter, has some blisters and scabs on his face and buttocks. On examination, the nurse notices moist, thin-roofed vesicles with a thin erythematous base and suspects: a. Eczema. b. Impetigo. c. Herpes zoster. d. Diaper dermatitis.

ANS: impetigo. Impetigo is moist, thin-roofed vesicles with a thin erythematous base. This is a contagious bacterial infection of the skin and most common in infants and children. Eczema is characterized by erythematous papules and vesicles with weeping, oozing, and crusts. Herpes zoster (chicken pox or varicella) is characterized by small, tight vesicles that are shiny with an erythematous base. Diaper dermatitis is characterized by red, moist maculopapular patches with poorly defined borders.

When performing a genitourinary assessment on a 16-year-old boy, the nurse notices a swelling in the scrotum that increases with increased intra-abdominal pressure and decreases when he is lying down. The patient complains of pain when straining. The nurse knows that this description is most consistent with a(n) _____ hernia.

ANS: indirect inguinal With indirect inguinal hernias there is pain with straining and a soft swelling that increases with increased intra-abdominal pressure, which may decrease when the patient lies down. These findings do not describe the other hernias. See Table 24-7 for descriptions of femoral and direct inguinal hernias.

The nurse is performing a cardiac assessment on a 65-year-old patient 3 days after her myocardial infarction. Heart sounds are normal when she is supine, but when she is sitting and leaning forward, the nurse hears a high-pitched, scratchy sound with the diaphragm of the stethoscope at the apex. It disappears on inspiration. The nurse suspects: a. Increased cardiac output. b. Another MI. c. Inflammation of the precordium. d. Ventricular hypertrophy resulting from muscle damage.

ANS: inflammation of the precordium. Inflammation of the precordium gives rise to a friction rub. The sound is high pitched and scratchy, like sandpaper being rubbed. It is best heard with the diaphragm of the stethoscope, with the person sitting up and leaning forward, and with the breath held in expiration. A friction rub can be heard any place on the precordium but usually is best heard at the apex and left lower sternal border, which are places where the pericardium comes in close contact with the chest wall.

The nurse knows that normal splitting of the second heart sound is associated with: a. Expiration. b. Inspiration. c. Exercise state. d. Low resting heart rate.

ANS: inspiration. Normal or physiologic splitting of the second heart sound is associated with inspiration because of the increased blood return to the right side of the heart, delaying closure of the pulmonic valve.

The nurse just noted from a patient's medical record that the patient has a lesion that is confluent in nature. On examination, the nurse would expect to find: a. Lesions that run together. b. Annular lesions that have grown together. c. Lesions arranged in a line along a nerve route. d. Lesions that are grouped or clustered together.

ANS: lesions that run together. Confluent lesions (as with urticaria [hives]) run together. Grouped lesions are clustered together. Annular lesions are circular in nature. Zosteriform lesions are arranged along a nerve route.

A patient has had a "terrible itch" for several months that he has been scratching continuously. On examination, the nurse might expect to find: a. A keloid. b. A fissure. c. Keratosis. d. Lichenification.

ANS: lichenification. Lichenification results from prolonged, intense scratching that eventually thickens the skin and produces tightly packed sets of papules. A keloid is a hypertrophic scar. A fissure is a linear crack with abrupt edges that extends into the dermis, and it can be dry or moist. Keratoses are lesions that are raised, thickened areas of pigmentation that look crusted, scaly, and warty.

In assessing the carotid arteries of an older patient with cardiovascular disease, the nurse would: a. Palpate the artery in the upper one third of the neck. b. Listen with the bell of the stethoscope to assess for bruits. c. Simultaneously palpate both arteries to compare amplitude. d. Instruct the patient to take slow deep breaths during auscultation.

ANS: listen with the bell of the stethoscope to assess for bruits. If cardiovascular disease is suspected, then the nurse should auscultate each carotid artery for the presence of a bruit. The nurse should avoid compressing the artery because this could create an artificial bruit, and it could compromise circulation if the carotid artery is already narrowed by atherosclerosis. Avoid excessive pressure on the carotid sinus area higher in the neck; excessive vagal stimulation here could slow down the heart rate, especially in older adults. Palpate only one carotid artery at a time to avoid compromising arterial blood to the brain.

A 30-year-old woman with a history of mitral valve problems states that she has been "very tired." She has started waking up at night and feels like her "heart is pounding." During the assessment, the nurse palpates a thrill and lift at the fifth left intercostal space midclavicular line. In the same area the nurse also auscultates a blowing, swishing sound right after S1. These findings would be most consistent with: a. Heart failure. b. Aortic stenosis. c. Pulmonary edema. d. Mitral regurgitation.

ANS: mitral regurgitation. Mitral regurgitation subjective findings include fatigue, palpitation, and orthopnea. Objective findings are (1) a thrill in systole at apex, (2) lift at apex, (3) apical impulse displaced down and to the left, (4) S1 diminished, S2 accentuated, S3 at apex often present, and (5) murmur: pansystolic, often loud, blowing, best heard at apex, radiating well to the left axilla.

The nurse is assessing the apical pulse of a 3-month-old infant and finds that the heart rate is 135 beats per minute. The nurse interprets this result as: a. Normal for this age. b. Lower than expected. c. Higher than expected, probably as a result of crying. d. Higher than expected, reflecting persistent tachycardia.

ANS: normal for this age. The heart rate may range from 100 to 180 beats per minute immediately after birth and then stabilize to an average of 120 to 140 beats per minute. Infants normally have wide fluctuations with activity, from 170 beats per minute or more with crying or being active to 70 to 90 beats per minute with sleeping. Persistent tachycardia is greater than 200 beats per minute in newborns or greater than 150 beats per minute in infants.

The nurse notices that a patient has a solid, elevated, circumscribed lesion that is less than 1 cm in diameter. When documenting this finding, the nurse would report this as a: a. Bulla. b. Wheal. c. Nodule. d. Papule.

ANS: papule. A papule is something one can feel, is solid, elevated, circumscribed, less than 1 cm in diameter, and is due to superficial thickening in the epidermis. A bulla is larger than 1 cm, superficial, and thin walled. A wheal is superficial, raised, transient, erythematous, and irregular in shape due to edema. A nodule is solid, elevated, hard or soft, and larger than 1 cm.

The sac that surrounds and protects the heart is called the: a. Pericardium. b. Myocardium. c. Endocardium. d. Pleural space.

ANS: pericardium. The pericardium is a tough fibrous double-walled sac that surrounds and protects the heart. It has two layers that contain a few milliliters of serous pericardial fluid.

During a physical examination, the nurse finds that a male patient's foreskin is fixed and tight and will not retract over the glans. The nurse recognizes that this condition is:

ANS: phimosis. With phimosis, the foreskin is nonretractable, forming a pointy tip of the penis with a tiny orifice at the end of the glans. The foreskin is advanced and so tight that it is impossible to retract over the glans. This may be congenital or acquired from adhesions related to infection. See Table 24-3 for information on urethral stricture. See Table 24-4 for information on epispadias and Peyronie's disease.

When documenting intimate partner violence and elder abuse, the nurse should include:

ANS: photographic documentation of injuries. Documentation of intimate partner violence and elder abuse must include detailed nonbiased progress notes, the use of injury maps, and photographic documentation. Written documentation needs to be verbatim, within reason. Not every statement can be documented.

The nurse is providing patient teaching about an erectile dysfunction drug. One of the drug's potential side effects is prolonged, painful erection of the penis without sexual stimulation, which is known as:

ANS: priapism. Priapism is prolonged, painful erection of the penis without sexual desire. Orchitis is inflammation of the testes. Stricture is a narrowing of the opening of the urethral meatus. Phimosis is the inability to retract the foreskin.

An accessory glandular structure for the male genital organs is the:

ANS: prostate. Glandular structures accessory to the male genital organs are the prostate, seminal vesicles, and bulbourethral glands

When the nurse is performing a genital examination on a male patient, the patient has an erection. The nurse's most appropriate action or response is to:

ANS: reassure the patient that this is a normal response and continue with the examination. When the male patient has an erection, the nurse should reassure the patient that this is a normal physiologic response to touch and proceed with the rest of the examination. The other responses are not correct and may be perceived as judgmental.

A 40-year-old woman reports a change in mole size, accompanied by color changes, itching, burning, and bleeding over the past month. She has a dark complexion and has no family history of skin cancer, but she has had many blistering sunburns in the past. The nurse would: a. Tell the patient to watch the lesion and report back in 2 months. b. Refer the patient because of the suggestion of melanoma on the basis of her symptoms. c. Ask additional questions regarding environmental irritants that may have caused this condition. d. Tell the patient that these signs suggest a compound nevus, which is very common in young to middle-aged adults.

ANS: refer the patient because of the suspicion of melanoma on the basis of her symptoms. The ABCD danger signs of melanoma are asymmetry, border irregularity, color variation, and diameter. In addition, individuals may report a change in size, development of itching, burning, bleeding, or a new-pigmented lesion. Any of these signs raise suspicion of malignant melanoma and warrant immediate referral.

The external male genital structures include the:

ANS: scrotum. The external male genital structures include the penis and scrotum. The testis, epididymis, and vas deferens are internal structures.

When assessing the scrotum of a male patient, the nurse notices the presence of multiple firm, nontender, yellow 1-cm nodules. The nurse knows that these nodules are most likely:

ANS: sebaceous cysts. Sebaceous cysts are commonly found on the scrotum. These are yellowish 1-cm nodules and are firm, nontender, and often multiple. The other options are not correct.

An older man is concerned about his sexual performance. The nurse knows that in the absence of disease, a withdrawal from sexual activity later in life may be due to:

ANS: side effects of medications. In the absence of disease, a withdrawal from sexual activity may be due to side effects of medications such as antihypertensives, antidepressants, or sedatives. The other options are not correct.

The component of the conduction system referred to as the pacemaker of the heart is the: a. Atrioventricular (AV) node. b. Sinoatrial (SA) node. c. Bundle of His. d. Bundle branches.

ANS: sinoatrial (SA) node. Specialized cells in the SA node near the superior vena cava initiate an electrical impulse. Because the SA node has an intrinsic rhythm, it is the "pacemaker."

A 45-year-old mother of two children is seen at the clinic for complaints of "losing my urine when I sneeze." The nurse documents that she is experiencing:

ANS: stress incontinence. Stress incontinence is involuntary urine loss with physical strain, sneezing, or coughing that occurs due to weakness of the pelvic floor. Urinary frequency is urinating more times than usual (more than 5 to 6 times per day). Enuresis is involuntary passage of urine at night after age 5 to 6 years (bed wetting). Urge incontinence is involuntary urine loss from overactive detrusor muscle in the bladder. It contracts, causing an urgent need to void.

When performing a scrotal assessment, the nurse notices that the scrotal contents transilluminate and show a red glow. On the basis of this finding the nurse would:

ANS: suspect the presence of serous fluid in the scrotum. Normal scrotal contents do not transilluminate. Serous fluid does transilluminate and shows as a red glow. Neither a mass nor a hernia would transilluminate.

A male patient with possible fertility problems asks the nurse where sperm is produced. The nurse knows that sperm production occurs in the:

ANS: testes. Sperm production occurs in the testes, not in the other structures listed.

The nurse knows that a common assessment finding in a boy younger than 2 years old is:

ANS: the presence of a hydrocele, or fluid in the scrotum. A common scrotal finding in boys younger than 2 years of age is a hydrocele, or fluid in the scrotum. The other options are not correct.

A 45-year-old farmer comes in for a skin evaluation and complains of hair loss on his head. He has noticed that his hair seems to be breaking off in patches and that he has some scaling on his head. The nurse would begin the examination suspecting: a. Tinea capitis. b. Folliculitis. c. Toxic alopecia. d. Seborrheic dermatitis.

ANS: tinea capitis. Tinea capitis is rounded patchy hair loss on the scalp, leaving broken-off hairs, pustules, and scales on the skin. It is caused by a fungal infection. Lesions are fluorescent under a Wood light. It is usually seen in children and farmers and is highly contagious. See Table 12-12, Abnormal Conditions of Hair, for descriptions of the other terms.

A mother brings her child in to the clinic for an examination of the scalp and hair. She states that the child has developed some places where there are irregularly shaped patches with broken-off, stub-like hair and she is worried that this could be some form of premature baldness. The nurse tells her that it is: a. Folliculitis that can be treated with an antibiotic. b. Traumatic alopecia that can be treated with antifungal medications. c. Tinea capitis that is highly contagious and needs immediate attention. d. Trichotillomania; her child probably has a habit of absentmindedly twirling her hair

ANS: trichotillomania and that her child probably has a habit of twirling her hair absentmindedly. Trichotillomania, self-induced hair loss, is usually due to habit. It forms irregularly-shaped patches with broken-off, stub-like hairs of varying lengths. A person is never completely bald. It occurs as a child rubs or twirls the area absently while falling asleep, reading, or watching television. See Table 12-12, Abnormal Conditions of Hair, for descriptions of the other terms.

During a home visit, the nurse notices that an elderly woman is caring for her bedridden husband. The woman states that this is her duty, she does the best she can, and her children come to help when they are in town. Her husband is unable to care for himself, and she appears thin, weak, and exhausted. The nurse notices that several of his prescription medication bottles are empty. This situation is best described by the term:

ANS: unintentional physical neglect. Unintentional physical neglect may occur despite good intentions. It is the failure of the family member or caregiver to provide basic goods or services. Physical abuse is defined as violent acts that result or could result in injury, pain, impairment, or disease. Financial neglect is failure to use the assets of the elderly person to provide services needed by the elderly person. Psychological abuse is defined as behaviors that result in mental anguish.

The nurse suspects abuse when a 10-year-old child is taken to the urgent care center for a leg injury. The best way to document the history and physical findings is to:

ANS: use the words the child has given to describe how the injury occurred. When documenting the history and physical findings of child abuse and neglect, use the words the child has given to describe how his or her injury occurred. Remember that the abuser may be accompanying the child.

A 15-year-old boy is seen in the clinic for complaints of "dull pain and pulling" in the scrotal area. On examination the nurse palpates a soft, irregular mass posterior to and above the testis on the left. This mass collapses when the patient is supine and refills when he is upright. This description is consistent with:

ANS: varicocele. A varicocele consists of dilated, tortuous varicose veins in the spermatic cord caused by incompetent valves within the vein. Symptoms include dull pain or constant pulling or dragging feeling, or the individual may be asymptomatic. When palpating the mass, the examiner will feel a soft, irregular mass posterior to and above the testis that collapses when the individual is supine and refills when the individual is upright. See Table 24-6 for more information and for descriptions of the other options.

The nurse is assessing the joints of a woman who has stated," I have a long family history of arthritis, and my joints hurt." THe nurse suspects that she has osteoarthritis. Which of these are symptoms of osteoarthritis?

Asymmetric joint involvement Pain with motion of affected joints Affected joints are swollen with hard, Bony protuberances.

1. The nurse is preparing to use the Lawton IADL instrument as part of an assessment. Which statement about the Lawton IADL instrument is true? a. The nurse uses direct observation to implement this tool. b. The Lawton IADL instrument is designed as a self-report measure of performance rather than ability. c. This instrument is not useful in the acute hospital setting. d. This tool is best used for those residing in an institutional setting.

B

A 21-year-old woman has been on a low-protein liquid diet for the past 2 months. She has had adequate calories and appears well nourished. In further assessing her, what would the nurse expect to find? A) Poor skin turgor B) Decreased serum albumin C) Increased lymphocyte count D) Triceps skinfold less than standard

B

A 35-year-old man is seen in the clinic for an infection in his left foot. Which of these findings should the nurse expect to see during an assessment of this patient? a. Hard and fixed cervical nodes b. Enlarged and tender inguinal nodes c. Bilateral enlargement of the popliteal nodes d. Pelletlike nodes in the supraclavicular region

B

A 4-month-old child is at the clinic for a well-baby check-up and immunizations. Which of these actions is most appropriate when the nurse is assessing an infant's vital signs? A) Palpate the infant's radial pulse and notice any fluctuations resulting from activity or exercise. B) Auscultate an apical rate for 1 minute and assess for any normal irregularities, such as sinus arrhythmia. C) Assess the infant's blood pressure by using a stethoscope with a large diaphragm piece to hear the soft muffled Korotkoff sounds. D) Watch the infant's chest and count the respiratory rate for 1 minute because the respiratory pattern may vary significantly.

B

A 70-year-old man has a blood pressure of 150/90 mm Hg in a lying position, 130/80 mm Hg in a sitting position, and 100/60 mm Hg in a standing position. How should the nurse evaluate these findings? A) This is a normal response due to changes in the patient's position. B) The change in blood pressure readings is called orthostatic hypotension. C) The blood pressure reading in the lying position is within normal limits. D) The change in blood pressure reading is considered within normal limits for the patient's age.

B

A female patient tells the nurse that she has had six pregnancies, with four live births at term and two spontaneous abortions. Her four children are still living. How would the nurse record this information? a.P-6, B-4, (S)Ab-2 b.Grav 6, Term 4, (S)Ab-2, Living 4 c.Patient has had four living babies. d.Patient has been pregnant six times.

B

A nurse is helping at a health fair at a local mall. When taking blood pressures on a variety of people, the nurse keeps in mind: A) after menopause, blood pressure in women is usually lower than in men. B) a black adult's blood pressure is usually higher than that of whites of the same age. C) blood pressure measurements in people who are overweight should be the same as those of people who are at a normal weight. D) a teen's blood pressure reading will be lower than that of an adult.

B

A patient complains of leg pain that wakes him at night. He states that he "has been having problems" with his legs. He has pain in his legs when they are elevated that disappears when he dangles them. He recently noticed "a sore" on the inner aspect of the right ankle. On the basis of this health history information, the nurse interprets that the patient is most likely experiencing: a. Pain related to lymphatic abnormalities. b. Problems related to arterial insufficiency. c. Problems related to venous insufficiency. d. Pain related to musculoskeletal abnormalities.

B

A patient has suddenly developed shortness of breath and appears to be in significant respiratory distress. After calling the physician and placing the patient on oxygen, which of these actions is the best for the nurse to take when further assessing the patient? a. Count the patients respirations. b. Bilaterally percuss the thorax, noting any differences in percussion tones. c. Call for a chest x-ray study, and wait for the results before beginning an assessment. d. Inspect the thorax for any new masses and bleeding associated with respirations.

B

A patient is brought by ambulance to the emergency department with multiple traumas received in an automobile accident. He is alert and cooperative, but his injuries are quite severe. How would the nurse proceed with data collection? a. Collect history information first, then perform the physical examination and institute life-saving measures. b. Simultaneously ask history questions while performing the examination and initiating life-saving measures. c. Collect all information on the history form, including social support patterns, strengths, and coping patterns. d. Perform life-saving measures and delay asking any history questions until the patient is transferred to the intensive care unit.

B

A patient tells the nurse that his food simply does not have any taste anymore. The nurse's best response would be: a. "That must be really frustrating." b. "When did you first notice this change?" c. "My food doesn't always have a lot of taste either." d. "Sometimes that happens, but your taste will come back."

B

An individual who takes the magicoreligious perspective of illness and disease is likely to believe that his or her illness was caused by: a. Germs and viruses. b. Supernatural forces. c. Eating imbalanced foods. d. An imbalance within his or her spiritual nature.

B

As part of the health history of a 6-year-old boy at a clinic for a sports physical examination, the nurse reviews his immunization record and notes that his last measles-mumps-rubella (MMR) vaccination was at 15 months of age. What recommendation should the nurse make? a.No further MMR immunizations are needed. b.MMR vaccination needs to be repeated at 4 to 6 years of age. c.MMR immunization needs to be repeated every 4 years until age 21 years. d.A recommendation cannot be made until the physician is consulted.

B

During a visit to the clinic, a woman in her seventh month of pregnancy complains that her legs feel "heavy in the calf" and that she often has foot cramps at night. The nurse notices that the patient has dilated, tortuous veins apparent in her lower legs. Which condition is reflected by these findings? a. Deep-vein thrombophlebitis b. Varicose veins c. Lymphedema d. Raynaud phenomenon

B

During an assessment of a 22-year-old woman who sustained a head injury from an automobile accident 4 hours earlier, the nurse notices the following changes: pupils were equal, but now the right pupil is fully dilated and nonreactive, and the left pupil is 4 mm and reacts to light. What do these findings suggest? a. Injury to the right eye b. Increased intracranial pressure c. Test inaccurately performed d. Normal response after a head injury

B

During an assessment of a patient who has been homeless for several years, the nurse notices that his tongue is magenta in color, which is an indication of a deficiency in what mineral and/or vitamin? a. Iron b. Riboflavin c. Vitamin D and calcium

B

During an assessment of a patient's family history, the nurse constructs a genogram. Which statement best describes a genogram? a.List of diseases present in a person's near relatives b.Graphic family tree that uses symbols to depict the gender, relationship, and age of immediate family members c.Drawing that depicts the patient's family members up to five generations back d.Description of the health of a person's children and grandchildren

B

During an assessment of the CNs, the nurse finds the following: asymmetry when the patient smiles or frowns, uneven lifting of the eyebrows, sagging of the lower eyelids, and escape of air when the nurse presses against the right puffed cheek. This would indicate dysfunction of which of these CNs? a. Motor component of CN IV b. Motor component of CN VII c. Motor and sensory components of CN XI d. Motor component of CN X and sensory component of CN VII

B

During an assessment, the nurse has elevated a patient's legs 12 inches off the table and has had him wag his feet to drain off venous blood. After helping him sit up and dangle his legs over the side of the table, the nurse should expect that a normal finding at this point would be: a. Significant elevational pallor. b. Venous filling within 15 seconds. c. No change in the coloration of the skin. d. Color returning to the feet within 20 seconds of assuming a sitting position.

B

During an assessment, the nurse notices that a patient's left arm is swollen from the shoulder down to the fingers, with nonpitting brawny edema. The right arm is normal. The patient had a left-sided mastectomy 1 year ago. The nurse suspects which problem? a. Venous stasis b. Lymphedema c. Arteriosclerosis d. Deep-vein thrombosis

B

During an assessment, the nurse uses the profile sign to detect: a. Pitting edema. b. Early clubbing. c. Symmetry of the fingers. d. Insufficient capillary refill.

B

During an examination, the nurse notices severe nystagmus in both eyes of a patient. Which conclusion by the nurse is correct? Severe nystagmus in both eyes: a. Is a normal occurrence. b. May indicate disease of the cerebellum or brainstem. c. Is a sign that the patient is nervous about the examination. d. Indicates a visual problem, and a referral to an ophthalmologist is indicated.

B

How should the nurse perform a triceps skinfold assessment? A) After pinching the skin and fat, apply the calipers vertically to the fat fold. B) Gently pinch the skin and fat on the front of the patient's arm and then apply calipers. C) After applying the calipers, wait 3 seconds before taking a reading. Repeat the procedure three times. D) Instruct the patient to stand with the back to the examiner and arms folded across the chest and pinch the skin on the forearm.

B

In obtaining a health history on a 74-year-old patient, the nurse notes that he drinks alcohol daily and that he has noticed a tremor in his hands that affects his ability to hold things. With this information, what response should the nurse make? a. "Does your family know you are drinking every day?" b. "Does the tremor change when you drink alcohol?" c. "We'll do some tests to see what is causing the tremor." d. "You really shouldn't drink so much alcohol; it may be causing your tremor."

B

In the majority culture of America, coughing, sweating, and diarrhea are symptoms of an illness. For some individuals of Mexican-American origin, however, these symptoms are a normal part of living. The nurse recognizes that this difference is true, probably because Mexican-Americans: a. Have less efficient immune systems and are often ill. b. Consider these symptoms part of normal living, not symptoms of ill health. c. Come from Mexico, and coughing is normal and healthy there. d. Are usually in a lower socioeconomic group and are more likely to be sick.

B

Symptoms, such as pain, are often influenced by a person's cultural heritage. Which of the following is a true statement regarding pain? a. Nurses' attitudes toward their patients' pain are unrelated to their own experiences with pain. b. Nurses need to recognize that many cultures practice silent suffering as a response to pain. c. A nurse's area of clinical practice will most likely determine his or her assessment of a patient's pain. d. A nurse's years of clinical experience and current position are strong indicators of his or her response to patient pain.

B

The ability that humans have to perform very skilled movements such as writing is controlled by the: a. Basal ganglia. b. Corticospinal tract. c. Spinothalamic tract. d. Extrapyramidal tract.

B

The assessment of a 60-year-old patient has taken longer than anticipated. In testing his pain perception, the nurse decides to complete the test as quickly as possible. When the nurse applies the sharp point of the pin on his arm several times, he is only able to identify these as one "very sharp prick." What would be the most accurate explanation for this? a. The patient has hyperesthesia as a result of the aging process. b. This response is most likely the result of the summation effect. c. The nurse was probably not poking hard enough with the pin in the other areas. d. The patient most likely has analgesia in some areas of arm and hyperalgesia in others.

B

The nurse has collected the following information on a patient: palpated blood pressure—180; auscultated blood pressure—170/100 mm Hg; apical pulse—60; radial pulse—70. What is the patient's pulse pressure? A) 10 B) 70 C) 80 D) 100

B

The nurse is assessing a new patient who has recently immigrated to the United States. Which question is appropriate to add to the health history? a."Why did you come to the United States?" b."When did you come to the United States and from what country?" c."What made you leave your native country?" d."Are you planning to return to your home?"

B

The nurse is assessing a patients skin during an office visit. What part of the hand and technique should be used to best assess the patients skin temperature? a. Fingertips; they are more sensitive to small changes in temperature. b. Dorsal surface of the hand; the skin is thinner on this surface than on the palms. c. Ulnar portion of the hand; increased blood supply in this area enhances temperature sensitivity. d. Palmar surface of the hand; this surface is the most sensitive to temperature variations because of its increased nerve supply in this area.

B

The nurse is assessing an 8-year-old child whose growth rate measures below the third percentile for a child his age. He appears much younger than his stated age, and he is chubby with infantile facial features. Which condition does this child have? A) Hypopituitary dwarfism B) Achondroplastic dwarfism C) Marfan syndrome D) Acromegaly

B

The nurse is assessing the body weight as a percentage of ideal body weight on an adolescent patient who was admitted for suspected anorexia nervosa. The patient's usual weight was 125 pounds, but today she weighs 98 pounds. The nurse calculates the patient's ideal body weight, and reaches which conclusion? A) She is experiencing mild malnutrition. B) She is experiencing moderate malnutrition. C) She is experiencing severe malnutrition. D) Her current weight is still within expected parameters.

B

The nurse is assessing the neurologic status of a patient who has a late-stage brain tumor. With the reflex hammer, the nurse draws a light stroke up the lateral side of the sole of the foot and inward, across the ball of the foot. In response, the patient's toes fan out, and the big toe shows dorsiflexion. The nurse interprets this result as: a. Negative Babinski sign, which is normal for adults. b. Positive Babinski sign, which is abnormal for adults. c. Clonus, which is a hyperactive response. d. Achilles reflex, which is an expected response.

B

The nurse is assessing the vital signs of a 20-year-old male marathon runner and documents the following vital signs: temperature—97 F; pulse—48 beats per minute; respirations—14 per minute; blood pressure—104/68 mm Hg. Which statement is true about these results? A) The patient is experiencing tachycardia. B) These are normal vital signs for a healthy, athletic adult. C) The patient's pulse rate is not normal—his physician should be notified. D) On the basis of today's readings, the patient should return to the clinic in 1 week.

B

The nurse is attempting to assess the femoral pulse in a patient who is obese. Which of these actions would be most appropriate? a. The patient is asked to assume a prone position. b. The patient is asked to bend his or her knees to the side in a froglike position. c. The nurse firmly presses against the bone with the patient in a semi-Fowler position. d. The nurse listens with a stethoscope for pulsations; palpating the pulse in an obese person is extremely difficult.

B

The nurse is conducting a class for new graduate nurses. During the teaching session, the nurse should keep in mind that novice nurses, without a background of skills and experience from which to draw, are more likely to make their decisions using: a. Intuition. b. A set of rules. c. Articles in journals. d. Advice from supervisors.

B

The nurse is conducting a health fair for older adults. Which statement is true regarding vital sign measurements in aging adults? A) The pulse is more difficult to palpate because of the stiffness of the blood vessels. B) An increased respiratory rate and a shallower inspiratory phase are expected findings. C) A decreased pulse pressure occurs from changes in systolic and diastolic blood pressures. D) Changes in the body's temperature regulatory mechanism leave the aging person more likely to develop a fever.

B

The nurse is conducting a heritage assessment. Which question is most appropriate for this assessment? a. "What is your religion?" b. "Do you mostly participate in the religious traditions of your family?" c. "Do you smoke?" d. "Do you have a history of heart disease?"

B

The nurse is counting an infant's respirations. Which technique is correct? A) Watch the chest rise and fall. B) Watch the abdomen for movement. C) Place a hand across the infant's chest. D) Use a stethoscope to listen to the breath sounds.

B

The nurse is examining a patient who is complaining of "feeling cold." Which is a mechanism of heat loss in the body? A) Exercise B) Radiation C) Metabolism D) Food digestion

B

The nurse is examining an infant and prepares to elicit the Moro reflex at which time during the examination? a. When the infant is sleeping b. At the end of the examination c. Before auscultation of the thorax d. Halfway through the examination

B

The nurse is examining the lymphatic system of a healthy 3-year-old child. Which finding should the nurse expect? a. Excessive swelling of the lymph nodes b. Presence of palpable lymph nodes c. No palpable nodes because of the immature immune system of a child d. Fewer numbers and a smaller size of lymph nodes compared with those of an adult

B

The nurse is performing a functional assessment on an 82-year-old patient who recently had a stroke. Which of these questions would be most important to ask? a."Do you wear glasses?" b."Are you able to dress yourself?" c."Do you have any thyroid problems?" d."How many times a day do you have a bowel movement?"

B

The nurse is preparing to examine a 4-year-old child. Which action is appropriate for this age group? a. Explain the procedures in detail to alleviate the childs anxiety. b. Give the child feedback and reassurance during the examination. c. Do not ask the child to remove his or her clothes because children at this age are usually very private. d. Perform an examination of the ear, nose, and throat first, and then examine the thorax and abdomen

B

The nurse is reviewing an assessment of a patient's peripheral pulses and notices that the documentation states that the radial pulses are "2+." The nurse recognizes that this reading indicates what type of pulse? a. Bounding b. Normal c. Weak d. Absent

B

The nurse is reviewing concepts of cultural aspects of pain. Which statement is true regarding pain? a. All patients will behave the same way when in pain. b. Just as patients vary in their perceptions of pain, so will they vary in their expressions of pain. c. Cultural norms have very little to do with pain tolerance, because pain tolerance is always biologically determined. d. A patient's expression of pain is largely dependent on the amount of tissue injury associated with the pain.

B

The nurse is reviewing the development of culture. Which statement is correct regarding the development of one's culture? Culture is: a. Genetically determined on the basis of racial background. b. Learned through language acquisition and socialization. c. A nonspecific phenomenon and is adaptive but unnecessary. d. Biologically determined on the basis of physical characteristics.

B

The nurse is reviewing the nutritional assessment of an 82-year-old patient. Which of these factors will most likely affect the nutritional status of an older adult? a. Increase in taste and smell b. Living alone on a fixed income c. Change in cardiovascular status d. Increase in gastrointestinal motility and absorption

B

The nurse is reviewing the risk factors for venous disease. Which of these situations best describes a person at highest risk for the development of venous disease? a. Woman in her second month of pregnancy b. Person who has been on bed rest for 4 days c. Person with a 30-year, 1 pack per day smoking habit d. Older adult taking anticoagulant medication

B

The nurse is reviewing theories of illness. The germ theory, which states that microscopic organisms such as bacteria and viruses are responsible for specific disease conditions, is a basic belief of which theory of illness? a. Holistic b. Biomedical c. Naturalistic d. Magicoreligious

B

The nurse is teaching a class on basic assessment skills. Which of these statements is true regarding the stethoscope and its use? a. Slope of the earpieces should point posteriorly (toward the occiput). b. Although the stethoscope does not magnify sound, it does block out extraneous room noise. c. Fit and quality of the stethoscope are not as important as its ability to magnify sound. d. Ideal tubing length should be 22 inches to dampen the distortion of sound.

B

The nurse is teaching a review class on the lymphatic system. A participant shows correct understanding of the material with which statement? a. "Lymph flow is propelled by the contraction of the heart." b. "The flow of lymph is slow, compared with that of the blood." c. "One of the functions of the lymph is to absorb lipids from the biliary tract." d. "Lymph vessels have no valves; therefore, lymph fluid flows freely from the tissue spaces into the bloodstream."

B

The nurse keeps in mind that the most important reason to share information and to offer brief teaching while performing the physical examination is to help the: a. Examiner feel more comfortable and to gain control of the situation. b. Examiner to build rapport and to increase the patients confidence in him or her. c. Patient understand his or her disease process and treatment modalities. d. Patient identify questions about his or her disease and the potential areas of patient education.

B

The nurse needs to perform anthropometric measures of an 80-year-old man who is confined to a wheelchair. Which of the following is true in this situation? A) Changes in fat distribution will affect the waist-to-hip ratio. B) Height measurements may not be accurate because of changes in bone. C) Declining muscle mass will affect the triceps skinfold measure. D) Mid-arm circumference is difficult to obtain because of loss of skin elasticity.

B

The nurse notices a colleague is preparing to check the blood pressure of a patient who is obese by using a standard-sized blood pressure cuff. The nurse should expect the reading to: A) yield a falsely low blood pressure. B) yield a falsely high blood pressure. C) be the same regardless of cuff size. D) vary as a result of the technique of the person performing the assessment.

B

The nurse places a key in the hand of a patient and he identifies it as a penny. What term would the nurse use to describe this finding? a. Extinction b. Astereognosis c. Graphesthesia d. Tactile discrimination

B

The nurse recognizes that an example of a person who is heritage consistent would be a: a. Woman who has adapted her clothing to the clothing style of her new country. b. Woman who follows the traditions that her mother followed regarding meals. c. Man who is not sure of his ancestor's country of origin. d. Child who is not able to speak his parents' native language.

B

The review of systems provides the nurse with: a.Physical findings related to each system. b.Information regarding health promotion practices. c.An opportunity to teach the patient medical terms. d.Information necessary for the nurse to diagnose the patient's medical problem.

B

When assessing the intensity of a patient's pain, which question by the nurse is appropriate? A) "What makes your pain better or worse?" B) "How much pain do you have now?" C) "How does pain limit your activities?" D) "What does your pain feel like?"

B

When the nurse is evaluating the reliability of a patient's responses, which of these statements would be correct? The patient: a.Has a history of drug abuse and therefore is not reliable. b.Provided consistent information and therefore is reliable. c.Smiled throughout interview and therefore is assumed reliable. d.Would not answer questions concerning stress and therefore is not reliable.

B

Which critical thinking skill helps the nurse see relationships among the data? a. Validation b. Clustering related cues c. Identifying gaps in data d. Distinguishing relevant from irrelevant

B

Which of these actions illustrates the correct technique the nurse should use when assessing oral temperature with a mercury thermometer? A) Wait 30 minutes if the patient has ingested hot or iced liquids. B) Leave the thermometer in place 3 to 4 minutes if the patient is afebrile. C) Place the thermometer in front of the tongue and have the patient close his or her lips. D) Shake the mercury-in-glass thermometer down to 98° F before taking the temperature.

B

Which of these specific measurements is the best index of a child's general health? A) Vital signs B) Height and weight C) Head circumference D) Chest circumference

B

Which statement concerning the areas of the brain is true? a. The cerebellum is the center for speech and emotions. b. The hypothalamus controls body temperature and regulates sleep. c. The basal ganglia are responsible for controlling voluntary movements. d. Motor pathways of the spinal cord and brainstem synapse in the thalamus.

B

Which statement is true regarding the arterial system? a. Arteries are large-diameter vessels. b. The arterial system is a high-pressure system. c. The walls of arteries are thinner than those of the veins. d. Arteries can greatly expand to accommodate a large blood volume increase.

B

With which of these patients would it be most appropriate for the nurse to use games during the assessment, such as having the patient blow out the light on the penlight? a. Infant b. Preschool child c. School-age child d. Adolescent

B

A patient says that she has recently noticed a lump in the front of her neck below her "Adam's apple" that seems to be getting bigger. During the assessment, the finding that leads the nurse to suspect that this may not be a cancerous thyroid nodule is that the lump (nodule): a.Is tender. b.Is mobile and not hard. c.Disappears when the patient smiles. d.Is hard and fixed to the surrounding structures.

B Painless, rapidly growing nodules may be cancerous, especially the appearance of a single nodule in a young person. However, cancerous nodules tend to be hard and fixed to surrounding structures, not mobile.

A patient has come in for an examination and states, "I have this spot in front of my ear lobe on my cheek that seems to be getting bigger and is tender. What do you think it is?" The nurse notes swelling below the angle of the jaw and suspects that it could be an inflammation of his: a.Thyroid gland. b.Parotid gland. c.Occipital lymph node. d.Submental lymph node.

B Swelling of the parotid gland is evident below the angle of the jaw and is most visible when the head is extended. Painful inflammation occurs with mumps, and swelling also occurs with abscesses or tumors. Swelling occurs anterior to the lower ear lobe.

A mother brings her newborn in for an assessment and asks, "Is there something wrong with my baby? His head seems so big." Which statement is true regarding the relative proportions of the head and trunk of the newborn? a.At birth, the head is one fifth the total length. b.Head circumference should be greater than chest circumference at birth. c.The head size reaches 90% of its final size when the child is 3 years old. d.When the anterior fontanel closes at 2 months, the head will be more proportioned to the body.

B The nurse recognizes that during the fetal period, head growth predominates. Head size is greater than chest circumference at birth, and the head size grows during childhood, reaching 90% of its final size when the child is age 6 years.

The physician reports that a patient with a neck tumor has a tracheal shift. The nurse is aware that this means that the patient's trachea is: a.Pulled to the affected side. b.Pushed to the unaffected side. c.Pulled downward. d.Pulled downward in a rhythmic pattern.

B The trachea is pushed to the unaffected side with an aortic aneurysm, a tumor, unilateral thyroid lobe enlargement, or a pneumothorax. The trachea is pulled to the affected side with large atelectasis, pleural adhesions, or fibrosis. Tracheal tug is a rhythmic downward pull that is synchronous with systole and occurs with aortic arch aneurysm.

During an admission assessment of a patient with dementia, the nurse assesses for pain because the patient has recently had several falls. Which of these are appropriate for the nurse to assess in a patient with dementia? Select all that apply. A) Ask the patient, "Do you have pain?" B) Assess the patient's breathing independent of vocalization. C) Note whether the patient is calling out, groaning, or crying. D) Have the patient rate pain on a 1 to 10 scale. E) Observe the patient's body language for pacing and agitation.

B C E

The nurse is assessing a patient who is obese for signs of metabolic syndrome. This condition is diagnosed when three or more certain risk factors are present. Which of these assessment findings are risk factors for metabolic syndrome? Select all that apply. a. Fasting plasma glucose level less than 100 mg/dL b. Fasting plasma glucose level greater than or equal to 110 mg/dL c. Blood pressure reading of 140/90 mm Hg d. Blood pressure reading of 110/80 mm Hg e. Triglyceride level of 120 mg/dL

B, C

The nurse is conducting a developmental history on a 5-year-old child. Which questions are appropriate to ask the parents for this part of the assessment? Select all that apply. a."How much junk food does your child eat?" b."How many teeth has he lost, and when did he lose them?" c."Is he able to tie his shoelaces?" d."Does he take a children's vitamin?" e."Can he tell time?" f."Does he have any food allergies?"

B, C, E

The nurse is assessing a 1-month-old infant at his well-baby checkup. Which assessment findings are appropriate for this age? Select all that apply. a.Head circumference equal to chest circumference b.Head circumference greater than chest circumference c.Head circumference less than chest circumference d.Fontanels firm and slightly concave e.Absent tonic neck reflex f.Nonpalpable cervical lymph nodes

B, D, F An infant's head circumference is larger than the chest circumference. At age 2 years, both measurements are the same. During childhood, the chest circumference grows to exceed the head circumference by 5 to 7 cm. The fontanels should feel firm and slightly concave in the infant, and they should close by age 9 months. The tonic neck reflex is present until between 3 and 4 months of age, and cervical lymph nodes are normally nonpalpable in an infant.

. The nurse is reviewing aspects of cultural care. Which statements illustrate proper cultural care? Select all that apply. a. Examine the patient within the context of one's own cultural health and illness practices. b. Select questions that are not complex. c. Ask questions rapidly. d. Touch patients within the cultural boundaries of their heritage. e. Pace questions throughout the physical examination.

B,D,E

While performing the otoscopic examination of a 3-year-old boy who has been pulling on his left ear, the nurse finds that his left tympanic membrane is bright red and that the light reflex is not visible. The nurse interprets these findings to indicate a(n): a. Fungal infection. b. Acute otitis media. c. Perforation of the eardrum. d. Cholesteatoma.

B. Acute otitis media. Absent or distorted light reflex and a bright red color of the eardrum are indicative of acute otitis media. (See Table 15-5 for descriptions of the other conditions.)

The nurse is assessing a 16-year-old patient who has suffered head injuries from a recent motor vehicle accident. Which of these statements indicates the most important reason for assessing for any drainage from the ear canal? a. If the drum has ruptured, then purulent drainage will result. b. Bloody or clear watery drainage can indicate a basal skull fracture. c. The auditory canal many be occluded from increased cerumen. d. Foreign bodies from the accident may cause occlusion of the canal.

B. Bloody or clear watery drainage can indicate a basal skull fracture. Frank blood or clear watery drainage (cerebrospinal leak) after a trauma suggests a basal skull fracture and warrants immediate referral. Purulent drainage indicates otitis externa or otitis media.

Which of these assessment findings would the nurse expect to see when examining the eyes of a black patient? a. Increased night vision b. Dark retinal background c. Increased photosensitivity d. Narrowed palpebral fissures

B. Dark retinal background An ethnically based variability in the color of the iris and in retinal pigmentation exists, with darker irides having darker retinas behind them.

A patient comes into the clinic complaining of pain in her right eye. On examination, the nurse sees a pustule at the lid margin that is painful to touch, red, and swollen. The nurse recognizes that this is a: a. Chalazion. b. Hordeolum (stye). c. Dacryocystitis. d. Blepharitis.

B. Hordeolum (stye). A hordeolum, or stye, is a painful, red, and swollen pustule at the lid margin. A chalazion is a nodule protruding on the lid, toward the inside, and is nontender, firm, with discrete swelling. Dacryocystitis is an inflammation of the lacrimal sac. Blepharitis is inflammation of the eyelids (see Table 14-3).

The nurse suspects that a patient has otitis media. Early signs of otitis media include which of these findings of the tympanic membrane? a. Red and bulging b. Hypomobility c. Retraction with landmarks clearly visible d. Flat, slightly pulled in at the center, and moves with insufflation

B. Hypomobility An early sign of otitis media is hypomobility of the tympanic membrane. As pressure increases, the tympanic membrane begins to bulge.

A 31-year-old patient tells the nurse that he has noticed pain in his left ear when people speak loudly to him. The nurse knows that this finding: a. Is normal for people of his age. b. Is a characteristic of recruitment. c. May indicate a middle ear infection. d. Indicates that the patient has a cerumen impaction.

B. Is a characteristic of recruitment. Recruitment is significant hearing loss occurring when speech is at low intensity, but sound actually becomes painful when the speaker repeats at a louder volume. The other responses are not correct.

The nurse is performing a middle ear assessment on a 15-year-old patient who has had a history of chronic ear infections. When examining the right tympanic membrane, the nurse sees the presence of dense white patches. The tympanic membrane is otherwise unremarkable. It is pearly, with the light reflex at 5 o'clock and landmarks visible. The nurse should: a. Refer the patient for the possibility of a fungal infection. b. Know that these are scars caused from frequent ear infections. c. Consider that these findings may represent the presence of blood in the middle ear. d. Be concerned about the ability to hear because of this abnormality on the tympanic membrane.

B. Know that these are scars caused from frequent ear infections. Dense white patches on the tympanic membrane are sequelae of repeated ear infections. They do not necessarily affect hearing.

The nurse is reviewing in age-related changes in the eye for a class. Which of these physiologic changes is responsible for presbyopia? a. Degeneration of the cornea b. Loss of lens elasticity c. Decreased adaptation to darkness d. Decreased distance vision abilities

B. Loss of lens elasticity The lens loses elasticity and decreases its ability to change shape to accommodate for near vision. This condition is called presbyopia.

The nurse is performing the diagnostic positions test. Normal findings would be which of these results? a. Convergence of the eyes b. Parallel movement of both eyes c. Nystagmus in extreme superior gaze d. Slight amount of lid lag when moving the eyes from a superior to an inferior position

B. Parallel movement of both eyes A normal response for the diagnostic positions test is parallel tracking of the object with both eyes. Eye movement that is not parallel indicates a weakness of an extraocular muscle or dysfunction of the CN that innervates it.

When examining the ear with an otoscope, the nurse notes that the tympanic membrane should appear: a. Light pink with a slight bulge. b. Pearly gray and slightly concave. c. Pulled in at the base of the cone of light. d. Whitish with a small fleck of light in the superior portion.

B. Pearly gray and slightly concave. The tympanic membrane is a translucent membrane with a pearly gray color and a prominent cone of light in the anteroinferior quadrant, which is the reflection of the otoscope light. The tympanic membrane is oval and slightly concave, pulled in at its center by the malleus, which is one of the middle ear ossicles.

A patient in her first trimester of pregnancy is diagnosed with rubella. Which of these statements is correct regarding the significance of this in relation to the infant's hearing? a. Rubella may affect the mother's hearing but not the infant's. b. Rubella can damage the infant's organ of Corti, which will impair hearing. c. Rubella is only dangerous to the infant in the second trimester of pregnancy. d. Rubella can impair the development of CN VIII and thus affect hearing.

B. Rubella can damage the infant's organ of Corti, which will impair hearing. If maternal rubella infection occurs during the first trimester, then it can damage the organ of

A patient comes to the emergency department after a boxing match, and his left eye is swollen almost shut. He has bruises on his face and neck. He says he is worried because he "can't see well" from his left eye. The physician suspects retinal damage. The nurse recognizes that signs of retinal detachment include: a. Loss of central vision. b. Shadow or diminished vision in one quadrant or one half of the visual field. c. Loss of peripheral vision. d. Sudden loss of pupillary constriction and accommodation.

B. Shadow or diminished vision in one quadrant or one half of the visual field. With retinal detachment, the person has shadows or diminished vision in one quadrant or one half of the visual field. The other responses are not signs of retinal detachment.

The nurse is conducting a visual examination. Which of these statements regarding visual pathways and visual fields is true? a. The right side of the brain interprets the vision for the right eye. b. The image formed on the retina is upside down and reversed from its actual appearance in the outside world. c. Light rays are refracted through the transparent media of the eye before striking the pupil. d. Light impulses are conducted through the optic nerve to the temporal lobes of the brain.

B. The image formed on the retina is upside down and reversed from its actual appearance in the outside world. The image formed on the retina is upside down and reversed from its actual appearance in the outside world. The light rays are refracted through the transparent media of the eye before striking the retina, and the nerve impulses are conducted through the optic nerve tract to the visual cortex of the occipital lobe of the brain. The left side of the brain interprets vision for the right eye.

A patient's vision is recorded as 20/30 when the Snellen eye chart is used. The nurse interprets these results to indicate that: a. At 30 feet the patient can read the entire chart. b. The patient can read at 20 feet what a person with normal vision can read at 30 feet. c. The patient can read the chart from 20 feet in the left eye and 30 feet in the right eye. d. The patient can read from 30 feet what a person with normal vision can read from 20 feet.

B. The patient can read at 20 feet what a person with normal vision can read at 30 feet. The top number indicates the distance the person is standing from the chart; the denominator gives the distance at which a normal eye can see.

The nurse is performing an eye assessment on an 80-year-old patient. Which of these findings is considered abnormal? a. Decrease in tear production b. Unequal pupillary constriction in response to light c. Presence of arcus senilis observed around the cornea d. Loss of the outer hair on the eyebrows attributable to a decrease in hair follicles

B. Unequal pupillary constriction in response to light Pupils are small in the older adult, and the pupillary light reflex may be slowed, but pupillary constriction should be symmetric. The assessment findings in the other responses are considered normal in older persons.

A 17-year-old student is a swimmer on her high school's swim team. She has had three bouts of otitis externa this season and wants to know what to do to prevent it. The nurse instructs her to: a. Use a cotton-tipped swab to dry the ear canals thoroughly after each swim. b. Use rubbing alcohol or 2% acetic acid eardrops after every swim. c. Irrigate the ears with warm water and a bulb syringe after each swim. d. Rinse the ears with a warmed solution of mineral oil and hydrogen peroxide.

B. Use rubbing alcohol or 2% acetic acid eardrops after every swim. With otitis externa (swimmer's ear), swimming causes the external canal to become waterlogged and swell; skinfolds are set up for infection. Otitis externa can be prevented by using rubbing alcohol or 2% acetic acid eardrops after every swim.

In performing a voice test to assess hearing, which of these actions would the nurse perform? a. Shield the lips so that the sound is muffled. b. Whisper a set of random numbers and letters, and then ask the patient to repeat them. c. Ask the patient to place his finger in his ear to occlude outside noise. d. Stand approximately 4 feet away to ensure that the patient can really hear at this distance.

B. Whisper a set of random numbers and letters, and then ask the patient to repeat them. With the head 30 to 60 cm (1 to 2 feet) from the patient's ear, the examiner exhales and slowly whispers a set of random numbers and letters, such as "5, B, 6." Normally, the patient is asked to repeat each number and letter correctly after hearing the examiner say them.

During an oral examination of a 4-year-old Native-American child, the nurse notices that her uvula is partially split. Which of these statements is accurate? a.This condition is a cleft palate and is common in Native Americans. b.A bifid uvula may occur in some Native-American groups. c.This condition is due to an injury and should be reported to the authorities. d.A bifid uvula is palatinus, which frequently occurs in Native Americans.

B. a bifid uvula may occur in some Native-American groups.

The nurse is using an otoscope to assess the nasal cavity. Which of these techniques is correct? a.Inserting the speculum at least 3 cm into the vestibule b.Avoiding touching the nasal septum with the speculum c.Gently displacing the nose to the side that is being examined d.Keeping the speculum tip medial to avoid touching the floor of the nares

B. avoiding touching the nasal septum with the speculum

A 40-year-old patient who has just finished chemotherapy for breast cancer tells the nurse that she is concerned about her mouth. During the assessment the nurse finds areas of buccal mucosa that are raw and red with some bleeding, as well as other areas that have a white, cheesy coating. The nurse recognizes that this abnormality is: a.Aphthous ulcers. b.Candidiasis. c.Leukoplakia. d.Koplik spots.

B. candidiasis

During an assessment, a patient mentions that "I just can't smell like I used to. I can barely smell the roses in my garden. Why is that?" For which possible causes of changes in the sense of smell will the nurse assess? Select all that apply. a.Chronic alcohol use b.Cigarette smoking c.Frequent episodes of strep throat d.Chronic allergies e.Aging f.Herpes simplex virus I

B. cigarette smoking D. chronic allergies E. aging

The primary purpose of the ciliated mucous membrane in the nose is to: a.Warm the inhaled air. b.Filter out dust and bacteria. c.Filter coarse particles from inhaled air. d.Facilitate the movement of air through the nares.

B. filter out dust and bacteria

The nurse is palpating the sinus areas. If the findings are normal, then the patient should report which sensation? a.No sensation b.Firm pressure c.Pain during palpation d.Pain sensation behind eyes

B. firm pressure

A woman who is in the second trimester of pregnancy mentions that she has had "more nosebleeds than ever" since she became pregnant. The nurse recognizes that this is a result of: a.A problem with the patient's coagulation system. b.Increased vascularity in the upper respiratory tract as a result of the pregnancy. c.Increased susceptibility to colds and nasal irritation. d.Inappropriate use of nasal sprays.

B. increased vascularity in the upper respiratory tract as a result of the pregnancy

In assessing the tonsils of a 30 year old, the nurse notices that they are involuted, granular in appearance, and appear to have deep crypts. What is correct response to these findings? a.Refer the patient to a throat specialist. b.No response is needed; this appearance is normal for the tonsils. c.Continue with the assessment, looking for any other abnormal findings. d.Obtain a throat culture on the patient for possible streptococcal (strep) infection.

B. no response is needed; this appearance is normal for the tonsils

During an examination, a patient states that she was diagnosed with open-angle glaucoma 2 years ago. The nurse assesses for characteristics of open-angle glaucoma. Which of these are characteristics of open-angle glaucoma? Select all that apply. a. Patient may experience sensitivity to light, nausea, and halos around lights. b. Patient experiences tunnel vision in the late stages. c. Immediate treatment is needed. d. Vision loss begins with peripheral vision. e. Open-angle glaucoma causes sudden attacks of increased pressure that cause blurred vision. f. Virtually no symptoms are exhibited.

B. patient experiences tunnel vision in the late stages. D. vision loss begins with peripheral vision F. virtually no symptoms are exhibited Open-angle glaucoma is the most common type of glaucoma; virtually no symptoms are exhibited. Vision loss begins with the peripheral vision, which often goes unnoticed because individuals learn to compensate intuitively by turning their heads. The other characteristics are those of closed-angle glaucoma.

The nurse is testing the hearing of a 78-year-old man and is reminded of the changes in hearing that occur with aging that include which of the following? Select all that apply. a. Hearing loss related to aging begins in the mid 40s. b. Progression of hearing loss is slow. c. The aging person has low-frequency tone loss. d. The aging person may find it harder to hear consonants than vowels. e. Sounds may be garbled and difficult to localize. f. Hearing loss reflects nerve degeneration of the middle ear.

B. progression of hearing loss is slow D. the aging person may find it harder to hear consonants than vowels E. sounds may be garbled and difficult to localize Presbycusis is a type of hearing loss that occurs with aging and is found in 60% of those older than 65 years. It is a gradual sensorineural loss caused by nerve degeneration in the inner ear or auditory nerve, and it slowly progresses after the age of 50 years. The person first notices a high-frequency tone loss; it is harder to hear consonants (high-pitched components of speech) than vowels, which makes words sound garbled. The ability to localize sound is also impaired.

While obtaining a health history, a patient tells the nurse that he has frequent nosebleeds and asks the best way to get them to stop. What would be the nurse's best response? a."While sitting up, place a cold compress over your nose." b."Sit up with your head tilted forward and pinch your nose." c."Just allow the bleeding to stop on its own, but don't blow your nose." d."Lie on your back with your head tilted back and pinch your nose."

B. sit up with your head tilted forward and pinch your nose

When examining the mouth of an older patient, the nurse recognizes which finding is due to the aging process? a.Teeth appearing shorter b.Tongue that looks smoother in appearance c.Buccal mucosa that is beefy red in appearance d.Small, painless lump on the dorsum of the tongue

B. tongue that looks smoother in appearance

The nurse is teaching a health class to high-school boys. When discussing the topic of using smokeless tobacco (SLT), which of these statements are accurate? Select all that apply. a.One pinch of SLT in the mouth for 30 minutes delivers the equivalent of one cigarette. b.Using SLT has been associated with a greater risk of oral cancer than smoking. c.Pain is an early sign of oral cancer. d.Pain is rarely an early sign of oral cancer. e.Tooth decay is another risk of SLT because of the use of sugar as a sweetener. f.SLT is considered a healthy alternative to smoking.

B. using SLT has been associated with a greater risk of oral cancer than smoking. D. Pain is rarely an early sign of oral cancer. E. tooth decay is another risk of SLT because of the use of sugar as a sweetener.

While performing an assessment of the mouth, the nurse notices that the patient has a 1-cm ulceration that is crusted with an elevated border and located on the outer third of the lower lip. What other information would be most important for the nurse to assess? a.Nutritional status b.When the patient first noticed the lesion c.Whether the patient has had a recent cold d.Whether the patient has had any recent exposure to sick animals

B. when the patient first noticed the lesion

A 2-year-old child has been brought to the clinic for a well-child checkup. The best way for the nurse to begin the assessment is to: a. Ask the parent to place the child on the examining table. b. Have the parent remove all of the childs clothing before the examination. c. Allow the child to keep a security object such as a toy or blanket during the examination. d. Initially focus the interactions on the child, essentially ignoring the parent until the childs trust has been obtained.

C

A 30-year-old woman tells the nurse that she has been very unsteady and has had difficulty in maintaining her balance. Which area of the brain that is related to these findings would concern the nurse? a. Thalamus b. Brainstem c. Cerebellum d. Extrapyramidal tract

C

A 50-year-old patient has been brought to the emergency department after a housemate found that the patient could not get out of bed alone. He has lived in a group home for years but for several months has not participated in the activities and has stayed in his room. The nurse assesses for signs of undernutrition, and an x-ray study reveals that he has osteomalacia, which is a deficiency of: a. Iron. b. Riboflavin. c. Vitamin D and calcium. d. Vitamin C.

C

A 50-year-old woman with elevated total cholesterol and triglyceride levels is visiting the clinic to find out about her laboratory results. What would be important for the nurse to include in patient teaching in relation to these tests? a. The risks of undernutrition should be included. b. Offer methods to reduce the stress in her life. c. Provide information regarding a diet low in saturated fat. d. This condition is hereditary; she can do nothing to change the levels.

C

A 65-year-old man is brought to the emergency department after he was found dazed and incoherent, alone in his apartment. He has an enlarged liver and is moderately dehydrated. When evaluating his serum albumin level, the nurse must keep in mind that: A) serum albumin levels will increase as liver function decreases. B) serum albumin levels are a sensitive measure of early protein malnutrition. C) low serum albumin levels may be caused by reasons other than protein-calorie malnutrition. D) the results of the serum albumin measurement along with the patient's hemoglobin level should be considered.

C

A 65-year-old patient is experiencing pain in his left calf when he exercises that disappears after resting for a few minutes. The nurse recognizes that this description is most consistent with _______ the left leg. a. Venous obstruction of b. Claudication due to venous abnormalities in c. Ischemia caused by a partial blockage of an artery supplying d. Ischemia caused by the complete blockage of an artery supplying

C

A man who was found wandering in a park at 2 AM has been brought to the emergency department for an examination; he said he fell and hit his head. During the examination, the nurse asks him to use his index finger to touch the nurse's finger, then his own nose, then the nurse's finger again (which has been moved to a different location). The patient is clumsy, unable to follow the instructions, and overshoots the mark, missing the finger. The nurse should suspect which of the following? a. Cerebral injury b. Cerebrovascular accident c. Acute alcohol intoxication d. Peripheral neuropathy

C

A patient has a severed spinal nerve as a result of trauma. Which statement is true in this situation? a. Because there are 31 pairs of spinal nerves, no effect results if only one nerve is severed. b. The dermatome served by this nerve will no longer experience any sensation. c. The adjacent spinal nerves will continue to carry sensations for the dermatome served by the severed nerve. d. A severed spinal nerve will only affect motor function of the patient because spinal nerves have no sensory component.

C

A patient is being seen in the clinic for complaints of "fainting episodes that started last week." How should the nurse proceed with the examination? A) Take his blood pressure in both arms and thighs. B) Assist him to a lying position and begin taking his blood pressure. C) Record his blood pressure in the lying, sitting, and standing positions. D) Record his blood pressure in the lying and sitting positions and average these numbers. to obtain a mean blood pressure.

C

A patient is unable to perform rapid alternating movements such as rapidly patting her knees. The nurse should document this inability as: a. Ataxia. b. Astereognosis. c. Presence of dysdiadochokinesia. d. Loss of kinesthesia.

C

A patient with a lack of oxygen to his heart will have pain in his chest and possibly in the shoulder, arms, or jaw. The nurse knows that the best explanation why this occurs is which one of these statements? a. A problem exists with the sensory cortex and its ability to discriminate the location. b. The lack of oxygen in his heart has resulted in decreased amount of oxygen to the areas experiencing the pain. c. The sensory cortex does not have the ability to localize pain in the heart; consequently, the pain is felt elsewhere. d. A lesion has developed in the dorsal root, which is preventing the sensation from being transmitted normally.

C

A patient's blood pressure is 118/82. He asks the nurse to explain "what the numbers mean." The nurse's best reply would be: A) "The numbers are within normal range and are nothing to worry about." B) "The bottom number is the diastolic pressure and reflects the stroke volume of the heart." C) "The top number is the systolic blood pressure and reflects the pressure of the blood against the arteries when the heart contracts." D) "The concept of blood pressure is difficult to understand. The main thing to be concerned about is the top number, or systolic blood pressure."

C

A visiting nurse is making an initial home visit for a patient who has many chronic medical problems. Which type of data base is most appropriate to collect in this setting? a. A follow-up data base to evaluate changes at appropriate intervals b. An episodic data base because of the continuing, complex medical problems of this patient c. A complete health data base because of the nurses primary responsibility for monitoring the patients health d. An emergency data base because of the need to collect information and make accurate diagnoses rapidly

C

After a class on culture and ethnicity, the new graduate nurse reflects a correct understanding of the concept of ethnicity with which statement? a. "Ethnicity is dynamic and ever changing." b. "Ethnicity is the belief in a higher power." c. "Ethnicity pertains to a social group within the social system that claims shared values and traditions." d. "Ethnicity is learned from birth through the processes of language acquisition and socialization."

C

An older Mexican-American woman with traditional beliefs has been admitted to an inpatient care unit. A culturally sensitive nurse would: a. Contact the hospital administrator about the best course of action. b. Automatically get a curandero for her, because requesting one herself is not culturally appropriate. c. Further assess the patient's cultural beliefs and offer the patient assistance in contacting a curandero or priest if she desires. d. Ask the family what they would like to do because Mexican-Americans traditionally give control of decision making to their families.

C

During a class on cultural practices, the nurse hears the term cultural taboo. Which statement illustrates the concept of a cultural taboo? a. Believing that illness is a punishment of sin b. Trying prayer before seeking medical help c. Refusing to accept blood products as part of treatment d. Stating that a child's birth defect is the result of the parents' sins

C

During a routine office visit, a patient takes off his shoes and shows the nurse "this awful sore that won't heal." On inspection, the nurse notes a 3-cm round ulcer on the left great toe, with a pale ischemic base, well-defined edges, and no drainage. The nurse should assess for other signs and symptoms of: a. Varicosities. b. Venous stasis ulcer. c. Arterial ischemic ulcer. d. Deep-vein thrombophlebitis.

C

During a seminar on cultural aspects of nursing, the nurse recognizes that the definition stating "the specific and distinct knowledge, beliefs, skills, and customs acquired by members of a society" reflects which term? a. Mores b. Norms c. Culture d. Social learning

C

During an assessment of an 80-year-old patient, the nurse notices the following: an inability to identify vibrations at her ankle and to identify the position of her big toe, a slower and more deliberate gait, and a slightly impaired tactile sensation. All other neurologic findings are normal. The nurse should interpret that these findings indicate: a. CN dysfunction. b. Lesion in the cerebral cortex. c. Normal changes attributable to aging. d. Demyelination of nerves attributable to a lesion.

C

During an assessment of an older adult, the nurse should expect to notice which finding as a normal physiologic change associated with the aging process? a. Hormonal changes causing vasodilation and a resulting drop in blood pressure b. Progressive atrophy of the intramuscular calf veins, causing venous insufficiency c. Peripheral blood vessels growing more rigid with age, producing a rise in systolic blood pressure d. Narrowing of the inferior vena cava, causing low blood flow and increases in venous pressure resulting in varicosities

C

During an examination, the nurse can assess mental status by which activity? a. Examining the patients electroencephalogram b. Observing the patient as he or she performs an intelligence quotient (IQ) test c. Observing the patient and inferring health or dysfunction d. Examining the patients response to a specific set of questions

C

During an examination, the nurse notices that a female patient has a round "moon" face, central trunk obesity, and a cervical hump. Her skin is fragile with bruises. The nurse determines that the patient has which condition? A) Marfan syndrome B) Gigantism C) Cushing syndrome D) Acromegaly

C

During the examination, offering some brief teaching about the patients body or the examiners findings is often appropriate. Which one of these statements by the nurse is most appropriate? a. Your atrial dysrhythmias are under control. b. You have pitting edema and mild varicosities. c. Your pulse is 80 beats per minute, which is within the normal range. d. Im using my stethoscope to listen for any crackles, wheezes, or rubs.

C

How should the nurse perform a triceps skinfold assessment? a. After pinching the skin and fat, the calipers are vertically applied to the fat fold. b. The skin and fat on the front of the patient's arm are gently pinched, and then the calipers are applied. c. After applying the calipers, the nurse waits 3 seconds before taking a reading. After repeating the procedure three times, an average is recorded. d. The patient is instructed to stand with his or her back to the examiner and arms folded across the chest. The skin on the forearm is pinched.

C

In response to a question about stress, a 39-year-old woman tells the nurse that her husband and mother both died in the past year. Which response by the nurse is most appropriate? a."This has been a difficult year for you." b."I don't know how anyone could handle that much stress in 1 year!" c."What did you do to cope with the loss of both your husband and mother?" d."That is a lot of stress; now let's go on to the next section of your history."

C

Many Asians believe in the yin/yang theory, which is rooted in the ancient Chinese philosophy of Tao. Which statement most accurately reflects "health" in an Asian with this belief? a. A person is able to work and produce. b. A person is happy, stable, and feels good. c. All aspects of the person are in perfect balance. d. A person is able to care for others and function socially.

C

The mother of an 8-year-old boy is concerned about the amount of weight her son has gained. To determine whether this is a problem, the nurse will measure: A) arm span. B) waist-to-hip ratio. C) skinfold thickness. D) mid-upper arm circumference.

C

The nurse is assessing a 30-year-old unemployed immigrant from an underdeveloped country who has been in the United States for 1 month. Which of these problems related to his nutritional status might the nurse expect to find? a. Obesity b. Hypotension c. Osteomalacia (softening of the bones) d. Coronary artery disease

C

The nurse is assessing the pulses of a patient who has been admitted for untreated hyperthyroidism. The nurse should expect to find a(n) _______ pulse. a. Normal b. Absent c. Bounding d. Weak, thready

C

The nurse is caring for a patient who has just had neurosurgery. To assess for increased intracranial pressure, what would the nurse include in the assessment? a. CNs, motor function, and sensory function b. Deep tendon reflexes, vital signs, and coordinated movements c. Level of consciousness, motor function, pupillary response, and vital signs d. Mental status, deep tendon reflexes, sensory function, and pupillary response

C

The nurse is concerned about the skeletal protein reserves of a patient who has been hospitalized frequently for chronic lung disease. Which of these measurements would be necessary to include in the assessment? A) Body mass index B) Weight and height C) Mid-arm muscle area D) Ideal body weight and frame size

C

The nurse is examining a patients lower leg and notices a draining ulceration. Which of these actions is most appropriate in this situation? a. Washing hands, and contacting the physician b. Continuing to examine the ulceration, and then washing hands c. Washing hands, putting on gloves, and continuing with the examination of the ulceration d. Washing hands, proceeding with rest of the physical examination, and then continuing with the examination of the leg ulceration

C

The nurse is helping another nurse to take a blood pressure reading on a patient's thigh. Which action is correct regarding thigh pressure? A) Auscultate either the popliteal or femoral vessels to obtain a thigh pressure. B) The best position to measure thigh pressure is the supine position with the knee slightly bent. C) If the blood pressure in the arm is high in an adolescent, then compare it with the thigh pressure. D) The thigh pressure is lower than that in the arm due to distance away from the heart and the size of the popliteal vessels.

C

The nurse is performing a neurologic assessment on a 41-year-old woman with a history of diabetes. When testing her ability to feel the vibrations of a tuning fork, the nurse notices that the patient is unable to feel vibrations on the great toe or ankle bilaterally, but she is able to feel vibrations on both patellae. Given this information, what would the nurse suspect? a. Hyperalgesia b. Hyperesthesia c. Peripheral neuropathy d. Lesion of sensory cortex

C

The nurse is performing a review of systems on a 76-year-old patient. Which of these statements is correct for this situation? a.The questions asked are identical for all ages. b.The interviewer will start incorporating different questions for patients 70 years of age and older. c.Questions that are reflective of the normal effects of aging are added. d.At this age, a review of systems is not necessary—the focus should be on current problems.

C

The nurse is performing a well-child checkup on a 5-year-old boy. He has no current condition that would lead the nurse to suspect an illness. His health history is unremarkable, and he received immunizations 1 week ago. Which of these findings should be considered normal in this patient? a. Enlarged, warm, and tender nodes b. Lymphadenopathy of the cervical nodes c. Palpable firm, small, shotty, mobile, and nontender lymph nodes d. Firm, rubbery, and large nodes, somewhat fixed to the underlying tissue

C

The nurse is performing an assessment on a 29-year-old woman who visits the clinic complaining of "always dropping things and falling down." While testing rapid alternating movements, the nurse notices that the woman is unable to pat both of her knees. Her response is extremely slow and she frequently misses. What should the nurse suspect? a. Vestibular disease b. Lesion of CN IX c. Dysfunction of the cerebellum d. Inability to understand directions

C

The nurse is performing an assessment on an adult. The adult's vital signs are normal, and capillary refill time is 5 seconds. What should the nurse do next? a. Ask the patient about a history of frostbite. b. Suspect that the patient has venous insufficiency. c. Consider this a delayed capillary refill time, and investigate further. d. Consider this a normal capillary refill time that requires no further assessment.

C

The nurse is preparing to measure the length, weight, chest, and head circumference of a 6-month-old infant. Which measurement technique is correct? A) Measure the infant's length by using a tape measure. B) Weigh the infant by placing him on an electronic standing scale. C) Measure chest circumference at the nipple line with a tape measure. D) Measure the head circumference by wrapping the tape measure over the nose and cheekbones.

C

The nurse is preparing to perform a modified Allen test. Which is an appropriate reason for this test? a. To measure the rate of lymphatic drainage b. To evaluate the adequacy of capillary patency before venous blood draws c. To evaluate the adequacy of collateral circulation before cannulating the radial artery d. To evaluate the venous refill rate that occurs after the ulnar and radial arteries are temporarily occluded

C

The nurse is unable to identify any changes in sound when percussing over the abdomen of an obese patient. What should the nurse do next? a. Ask the patient to take deep breaths to relax the abdominal musculature. b. Consider this finding as normal, and proceed with the abdominal assessment. c. Increase the amount of strength used when attempting to percuss over the abdomen. d. Decrease the amount of strength used when attempting to percuss over the abdomen.

C

The nurse is unable to palpate the right radial pulse on a patient. The best action would be to: a. Auscultate over the area with a fetoscope. b. Use a goniometer to measure the pulsations. c. Use a Doppler device to check for pulsations over the area. d. Check for the presence of pulsations with a stethoscope.

C

The nurse knows that which statement is true regarding the pain experienced by infants? A) Pain in infants can only be assessed by physiologic changes, such as increased heart rate. B) The Faces Pain Scale—revised (FPS-R) can be used to assess pain in infants. C) A procedure that induces pain in adults will also induce pain in the infant. D) Infants feel pain less than adults do.

C

The nurse recognizes that the concept of prevention in describing health is essential because: a. Disease can be prevented by treating the external environment. b. The majority of deaths among Americans under age 65 years are not preventable. c. Prevention places the emphasis on the link between health and personal behavior. d. The means to prevention is through treatment provided by primary health care practitioners.

C

The nurse should measure rectal temperatures in which of these patients? A) School-age child B) Elderly adult C) Comatose adult D) Patient receiving oxygen by nasal cannula

C

What type of blood pressure measurement error is most likely to occur if the nurse does not check for the presence of an auscultatory gap? A) The diastolic blood pressure may not be heard. B) The diastolic blood pressure may be falsely low. C) The systolic blood pressure may be falsely low. D) The systolic blood pressure may be falsely high.

C

When assessing the force, or strength, of a pulse, the nurse recalls that it: A) is usually recorded on a 0- to 2-point scale. B) demonstrates elasticity of the vessel wall. C) is a reflection of the heart's stroke volume. D) reflects the blood volume in the arteries during diastole.

C

When considering priority setting of problems, the nurse keeps in mind that second-level priority problems include which of these aspects? a. Low self-esteem b. Lack of knowledge c. Abnormal laboratory values d. Severely abnormal vital signs

C

When considering the concepts related to blood pressure, the nurse knows that the concept of mean arterial pressure (MAP) is best described by which statement? A) MAP is the pressure of the arterial pulse. B) MAP reflects the stroke volume of the heart. C) It is the pressure forcing blood into the tissues, averaged over the cardiac cycle. D) It is an average of the systolic and diastolic blood pressures and reflects tissue perfusion.

C

When evaluating a patient's pain, the nurse knows that an example of acute pain would be: A) arthritic pain. B) fibromyalgia. C) kidney stones. D) low back pain.

C

When listening to a patients breath sounds, the nurse is unsure of a sound that is heard. The nurses next action should be to: a. Immediately notify the patients physician. b. Document the sound exactly as it was heard. c. Validate the data by asking a coworker to listen to the breath sounds. d. Assess again in 20 minutes to note whether the sound is still present.

C

When measuring a patient's body temperature, the nurse keeps in mind that body temperature is influenced by: A) constipation. B) patient's emotional state. C) the diurnal cycle. D) the nocturnal cycle.

C

When taking the health history on a patient with a seizure disorder, the nurse assesses whether the patient has an aura. Which of these would be the best question for obtaining this information? a. "Does your muscle tone seem tense or limp?" b. "After the seizure, do you spend a lot of time sleeping?" c. "Do you have any warning sign before your seizure starts?" d. "Do you experience any color change or incontinence during the seizure?"

C

When the nurse asks for a description of who lives with a child, the method of discipline, and the support system of the child, what part of the assessment is being performed? a.Family history b.Review of systems c.Functional assessment d.Reason for seeking care

C

When the nurse is testing the triceps reflex, what is the expected response? a. Flexion of the hand b. Pronation of the hand c. Extension of the forearm d. Flexion of the forearm

C

Which of these interventions is most appropriate when the nurse is planning nutritional interventions for a healthy, active 74-year-old woman? A) Decrease the amount of carbohydrates to prevent lean muscle catabolism. B) Increase the amount of soy and tofu in her diet to promote bone growth and reverse osteoporosis. C) Decrease the number of calories she is eating because of the decrease in energy requirements from loss of lean body mass. D) Increase the number of calories she is eating because of the increased energy needs of the elderly.

C

Which of these interventions is most appropriate when the nurse is planning nutritional interventions for a healthy, active 74-year-old woman? a. Decreasing the amount of carbohydrates to prevent lean muscle catabolism b. Increasing the amount of soy and tofu in her diet to promote bone growth and reverse osteoporosis c. Decreasing the number of calories she is eating because of the decrease in energy requirements from the loss of lean body mass d. Increasing the number of calories she is eating because of the increased energy needs of the older adult

C

Which of these statements represents subjective data the nurse obtained from the patient regarding the patient's skin? a.Skin appears dry. b.No lesions are obvious. c.Patient denies any color change. d.Lesion is noted on the lateral aspect of the right arm.

C

Which of these would be formulated by a nurse using diagnostic reasoning? a. Nursing diagnosis b. Medical diagnosis c. Diagnostic hypothesis d. Diagnostic assessment

C

While gathering equipment after an injection, a nurse accidentally received a prick from an improperly capped needle. To interpret this sensation, which of these areas must be intact? a. Corticospinal tract, medulla, and basal ganglia b. Pyramidal tract, hypothalamus, and sensory cortex c. Lateral spinothalamic tract, thalamus, and sensory cortex d. Anterior spinothalamic tract, basal ganglia, and sensory cortex

C

The nurse notices that an infant has a large, soft lump on the side of his head and that his mother is very concerned. She tells the nurse that she noticed the lump approximately 8 hours after her baby's birth and that it seems to be getting bigger. One possible explanation for this is: a.Hydrocephalus. b.Craniosynostosis. c.Cephalhematoma. d.Caput succedaneum.

C A cephalhematoma is a subperiosteal hemorrhage that is the result of birth trauma. It is soft, fluctuant, and well defined over one cranial bone. It appears several hours after birth and gradually increases in size.

During an admission assessment, the nurse notices that a male patient has an enlarged and rather thick skull. The nurse suspects acromegaly and would further assess for: a.Exophthalmos. b.Bowed long bones. c.Coarse facial features. d.Acorn-shaped cranium.

C Acromegaly is excessive secretion of growth hormone that creates an enlarged skull and thickened cranial bones. Patients will have elongated heads, massive faces, prominent noses and lower jaws, heavy eyebrow ridges, and coarse facial features. Exophthalmos is associated with hyperthyroidism. Bowed long bones and an acorn-shaped cranium result from Paget disease.

During an examination of a female patient, the nurse notes lymphadenopathy and suspects an acute infection. Acutely infected lymph nodes would be: a.Clumped. b.Unilateral. c.Firm but freely movable. d.Firm and nontender.

C Acutely infected lymph nodes are bilateral, enlarged, warm, tender, and firm but freely movable. Unilaterally enlarged nodes that are firm and nontender may indicate cancer.

A mother brings in her newborn infant for an assessment and tells the nurse that she has noticed that whenever her newborn's head is turned to the right side, she straightens out the arm and leg on the same side and flexes the opposite arm and leg. After observing this on examination, the nurse tells her that this reflex is: a.Abnormal and is called the atonic neck reflex. b.Normal and should disappear by the first year of life. c.Normal and is called the tonic neck reflex, which should disappear between 3 and 4 months of age. d.Abnormal. The baby should be flexing the arm and leg on the right side of his body when the head is turned to the right.

C By 2 weeks, the infant shows the tonic neck reflex when supine and the head is turned to one side (extension of same arm and leg, flexion of opposite arm and leg). The tonic neck reflex disappears between 3 and 4 months of age.

A visitor from Poland who does not speak English seems to be somewhat apprehensive about the nurse examining his neck. He would probably be more comfortable with the nurse examining his thyroid gland from: a.Behind with the nurse's hands placed firmly around his neck. b.The side with the nurse's eyes averted toward the ceiling and thumbs on his neck. c.The front with the nurse's thumbs placed on either side of his trachea and his head tilted forward. d.The front with the nurse's thumbs placed on either side of his trachea and his head tilted backward.

C Examining this patient's thyroid gland from the back may be unsettling for him. It would be best to examine his thyroid gland using the anterior approach, asking him to tip his head forward and to the right and then to the left.

During a well-baby checkup, the nurse notices that a 1-week-old infant's face looks small compared with his cranium, which seems enlarged. On further examination, the nurse also notices dilated scalp veins and downcast or "setting sun" eyes. The nurse suspects which condition? a.Craniotabes b.Microcephaly c.Hydrocephalus d.Caput succedaneum

C Hydrocephalus occurs with the obstruction of drainage of cerebrospinal fluid that results in excessive accumulation, increasing intracranial pressure, and an enlargement of the head. The face looks small, compared with the enlarged cranium, and dilated scalp veins and downcast or "setting sun" eyes are noted. Craniotabes is a softening of the skull's outer layer. Microcephaly is an abnormally small head. A caput succedaneum is edematous swelling and ecchymosis of the presenting part of the head caused by birth trauma.

A patient's thyroid gland is enlarged, and the nurse is preparing to auscultate the thyroid gland for the presence of a bruit. A bruit is a __________ sound that is heard best with the __________ of the stethoscope. a.Low gurgling; diaphragm b.Loud, whooshing, blowing; bell c.Soft, whooshing, pulsatile; bell d.High-pitched tinkling; diaphragm

C If the thyroid gland is enlarged, then the nurse should auscultate it for the presence of a bruit, which is a soft, pulsatile, whooshing, blowing sound heard best with the bell of the stethoscope

A woman comes to the clinic and states, "I've been sick for so long! My eyes have gotten so puffy, and my eyebrows and hair have become coarse and dry." The nurse will assess for other signs and symptoms of: a.Cachexia. b.Parkinson syndrome. c.Myxedema. d.Scleroderma.

C Myxedema (hypothyroidism) is a deficiency of thyroid hormone that, when severe, causes a nonpitting edema or myxedema. The patient will have a puffy edematous face, especially around the eyes (periorbital edema); coarse facial features; dry skin; and dry, coarse hair and eyebrows.

A physician tells the nurse that a patient's vertebra prominens is tender and asks the nurse to reevaluate the area in 1 hour. The area of the body the nurse will assess is: a.Just above the diaphragm. b.Just lateral to the knee cap. c.At the level of the C7 vertebra. d.At the level of the T11 vertebra.

C The C7 vertebra has a long spinous process, called the vertebra prominens, which is palpable when the head is flexed.

A patient, an 85-year-old woman, is complaining about the fact that the bones in her face have become more noticeable. What explanation should the nurse give her? a.Diets low in protein and high in carbohydrates may cause enhanced facial bones. b.Bones can become more noticeable if the person does not use a dermatologically approved moisturizer. c.More noticeable facial bones are probably due to a combination of factors related to aging, such as decreased elasticity, subcutaneous fat, and moisture in her skin. d.Facial skin becomes more elastic with age. This increased elasticity causes the skin to be more taught, drawing attention to the facial bones.

C The facial bones and orbits appear more prominent in the aging adult, and the facial skin sags, which is attributable to decreased elasticity, decreased subcutaneous fat, and decreased moisture in the skin.

The nurse needs to palpate the temporomandibular joint for crepitation. This joint is located just below the temporal artery and anterior to the: a.Hyoid bone. b.Vagus nerve. c.Tragus. d.Mandible.

C The temporomandibular joint is just below the temporal artery and anterior to the tragus.

During an examination of a patient in her third trimester of pregnancy, the nurse notices that the patient's thyroid gland is slightly enlarged. No enlargement had been previously noticed. The nurse suspects that the patient: a.Has an iodine deficiency. b.Is exhibiting early signs of goiter. c.Is exhibiting a normal enlargement of the thyroid gland during pregnancy. d.Needs further testing for possible thyroid cancer.

C The thyroid gland enlarges slightly during pregnancy because of hyperplasia of the tissue and increased vascularity

During an examination, the nurse finds that a patient's left temporal artery is tortuous and feels hardened and tender, compared with the right temporal artery. The nurse suspects which condition? a.Crepitation b.Mastoiditis c.Temporal arteritis d.Bell palsy

C With temporal arteritis, the artery appears more tortuous and feels hardened and tender. These assessment findings are not consistent with the other responses.

A pregnant woman who has breast implants asks the nurse if she can still breastfeed. What is the nurse's best response? A. "You should not have any problems breast feeding because your implants do not affect milk production." B. "When the breast implants are inserted they usually affect the milk glands, and breastfeeding is not possible." C. "This would depend on which type of implants were placed and which procedure was used by the surgeon. Check with your surgeon to see if your milk production will be affected."

C Milk production may vary depending on the procedure. Many women can still breastfeed after breast augmentation. The surgeon would be able to determine this ability. Answers 1 and 2 are incorrect because they may not be true. Only the surgeon can determine if milk production will be affected.

9. A mother asks when her newborn infant's eyesight will be developed. The nurse should reply: a. "Vision is not totally developed until 2 years of age." b. "Infants develop the ability to focus on an object at approximately 8 months of age." c. "By approximately 3 months of age, infants develop more coordinated eye movements and can fixate on an object." d. "Most infants have uncoordinated eye movements for the first year of life."

C. "By approximately 3 months of age, infants develop more coordinated eye movements and can fixate on an object." Eye movements may be poorly coordinated at birth, but by 3 to 4 months of age, the infant should establish binocularity and should be able to fixate simultaneously on a single image with both eyes.

The nurse notices the presence of periorbital edema when performing an eye assessment on a 70-year-old patient. The nurse should: a. Check for the presence of exophthalmos. b. Suspect that the patient has hyperthyroidism. c. Ask the patient if he or she has a history of heart failure. d. Assess for blepharitis, which is often associated with periorbital edema.

C. Ask the patient if he or she has a history of heart failure. Periorbital edema occurs with local infections, crying, and systemic conditions such as heart failure, renal failure, allergy, and hypothyroidism. Periorbital edema is not associated with blepharitis.

A patient has been shown to have a sensorineural hearing loss. During the assessment, it would be important for the nurse to: a. Speak loudly so the patient can hear the questions. b. Assess for middle ear infection as a possible cause. c. Ask the patient what medications he is currently taking. d. Look for the source of the obstruction in the external ear.

C. Ask the patient what medications he is currently taking. A simple increase in amplitude may not enable the person to understand spoken words. Sensorineural hearing loss may be caused by presbycusis, which is a gradual nerve degeneration that occurs with aging and by ototoxic drugs, which affect the hair cells in the

During an assessment, the nurse notices that an older adult patient has tears rolling down his face from his left eye. Closer examination shows that the lower lid is loose and rolling outward. The patient complains of his eye feeling "dry and itchy." Which action by the nurse is correct? a. Assessing the eye for a possible foreign body b. Documenting the finding as ptosis c. Assessing for other signs of ectropion d. Contacting the prescriber; these are signs of basal cell carcinoma

C. Assessing for other signs of ectropion The condition described is known as ectropion, and it occurs in older adults and is attributable to atrophy of the elastic and fibrous tissues. The lower lid does not approximate to the eyeball, and, as a result, the puncta cannot effectively siphon tears; excessive tearing results. Ptosis is a drooping of the upper eyelid. These signs do not suggest the presence of a foreign body in the eye or basal cell carcinoma.

A patient with a middle ear infection asks the nurse, "What does the middle ear do?" The nurse responds by telling the patient that the middle ear functions to: a. Maintain balance. b. Interpret sounds as they enter the ear. c. Conduct vibrations of sounds to the inner ear. d. Increase amplitude of sound for the inner ear to function.

C. Conduct vibrations of sounds to the inner ear. Among its other functions, the middle ear conducts sound vibrations from the outer ear to the central hearing apparatus in the inner ear. The other responses are not functions of the middle

In using the ophthalmoscope to assess a patient's eyes, the nurse notices a red glow in the patient's pupils. On the basis of this finding, the nurse would: a. Suspect that an opacity is present in the lens or cornea. b. Check the light source of the ophthalmoscope to verify that it is functioning. c. Consider the red glow a normal reflection of the ophthalmoscope light off the inner retina. d. Continue with the ophthalmoscopic examination, and refer the patient for further evaluation.

C. Consider the red glow a normal reflection of the ophthalmoscope light off the inner retina. The red glow filling the person's pupil is the red reflex and is a normal finding caused by the reflection of the ophthalmoscope light off the inner retina. The other responses are not correct.

The nurse is performing an assessment on a 65-year-old man. He reports a crusty nodule behind the pinna. It intermittently bleeds and has not healed over the past 6 months. On physical assessment, the nurse finds an ulcerated crusted nodule with an indurated base. The preliminary analysis in this situation is that this: a. Is most likely a benign sebaceous cyst. b. Is most likely a keloid. c. Could be a potential carcinoma, and the patient should be referred for a biopsy. d. Is a tophus, which is common in the older adult and is a sign of gout.

C. Could be a potential carcinoma, and the patient should be referred for a biopsy. An ulcerated crusted nodule with an indurated base that fails to heal is characteristic of a carcinoma. These lesions fail to heal and intermittently bleed. Individuals with such symptoms should be referred for a biopsy (see Table 15-2). The other responses are not correct.

During an assessment of a 20-year-old Asian patient, the nurse notices that he has dry, flaky cerumen in his canal. What is the significance of this finding? This finding: a. Is probably the result of lesions from eczema in his ear. b. Represents poor hygiene. c. Is a normal finding, and no further follow-up is necessary. d. Could be indicative of change in cilia; the nurse should assess for hearing loss

C. Is a normal finding, and no further follow-up is necessary. Asians and Native Americans are more likely to have dry cerumen, whereas Blacks and Whites usually have wet cerumen.

A patient comes into the clinic complaining of facial pain, fever, and malaise. On examination, the nurse notes swollen turbinates and purulent discharge from the nose. The patient also complains of a dull, throbbing pain in his cheeks and teeth on the right side and pain when the nurse palpates the areas. The nurse recognizes that this patient has: a.Posterior epistaxis. b.Frontal sinusitis. c.Maxillary sinusitis. d.Nasal polyps.

C. Maxillary sinusitis

A 70-year-old patient tells the nurse that he has noticed that he is having trouble hearing, especially in large groups. He says that he "can't always tell where the sound is coming from" and the words often sound "mixed up." What might the nurse suspect as the cause for this change? a. Atrophy of the apocrine glands b. Cilia becoming coarse and stiff c. Nerve degeneration in the inner ear d. Scarring of the tympanic membrane

C. Nerve degeneration in the inner ear Presbycusis is a type of hearing loss that occurs in 60% of those older than 65 years of age, even in those living in a quiet environment. This sensorineural loss is gradual and caused by nerve degeneration in the inner ear. Words sound garbled, and the ability to localize sound is also impaired. This communication dysfunction is accentuated when background noise is present.

A patient has been admitted after an accident at work. During the assessment, the patient is having trouble hearing and states, "I don't know what the matter is. All of a sudden, I can't hear you out of my left ear!" What should the nurse do next? a. Make note of this finding for the report to the next shift. b. Prepare to remove cerumen from the patient's ear. c. Notify the patient's health care provider. d. Irrigate the ear with rubbing alcohol.

C. Notify the patient's health care provider. Any sudden loss of hearing in one or both ears that is not associated with an upper respiratory infection needs to be reported at once to the patient's health care provider. Hearing loss associated with trauma is often sudden. Irrigating the ear or removing cerumen is not appropriate at this time.

A 60-year-old man is at the clinic for an eye examination. The nurse suspects that he has ptosis of one eye. How should the nurse check for this? a. Perform the confrontation test. b. Assess the individual's near vision. c. Observe the distance between the palpebral fissures. d. Perform the corneal light test, and look for symmetry of the light reflex.

C. Observe the distance between the palpebral fissures. Ptosis is a drooping of the upper eyelid that would be apparent by observing the distance between the upper and lower eyelids. The confrontation test measures peripheral vision. Measuring near vision or the corneal light test does not check for ptosis.

When a light is directed across the iris of a patient's eye from the temporal side, the nurse is assessing for: a. Drainage from dacryocystitis. b. Presence of conjunctivitis over the iris. c. Presence of shadows, which may indicate glaucoma. d. Scattered light reflex, which may be indicative of cataracts.

C. Presence of shadows, which may indicate glaucoma. The presence of shadows in the anterior chamber may be a sign of acute angle-closure glaucoma. The normal iris is flat and creates no shadows. This method is not correct for the assessment of dacryocystitis, conjunctivitis, or cataracts.

The nurse is performing an otoscopic examination on an adult. Which of these actions is correct? a. Tilting the person's head forward during the examination b. Once the speculum is in the ear, releasing the traction c. Pulling the pinna up and back before inserting the speculum d. Using the smallest speculum to decrease the amount of discomfort

C. Pulling the pinna up and back before inserting the speculum The pinna is pulled up and back on an adult or older child, which helps straighten the S-shape of the canal. Traction should not be released on the ear until the examination is completed and the otoscope is removed.

In a patient who has anisocoria, the nurse would expect to observe: a. Dilated pupils. b. Excessive tearing. c. Pupils of unequal size. d. Uneven curvature of the lens.

C. Pupils of unequal size. Unequal pupil size is termed anisocoria. It normally exists in 5% of the population but may also be indicative of central nervous system disease.

When assessing the pupillary light reflex, the nurse should use which technique? a. Shine a penlight from directly in front of the patient, and inspect for pupillary constriction. b. Ask the patient to follow the penlight in eight directions, and observe for bilateral pupil constriction. c. Shine a light across the pupil from the side, and observe for direct and consensual pupillary constriction. d. Ask the patient to focus on a distant object. Then ask the patient to follow the penlight to approximately 7 cm from the nose.

C. Shine a light across the pupil from the side, and observe for direct and consensual pupillary constriction. To test the pupillary light reflex, the nurse should advance a light in from the side and note the direct and consensual pupillary constriction.

The nurse is assessing color vision of a male child. Which statement is correct? The nurse should: a. Check color vision annually until the age of 18 years. b. Ask the child to identify the color of his or her clothing. c. Test for color vision once between the ages of 4 and 8 years. d. Begin color vision screening at the child's 2-year checkup.

C. Test for color vision once between the ages of 4 and 8 years. Test boys only once for color vision between the ages of 4 and 8 years. Color vision is not tested in girls because it is rare in girls. Testing is performed with the Ishihara test, which is a series of polychromatic cards.

The nurse is preparing to perform an otoscopic examination of a newborn infant. Which statement is true regarding this examination? a. Immobility of the drum is a normal finding. b. An injected membrane would indicate an infection. c. The normal membrane may appear thick and opaque. d. The appearance of the membrane is identical to that of an adult.

C. The normal membrane may appear thick and opaque. During the first few days after the birth, the tympanic membrane of a newborn often appears thickened and opaque. It may look injected and have a mild redness from increased vascularity. The other statements are not correct.

During an examination, the patient states he is hearing a buzzing sound and says that it is "driving me crazy!" The nurse recognizes that this symptom indicates: a. Vertigo. b. Pruritus. c. Tinnitus. d. Cholesteatoma.

C. Tinnitus. Tinnitus is a sound that comes from within a person; it can be a ringing, crackling, or buzzing sound. It accompanies some hearing or ear disorders

The nurse is preparing to assess the visual acuity of a 16-year-old patient. How should the nurse proceed? a. Perform the confrontation test. b. Ask the patient to read the print on a handheld Jaeger card. c. Use the Snellen chart positioned 20 feet away from the patient. d. Determine the patient's ability to read newsprint at a distance of 12 to 14 inches.

C. Use the Snellen chart positioned 20 feet away from the patient. The Snellen alphabet chart is the most commonly used and most accurate measure of visual acuity. The confrontation test is a gross measure of peripheral vision. The Jaeger card or newspaper tests are used to test near vision.

When examining the nares of a 45-year-old patient who has complaints of rhinorrhea, itching of the nose and eyes, and sneezing, the nurse notices the following: pale turbinates, swelling of the turbinates, and clear rhinorrhea. Which of these conditions is most likely the cause? a.Nasal polyps b.Acute sinusitis c.Allergic rhinitis d.Acute rhinitis

C. allergic rhinitis

The nurse notices that the mother of a 2-year-old boy brings him into the clinic quite frequently for various injuries and suspects there may be some child abuse involved. During an inspection of his mouth, the nurse should look for: a.Swollen, red tonsils. b.Ulcerations on the hard palate. c.Bruising on the buccal mucosa or gums. d.Small yellow papules along the hard palate.

C. bruising on the buccal mucosa or gums

When examining a patient's eyes, the nurse recalls that stimulation of the sympathetic branch of the autonomic nervous system: a. Causes pupillary constriction. b. Adjusts the eye for near vision. c. Elevates the eyelid and dilates the pupil. d. Causes contraction of the ciliary body.

C. elevates the eyelid and dilates the pupil Stimulation of the sympathetic branch of the autonomic nervous system dilates the pupil and elevates the eyelid. Parasympathetic nervous system stimulation causes the pupil to constrict. The muscle fibers of the iris contract the pupil in bright light to accommodate for near vision. The ciliary body controls the thickness of the lens.

The nurse is assessing a 3 year old for "drainage from the nose." On assessment, a purulent drainage that has a very foul odor is noted from the left naris and no drainage is observed from the right naris. The child is afebrile with no other symptoms. What should the nurse do next? a.Refer to the physician for an antibiotic order. b.Have the mother bring the child back in 1 week. c.Perform an otoscopic examination of the left nares. d.Tell the mother that this drainage is normal for a child of this age.

C. perform an otoscopic examination of the left nares

During an oral assessment of a 30-year-old Black patient, the nurse notices bluish lips and a dark line along the gingival margin. What action would the nurse perform in response to this finding? a.Check the patient's hemoglobin for anemia. b.Assess for other signs of insufficient oxygen supply. c.Proceed with the assessment, knowing that this appearance is a normal finding. d.Ask if he has been exposed to an excessive amount of carbon monoxide.

C. proceed with the assessment, knowing that this appearance is a normal finding

Immediately after birth, the nurse is unable to suction the nares of a newborn. An attempt is made to pass a catheter through both nasal cavities with no success. What should the nurse do next? a.Attempt to suction again with a bulb syringe. b.Wait a few minutes, and try again once the infant stops crying. c.Recognize that this situation requires immediate intervention. d.Contact the physician to schedule an appointment for the infant at his or her next hospital visit.

C. recognize that this situation requires immediate intervention

A patient has been diagnosed with strep throat. The nurse is aware that without treatment, which complication may occur? a.Rubella b.Leukoplakia c.Rheumatic fever d.Scarlet fever

C. rheumatic fever

During an assessment of a 26 year old at the clinic for "a spot on my lip I think is cancer," the nurse notices a group of clear vesicles with an erythematous base around them located at the lip-skin border. The patient mentions that she just returned from Hawaii. What would be the most appropriate response by the nurse? a.Tell the patient she needs to see a skin specialist. b.Discuss the benefits of having a biopsy performed on any unusual lesion. c.Tell the patient that these vesicles are indicative of herpes simplex I or cold sores and that they will heal in 4 to 10 days. d.Tell the patient that these vesicles are most likely the result of a riboflavin deficiency and discuss nutrition.

C. tell the patient that these vesicles are indicative of herpes simplex 1 or cold sores and that they will heal in 4 to 10 days.

The nurse is examining a patient's ears and notices cerumen in the external canal. Which of these statements about cerumen is correct? a. Sticky honey-colored cerumen is a sign of infection. b. The presence of cerumen is indicative of poor hygiene. c. The purpose of cerumen is to protect and lubricate the ear. d. Cerumen is necessary for transmitting sound through the auditory canal.

C. the purpose of cerumen is to protect and lubricate the ear The ear is lined with glands that secrete cerumen, which is a yellow waxy material that lubricates and protects the ear.

A mother is concerned because her 18-month-old toddler has 12 teeth. She is wondering if this is normal for a child of this age. The nurse's best response would be: a."How many teeth did you have at this age?" b."All 20 deciduous teeth are expected to erupt by age 4 years." c."This is a normal number of teeth for an 18 month old." d."Normally, by age 2 years, 16 deciduous teeth are expected."

C. this is a normal number of teeth for an 18 month old

A 10 year old is at the clinic for "a sore throat that has lasted 6 days." Which of these findings would be consistent with an acute infection? a.Tonsils 1+/1-4+ and pink; the same color as the oral mucosa b.Tonsils 2+/1-4+ with small plugs of white debris c.Tonsils 3+/1-4+ with large white spots d.Tonsils 3+/1-4+ with pale coloring

C. tonsils 3+/1-4+ with large white spots

The projections in the nasal cavity that increase the surface area are called the: a.Meatus. b.Septum. c.Turbinates. d.Kiesselbach plexus.

C. turbinates

A 65-year-old man with emphysema and bronchitis has come to the clinic for a follow-up appointment. On assessment, the nurse might expect to see which assessment finding? a. Anasarca b. Scleroderma c. Pedal erythema d. Clubbing of the nails

Clubbing of the nails Clubbing of the nails occurs with congenital cyanotic heart disease, neoplastic, and pulmonary diseases. The other responses are assessment findings not associated with pulmonary diseases.

1. Consider the below scenario phone conversation when answering the following three questions: "Dr. Jones, this is Mary Smith, RN, on the postsurgical unit at City Hospital. I'm calling about Tom King, your 46-year-old patient who had an inguinal hernia repair this morning. He has not voided since surgery, 8 hours ago. He has received 1900 mL Lactated Ringers IV and 720 mL oral fluids. He can't initiate a stream, but states that he "feels the need to urinate." His bladder is distended by palpation and shows a volume of 800 mL when scanned with the bladder scanner. We've tried standing him to void, providing privacy, and running water, but he is still unable to go. He appears to have urinary retention and I'd like to try using a straight catheter to relieve his retention, what do you think?" In the above scenario, what part of the SBAR communication tool is the underlined information? a. S b. B c. A d. R

D

A 29-year-old woman tells the nurse that she has "excruciating pain" in her back. Which would be the nurse's appropriate response to the woman's statement? a."How does your family react to your pain?" b."The pain must be terrible. You probably pinched a nerve." c."I've had back pain myself, and it can be excruciating." d."How would you say the pain affects your ability to do your daily activities?"

D

A 32-year-old woman tells the nurse that she has noticed "very sudden, jerky movements" mainly in her hands and arms. She says, "They seem to come and go, primarily when I am trying to do something. I haven't noticed them when I'm sleeping." This description suggests: a. Tics. b. Athetosis. c. Myoclonus. d. Chorea.

D

A 42-year-old patient of Asian descent is being seen at the clinic for an initial examination. The nurse knows that including cultural information in his health assessment is important to: a. Identify the cause of his illness. b. Make accurate disease diagnoses. c. Provide cultural health rights for the individual. d. Provide culturally sensitive and appropriate care.

D

A 50-year-old woman is in the clinic for weakness in her left arm and leg that she has noticed for the past week. The nurse should perform which type of neurologic examination? a. Glasgow Coma Scale b. Neurologic recheck examination c. Screening neurologic examination d. Complete neurologic examination

D

A 60-year-old woman has developed reflexive sympathetic dystrophy after arthroscopic repair of her shoulder. A key feature of this condition is that: A) the affected extremity will eventually regain its function. B) the pain is felt at one site but originates from another location. C) her pain will be associated with nausea, pallor, and diaphoresis. D) the slightest touch, such as a sleeve brushing against her arm, causes severe, intense pain.

D

A 63-year-old Chinese-American man enters the hospital with complaints of chest pain, shortness of breath, and palpitations. Which statement most accurately reflects the nurse's best course of action? a. The nurse should focus on performing a full cardiac assessment. b. The nurse should focus on psychosomatic complaints because the patient has just learned that his wife has cancer. c. This patient is not in any danger at present; therefore, the nurse should send him home with instructions to contact his physician. d. It is unclear what is happening with this patient; consequently, the nurse should perform an assessment in both the physical and the psychosocial realms.

D

A 70-year-old patient is scheduled for open-heart surgery. The surgeon plans to use the great saphenous vein for the coronary bypass grafts. The patient asks, "What happens to my circulation when this vein is removed?" The nurse should reply: a. "Venous insufficiency is a common problem after this type of surgery." b. "Oh, you have lots of veins—you won't even notice that it has been removed." c. "You will probably experience decreased circulation after the vein is removed." d. "This vein can be removed without harming your circulation because the deeper veins in your leg are in good condition."

D

A 70-year-old woman tells the nurse that every time she gets up in the morning or after she's been sitting, she gets "really dizzy" and feels like she is going to fall over. The nurse's best response would be: a. "Have you been extremely tired lately?" b. "You probably just need to drink more liquids." c. "I'll refer you for a complete neurologic examination." d. "You need to get up slowly when you've been lying down or sitting."

D

A 90-year-old patient tells the nurse that he cannot remember the names of the medications he is taking or for what reason he is taking them. An appropriate response from the nurse would be: a."Can you tell me what they look like?" b."Don't worry about it. You are only taking two medications." c."How long have you been taking each of the pills?" d."Would you have a family member bring in your medications?"

D

A patient has been diagnosed with venous stasis. Which of these findings would the nurse most likely observe? a. Unilateral cool foot b. Thin, shiny, atrophic skin c. Pallor of the toes and cyanosis of the nail beds d. Brownish discoloration to the skin of the lower leg

D

A patient is asked to indicate on a form how many times he eats a specific food. This method describes which of these tools for obtaining dietary information? a. Food diary b. Calorie count c. 24-hour recall d. Food-frequency questionnaire

D

After a symptom is recognized, the first effort at treatment is often self-care. Which of the following statements about self-care is true? "Self-care is: a. Not recognized as valuable by most health care providers." b. Usually ineffective and may delay more effective treatment." c. Always less expensive than biomedical alternatives." d. Influenced by the accessibility of over-the-counter medicines."

D

An examiner is using an ophthalmoscope to examine a patients eyes. The patient has astigmatism and is nearsighted. The use of which of these techniques would indicate that the examination is being correctly performed? a. Using the large full circle of light when assessing pupils that are not dilated b. Rotating the lens selector dial to the black numbers to compensate for astigmatism c. Using the grid on the lens aperture dial to visualize the external structures of the eye d. Rotating the lens selector dial to bring the object into focus

D

An older adult patient in a nursing home has been receiving tube feedings for several months. During an oral examination, the nurse notes that patient's gums are swollen, ulcerated, and bleeding in some areas. The nurse suspects that the patient has what condition? a. Rickets b. Vitamin A deficiency c. Linoleic-acid deficiency d. Vitamin C deficiency

D

Before auscultating the abdomen for the presence of bowel sounds on a patient, the nurse should: a. Warm the endpiece of the stethoscope by placing it in warm water. b. Leave the gown on the patient to ensure that he or she does not get chilled during the examination. c. Ensure that the bell side of the stethoscope is turned to the on position. d. Check the temperature of the room, and offer blankets to the patient if he or she feels cold.

D

During a class on religion and spirituality, the nurse is asked to define spirituality. Which answer is correct? "Spirituality: a. Is a personal search to discover a supreme being." b. Is an organized system of beliefs concerning the cause, nature, and purpose of the universe." c. Is a belief that each person exists forever in some form, such as a belief in reincarnation or the afterlife." d. Arises out of each person's unique life experience and his or her personal effort to find purpose in life."

D

During a class on the aspects of culture, the nurse shares that culture has four basic characteristics. Which statement correctly reflects one of these characteristics? a. Cultures are static and unchanging, despite changes around them. b. Cultures are never specific, which makes them hard to identify. c. Culture is most clearly reflected in a person's language and behavior. d. Culture adapts to specific environmental factors and available natural resources.

D

During a staff meeting, nurses discuss the problems with accessing research studies to incorporate evidence-based clinical decision making into their practice. Which suggestion by the nurse manager would best help these problems? a. Form a committee to conduct research studies. b. Post published research studies on the units bulletin boards. c. Encourage the nurses to visit the library to review studies. d. Teach the nurses how to conduct electronic searches for research studies.

D

During an assessment of a 32-year-old patient with a recent head injury, the nurse notices that the patient responds to pain by extending, adducting, and internally rotating his arms. His palms pronate, and his lower extremities extend with plantar flexion. Which statement concerning these findings is most accurate? This patient's response: a. Indicates a lesion of the cerebral cortex. b. Indicates a completely nonfunctional brainstem. c. Is normal and will go away in 24 to 48 hours. d. Is a very ominous sign and may indicate brainstem injury.

D

During an assessment, the nurse uses the CAGE test. The patient answers "yes" to two of the questions. What could this be indicating? a.The patient is an alcoholic. b.The patient is annoyed at the questions. c.The patient should be thoroughly examined for possible alcohol withdrawal symptoms. d.The nurse should suspect alcohol abuse and continue with a more thorough substance abuse assessment.

D

During the taking of the health history of a 78-year-old man, his wife states that he occasionally has problems with short-term memory loss and confusion: "He can't even remember how to button his shirt." When assessing his sensory system, which action by the nurse is most appropriate? a. The nurse would not test the sensory system as part of the examination because the results would not be valid. b. The nurse would perform the tests, knowing that mental status does not affect sensory ability. c. The nurse would proceed with an explanation of each test, making certain that the wife understands. d. Before testing, the nurse would assess the patient's mental status and ability to follow directions.

D

For the first time, the nurse is seeing a patient who has no history of nutrition-related problems. The initial nutritional screening should include which activity? a. Calorie count of nutrients b. Anthropometric measures c. Complete physical examination d. Measurement of weight and weight history

D

In assessing a 70-year-old patient who has had a recent cerebrovascular accident, the nurse notices right-sided weakness. What might the nurse expect to find when testing his reflexes on the right side? a. Lack of reflexes b. Normal reflexes c. Diminished reflexes d. Hyperactive reflexes

D

In obtaining a review of systems on a "healthy" 7-year-old girl, the health care provider knows that it would be important to include the: a.Last glaucoma examination. b.Frequency of breast self-examinations. c.Date of her last electrocardiogram. d.Limitations related to her involvement in sports activities.

D

In recording the childhood illnesses of a patient who denies having had any, which note by the nurse would be most accurate? a.Patient denies usual childhood illnesses. b.Patient states he was a "very healthy" child. c.Patient states his sister had measles, but he didn't. d.Patient denies measles, mumps, rubella, chickenpox, pertussis, and strep throat.

D

In the health promotion model, the focus of the health professional includes: a. Changing the patients perceptions of disease. b. Identifying biomedical model interventions. c. Identifying negative health acts of the consumer. d. Helping the consumer choose a healthier lifestyle.

D

In the hot/cold theory, illnesses are believed to be caused by hot or cold entering the body. Which of these patient conditions is most consistent with a cold condition? a. Patient with diabetes and renal failure b. Teenager with an abscessed tooth c. Child with symptoms of itching and a rash d. Older man with gastrointestinal discomfort

D

The mother of a 16-month-old toddler tells the nurse that her daughter has an earache. What would be an appropriate response? a."Maybe she is just teething." b."I will check her ear for an ear infection." c."Are you sure she is really having pain?" d."Describe what she is doing to indicate she is having pain."

D

The nurse hears bilateral loud, long, and low tones when percussing over the lungs of a 4-year-old child. The nurse should: a. Palpate over the area for increased pain and tenderness. b. Ask the child to take shallow breaths, and percuss over the area again. c. Immediately refer the child because of an increased amount of air in the lungs. d. Consider this finding as normal for a child this age, and proceed with the examination.

D

The nurse in a family practice clinic is reviewing the patients scheduled for appointments. Which of these statements is true regarding routine laboratory testing in the following individuals? A) In pregnancy, no laboratory testing is needed unless problems with the pregnancy are suspected. B) In the elderly, laboratory values regarding cholesterol and triglycerides are the most important because of the risk of disease. C) Routine laboratory testing is not necessary during adolescence, except in cases of illness. D) Laboratory tests for iron and lead levels should be assessed at 9 to 12 months.

D

The nurse is assessing children in a pediatric clinic. Which statement is true regarding measurement of blood pressure in children? A) The blood pressure guidelines for children are based on age. B) Phase II Korotkoff sounds are the best indicator of systolic blood pressure in children. C) Use of Doppler device is recommended for accurate blood pressure measurements until adolescence. D) The disappearance of phase V Korotkoff sounds can be used for the diastolic reading in children.

D

The nurse is conducting a class on priority setting for a group of new graduate nurses. Which is an example of a first-level priority problem? a. Patient with postoperative pain b. Newly diagnosed patient with diabetes who needs diabetic teaching c. Individual with a small laceration on the sole of the foot d. Individual with shortness of breath and respiratory distress

D

The nurse is examining a 2-year-old child and asks, May I listen to your heart now? Which critique of the nurses technique is most accurate? a. Asking questions enhances the childs autonomy b. Asking the child for permission helps develop a sense of trust c. This question is an appropriate statement because children at this age like to have choices d. Children at this age like to say, No. The examiner should not offer a choice when no choice is available

D

The nurse is measuring a patient's frame size. Which of these statements best describes the correct technique for measuring frame size? A) With the patient standing, measure the distance from the top of the head to the back of the heel. B) With the patient in a sitting position, measure the distance from the condyle of the humerus to the clavicle. C) With the patient's right arm extended forward and the elbow extended, measure the distance from fingertips to the condyle of the humerus. D) With the right arm extended forward and the elbow bent, use the calipers to measure the distance between the condyles of the humerus.

D

The nurse is obtaining a health history on an 87-year-old woman. Which of the following areas of questioning would be most useful at this time? a.Obstetric history b.Childhood illnesses c.General health for the past 20 years d.Current health promotion activities

D

The nurse is performing a physical assessment on a newly admitted patient. An example of objective information obtained during the physical assessment includes the: a. Patients history of allergies. b. Patients use of medications at home. c. Last menstrual period 1 month ago. d. 2 5 cm scar on the right lower forearm.

D

The nurse is preparing to assess a patients abdomen by palpation. How should the nurse proceed? a. Palpation of reportedly tender areas are avoided because palpation in these areas may cause pain. b. Palpating a tender area is quickly performed to avoid any discomfort that the patient may experience. c. The assessment begins with deep palpation, while encouraging the patient to relax and to take deep breaths. d. The assessment begins with light palpation to detect surface characteristics and to accustom the patient to being touched.

D

The nurse is preparing to assess the ankle-brachial index (ABI) of a patient. Which statement about the ABI is true? a. Normal ABI indices are from 0.5 to 1.0. b. Normal ankle pressure is slightly lower than the brachial pressure. c. The ABI is a reliable measurement of peripheral vascular disease in individuals with diabetes. d. An ABI of 0.9 to 0.7 indicates the presence of peripheral vascular disease and mild claudication.

D

The nurse is preparing to assess the dorsalis pedis artery. Where is the correct location for palpation? a. Behind the knee b. Over the lateral malleolus c. In the groove behind the medial malleolus d. Lateral to the extensor tendon of the great toe

D

The nurse is preparing to complete a health assessment on a 16-year-old girl whose parents have brought her to the clinic. Which instruction would be appropriate for the parents before the interview begins? a."Please stay during the interview; you can answer for her if she does not know the answer." b."It would help to interview the three of you together." c."While I interview your daughter, will you please stay in the room and complete these family health history questionnaires?" d."While I interview your daughter, will you step out to the waiting room and complete these family health history questionnaires?"

D

The nurse is preparing to conduct a health history. Which of these statements best describes the purpose of a health history? a.To provide an opportunity for interaction between the patient and the nurse b.To provide a form for obtaining the patient's biographic information c.To document the normal and abnormal findings of a physical assessment d.To provide a database of subjective information about the patient's past and current health

D

The nurse is preparing to do a functional assessment. Which statement best describes the purpose of a functional assessment? a.The functional assessment assesses how the individual is coping with life at home. b.It determines how children are meeting developmental milestones. c.The functional assessment can identify any problems with memory the individual may be experiencing. d.It helps determine how a person is managing day-to-day activities.

D

The nurse is preparing to measure fat and lean body mass and bone mineral density. Which tool is appropriate? A) Measuring tape B) Skin fold calipers C) Bioelectrical impedance analysis D) Dual-energy x-ray absorptiometry

D

The nurse is preparing to measure fat and lean body mass and bone mineral density. Which tool is appropriate? a. Measuring tape b. Skinfold calipers c. Bioelectrical impedance analysis (BIA) d. Dual-energy x-ray absorptiometry (DEXA)

D

The nurse is preparing to perform a physical assessment. The correct action by the nurse is reflected by which statement? The nurse: a. Performs the examination from the left side of the bed. b. Examines tender or painful areas first to help relieve the patients anxiety. c. Follows the same examination sequence, regardless of the patients age or condition. d. Organizes the assessment to ensure that the patient does not change positions too often.

D

The nurse is preparing to use an otoscope for an examination. Which statement is true regarding the otoscope? The otoscope: a. Is often used to direct light onto the sinuses. b. Uses a short, broad speculum to help visualize the ear. c. Is used to examine the structures of the internal ear. d. Directs light into the ear canal and onto the tympanic membrane.

D

The nurse is reviewing laboratory studies on a patient who may have protein malnutrition. Which of these measurements is an early indicator of protein malnutrition? A) Serum albumin B) Serum creatinine C) Nitrogen balance D) Serum transferrin

D

The nurse is reviewing principles of pain. Which type of pain is due to an abnormal processing of the pain impulse through the peripheral or central nervous system? A) Visceral B) Referred C) Cutaneous D) Neuropathic

D

The nurse is reviewing the hot/cold theory of health and illness. Which statement best describes the basic tenets of this theory? a. The causation of illness is based on supernatural forces that influence the humors of the body. b. Herbs and medicines are classified on their physical characteristics of hot and cold and the humors of the body. c. The four humors of the body consist of blood, yellow bile, spiritual connectedness, and social aspects of the individual. d. The treatment of disease consists of adding or subtracting cold, heat, dryness, or wetness to restore the balance of the humors of the body.

D

The nurse is testing the function of CN XI. Which statement best describes the response the nurse should expect if this nerve is intact? The patient: a. Demonstrates the ability to hear normal conversation. b. Sticks out the tongue midline without tremors or deviation. c. Follows an object with his or her eyes without nystagmus or strabismus. d. Moves the head and shoulders against resistance with equal strength.

D

The nurse manager is explaining culturally competent care during a staff meeting. Which statement accurately describes the concept of culturally competent care? "The caregiver: a. Is able to speak the patient's native language." b. Possesses some basic knowledge of the patient's cultural background." c. Applies the proper background knowledge of a patient's cultural background to provide the best possible health care." d. Understands and attends to the total context of the patient's situation."

D

The nurse recognizes that categories such as ethnicity, gender, and religion illustrate the concept of: a. Family. b. Cultures. c. Spirituality. d. Subcultures.

D

The nursing process is a sequential method of problem solving that nurses use and includes which steps? a. Assessment, treatment, planning, evaluation, discharge, and follow-up b. Admission, assessment, diagnosis, treatment, and discharge planning c. Admission, diagnosis, treatment, evaluation, and discharge planning d. Assessment, diagnosis, outcome identification, planning, implementation, and evaluation

D

To assess the muscle mass and fat stores on a 40-year-old woman, the nurse would use: A) triceps skinfold. B) mid-thigh muscle area. C) percent ideal body weight. D) mid-upper arm circumference.

D

What step of the nursing process includes data collection by health history, physical examination, and interview? a. Planning b. Diagnosis c. Evaluation d. Assessment

D

When assessing a 75-year-old patient who has asthma, the nurse notes that he assumes a tripod position, leaning forward with arms braced on the chair. On the basis of this observation, the nurse should: A) assume that the patient is eager and interested in participating in the interview. B) evaluate the patient for abdominal pain, which may be exacerbated in the sitting position. C) assume that the patient is having difficulty breathing and assist him to a supine position. D) recognize that a tripod position is often used when a patient is having respiratory difficulties.

D

When assessing a patient's nutritional status, the nurse recalls that the best definition of optimal nutritional status is sufficient nutrients that: a. Are in excess of daily body requirements. b. Provide for the minimum body needs. c. Provide for daily body requirements but do not support increased metabolic demands. d. Provide for daily body requirements and support increased metabolic demands.

D

When assessing a patient's pulse, the nurse notes that the amplitude is weaker during inspiration and stronger during expiration. When the nurse measures the blood pressure, the reading decreases 20 mm Hg during inspiration and increases with expiration. This patient is experiencing pulsus: a. Alternans. b. Bisferiens. c. Bigeminus. d. Paradoxus.

D

When auscultating over a patient's femoral arteries, the nurse notices the presence of a bruit on the left side. The nurse knows that bruits: a. Are often associated with venous disease. b. Occur in the presence of lymphadenopathy. c. In the femoral arteries are caused by hypermetabolic states. d. Occur with turbulent blood flow, indicating partial occlusion.

D

When discussing the use of the term subculture, the nurse recognizes that it is best described as: a. Fitting as many people into the majority culture as possible. b. Defining small groups of people who do not want to be identified with the larger culture. c. Singling out groups of people who suffer differential and unequal treatment as a result of cultural variations. d. Identifying fairly large groups of people with shared characteristics that are not common to all members of a culture.

D

When examining a 16-year-old male teenager, the nurse should: a. Discuss health teaching with the parent because the teen is unlikely to be interested in promoting wellness. b. Ask his parent to stay in the room during the history and physical examination to answer any questions and to alleviate his anxiety. c. Talk to him the same manner as one would talk to a younger child because a teens level of understanding may not match his or her speech. d. Provide feedback that his body is developing normally, and discuss the wide variation among teenagers on the rate of growth and development.

D

When measuring a patient's weight, the nurse keeps in mind which of these guidelines? A) Always weigh the patient with only his or her undergarments on. B) It does not matter what type of scale is used, as long as the weights are similar from day to day. C) The patient may leave on his or her jacket and shoes as long as this is documented next to the weight. D) Attempt to weigh the patient at approximately the same time of day, if a sequence of weights is necessary.

D

When performing an assessment of a patient, the nurse notices the presence of an enlarged right epitrochlear lymph node. What should the nurse do next? a. Assess the patient's abdomen, and notice any tenderness. b. Carefully assess the cervical lymph nodes, and check for any enlargement. c. Ask additional health history questions regarding any recent ear infections or sore throats. d. Examine the patient's lower arm and hand, and check for the presence of infection or lesions.

D

When planning a cultural assessment, the nurse should include which component? a. Family history b. Chief complaint c. Medical history d. Health-related beliefs

D

When providing culturally competent care, nurses must incorporate cultural assessments into their health assessments. Which statement is most appropriate to use when initiating an assessment of cultural beliefs with an older American-Indian patient? a. "Are you of the Christian faith?" b. "Do you want to see a medicine man?" c. "How often do you seek help from medical providers?" d. "What cultural or spiritual beliefs are important to you?"

D

When reviewing the concepts of health, the nurse recalls that the components of holistic health include which of these? a. Disease originates from the external environment. b. The individual human is a closed system. c. Nurses are responsible for a patients health state. d. Holistic health views the mind, body, and spirit as interdependent.

D

When the nurse asks a 68-year-old patient to stand with his feet together and arms at his side with his eyes closed, he starts to sway and moves his feet farther apart. The nurse would document this finding as: a. Ataxia. b. Lack of coordination. c. Negative Homans sign. d. Positive Romberg sign.

D

Which of the following reflects the traditional health and illness beliefs and practices of those of African heritage? Health is: a. Being rewarded for good behavior. b. The balance of the body and spirit. c. Maintained by wearing jade amulets. d. Being in harmony with nature.

D

Which of these responses might the nurse expect during a functional assessment of a patient whose leg is in a cast? a."I broke my right leg in a car accident 2 weeks ago." b."The pain is decreasing, but I still need to take acetaminophen." c."I check the color of my toes every evening just like I was taught." d."I'm able to transfer myself from the wheelchair to the bed without help."

D

Which of these statements about the peripheral nervous system is correct? a. The CNs enter the brain through the spinal cord. b. Efferent fibers carry sensory input to the central nervous system through the spinal cord. c. The peripheral nerves are inside the central nervous system and carry impulses through their motor fibers. d. The peripheral nerves carry input to the central nervous system by afferent fibers and away from the central nervous system by efferent fibers.

D

Which situation is most appropriate during which the nurse performs a focused or problem-centered history? a. Patient is admitted to a long-term care facility. b. Patient has a sudden and severe shortness of breath. c. Patient is admitted to the hospital for surgery the following day. d. Patient in an outpatient clinic has cold and influenza-like symptoms.

D

Which statement best describes a proficient nurse? A proficient nurse is one who: a. Has little experience with a specified population and uses rules to guide performance. b. Has an intuitive grasp of a clinical situation and quickly identifies the accurate solution. c. Sees actions in the context of daily plans for patients. d. Understands a patient situation as a whole rather than a list of tasks and recognizes the long-term goals for the patient.

D

Which vein(s) is(are) responsible for most of the venous return in the arm? a. Deep b. Ulnar c. Subclavian d. Superficial

D

While measuring a patient's blood pressure, the nurse recalls that certain factors help to determine blood pressure, such as: A) pulse rate. B) pulse pressure. C) vascular output. D) peripheral vascular resistance.

D

While the nurse is taking the history of a 68-year-old patient who sustained a head injury 3 days earlier, he tells the nurse that he is on a cruise ship and is 30 years old. The nurse knows that this finding is indicative of a(n): a. Great sense of humor. b. Uncooperative behavior. c. Inability to understand questions. d. Decreased level of consciousness.

D

A mother brings her 2-month-old daughter in for an examination and says, "My daughter rolled over against the wall, and now I have noticed that she has this spot that is soft on the top of her head. Is something terribly wrong?" The nurse's best response would be: a."Perhaps that could be a result of your dietary intake during pregnancy." b."Your baby may have craniosynostosis, a disease of the sutures of the brain." c."That 'soft spot' may be an indication of cretinism or congenital hypothyroidism." d."That 'soft spot' is normal, and actually allows for growth of the brain during the first year of your baby's life."

D Membrane-covered "soft spots" allow for growth of the brain during the first year of life. They gradually ossify; the triangular-shaped posterior fontanel is closed by 1 to 2 months, and the diamond-shaped anterior fontanel closes between 9 months and 2 years.

A patient complains that while studying for an examination he began to notice a severe headache in the frontotemporal area of his head that is throbbing and is somewhat relieved when he lies down. He tells the nurse that his mother also had these headaches. The nurse suspects that he may be suffering from: a.Hypertension. b.Cluster headaches. c.Tension headaches. d.Migraine headaches

D Migraine headaches tend to be supraorbital, retroorbital, or frontotemporal with a throbbing quality. They are severe in quality and are relieved by lying down. Migraines are associated with a family history of migraine headaches.

The nurse is aware that the four areas in the body where lymph nodes are accessible are the: a.Head, breasts, groin, and abdomen. b.Arms, breasts, inguinal area, and legs. c.Head and neck, arms, breasts, and axillae. d.Head and neck, arms, inguinal area, and axillae.

D Nodes are located throughout the body, but they are accessible to examination only in four areas: head and neck, arms, inguinal region, and axillae.

The nurse notices that a patient's submental lymph nodes are enlarged. In an effort to identify the cause of the node enlargement, the nurse would assess the patient's: a.Infraclavicular area. b.Supraclavicular area. c.Area distal to the enlarged node. d.Area proximal to the enlarged node.

D When nodes are abnormal, the nurse should check the area into which they drain for the source of the problem. The area proximal (upstream) to the location of the abnormal node should be explored.

A patient visits the clinic because he has recently noticed that the left side of his mouth is paralyzed. He states that he cannot raise his eyebrow or whistle. The nurse suspects that he has: a.Cushing syndrome. b.Parkinson disease. c.Bell palsy. d.Experienced a cerebrovascular accident (CVA) or stroke.

D With an upper motor neuron lesion, as with a CVA, the patient will have paralysis of lower facial muscles, but the upper half of the face will not be affected owing to the intact nerve from the unaffected hemisphere. The person is still able to wrinkle the forehead and close the eyes.

. The nurse is assessing the abilities of an older adult. Which activities are considered IADLs? (Select all that apply.) a. Walking b. Toileting c. Feeding oneself d. Preparing a meal e. Grocery shopping f. Balancing a checkbook

D, e, f

The mother of a 2-year-old is concerned because her son has had three ear infections in the past year. What would be an appropriate response by the nurse? a. "It is unusual for a small child to have frequent ear infections unless something else is wrong." b. "We need to check the immune system of your son to determine why he is having so many ear infections." c. "Ear infections are not uncommon in infants and toddlers because they tend to have more cerumen in the external ear." d. "Your son's eustachian tube is shorter and wider than yours because of his age, which allows for infections to develop more easily."

D. "Your son's eustachian tube is shorter and wider than yours because of his age, which allows for infections to develop more easily." The infant's eustachian tube is relatively shorter and wider than the adult's eustachian tube, and its position is more horizontal; consequently, pathogens from the nasopharynx can more easily migrate through to the middle ear. The other responses are not appropriate.

During an examination of the eye, the nurse would expect what normal finding when assessing the lacrimal apparatus? a. Presence of tears along the inner canthus b. Blocked nasolacrimal duct in a newborn infant c. Slight swelling over the upper lid and along the bony orbit if the individual has a cold d. Absence of drainage from the puncta when pressing against the inner orbital rim

D. Absence of drainage from the puncta when pressing against the inner orbital rim No swelling, redness, or drainage from the puncta should be observed when it is pressed. Regurgitation of fluid from the puncta, when pressed, indicates duct blockage. The lacrimal glands are not functional at birth.

The nurse is reviewing causes of increased intraocular pressure. Which of these factors determines intraocular pressure? a. Thickness or bulging of the lens b. Posterior chamber as it accommodates increased fluid c. Contraction of the ciliary body in response to the aqueous within the eye d. Amount of aqueous produced and resistance to its outflow at the angle of the anterior chamber

D. Amount of aqueous produced and resistance to its outflow at the angle of the anterior chamber Intraocular pressure is determined by a balance between the amount of aqueous produced and the resistance to its outflow at the angle of the anterior chamber. The other responses are incorrect.

An assessment of a 23-year-old patient reveals the following: an auricle that is tender and reddish-blue in color with small vesicles. The nurse would need to know additional information that includes which of these? a. Any change in the ability to hear b. Any recent drainage from the ear c. Recent history of trauma to the ear d. Any prolonged exposure to extreme cold

D. Any prolonged exposure to extreme cold Frostbite causes reddish-blue discoloration and swelling of the auricle after exposure to extreme cold. Vesicles or bullae may develop, and the person feels pain and tenderness.

A patient has a normal pupillary light reflex. The nurse recognizes that this reflex indicates that: a. The eyes converge to focus on the light. b. Light is reflected at the same spot in both eyes. c. The eye focuses the image in the center of the pupil. d. Constriction of both pupils occurs in response to bright light.

D. Constriction of both pupils occurs in response to bright light. The pupillary light reflex is the normal constriction of the pupils when bright light shines on the retina. The other responses are not correct.

The nurse is assessing a patient's eyes for the accommodation response and would expect to see which normal finding? a. Dilation of the pupils b. Consensual light reflex c. Conjugate movement of the eyes d. Convergence of the axes of the eyes

D. Convergence of the axes of the eyes The accommodation reaction includes pupillary constriction and convergence of the axes of the eyes. The other responses are not correct.

In an individual with otitis externa, which of these signs would the nurse expect to find on assessment? a. Rhinorrhea b. Periorbital edema c. Pain over the maxillary sinuses d. Enlarged superficial cervical nodes

D. Enlarged superficial cervical nodes The lymphatic drainage of the external ear flows to the parotid, mastoid, and superficial cervical nodes. The signs are severe swelling of the canal, inflammation, and tenderness. Rhinorrhea, periorbital edema, and pain over the maxillary sinuses do not occur with otitis externa.

An ophthalmic examination reveals papilledema. The nurse is aware that this finding indicates: a. Retinal detachment. b. Diabetic retinopathy. c. Acute-angle glaucoma. d. Increased intracranial pressure.

D. Increased intracranial pressure. Papilledema, or choked disk, is a serious sign of increased intracranial pressure, which is caused by a space-occupying mass such as a brain tumor or hematoma. This pressure causes venous stasis in the globe, showing redness, congestion, and elevation of the optic disc, blurred margins, hemorrhages, and absent venous pulsations. Papilledema is not associated with the conditions in the other responses.

The nurse is taking the history of a patient who may have a perforated eardrum. What would be an important question in this situation? a. "Do you ever notice ringing or crackling in your ears?" b. "When was the last time you had your hearing checked?" c. "Have you ever been told that you have any type of hearing loss?" d. Is a normal finding, and no further follow-up is necessary.

D. Is a normal finding, and no further follow-up is necessary. Typically with perforation, ear pain occurs first, stopping with a popping sensation, and then drainage occurs.

The nurse is reviewing the structures of the ear. Which of these statements concerning the eustachian tube is true? a. The eustachian tube is responsible for the production of cerumen. b. It remains open except when swallowing or yawning. c. The eustachian tube allows passage of air between the middle and outer ear. d. It helps equalize air pressure on both sides of the tympanic membrane.

D. It helps equalize air pressure on both sides of the tympanic membrane. The eustachian tube allows an equalization of air pressure on each side of the tympanic membrane so that the membrane does not rupture during, for example, altitude changes in an airplane. The tube is normally closed, but it opens with swallowing or yawning.

A 52-year-old patient describes the presence of occasional floaters or spots moving in front of his eyes. The nurse should: a. Examine the retina to determine the number of floaters. b. Presume the patient has glaucoma and refer him for further testing. c. Consider these to be abnormal findings, and refer him to an ophthalmologist. d. Know that floaters are usually insignificant and are caused by condensed vitreous fibers.

D. Know that floaters are usually insignificant and are caused by condensed vitreous fibers. Floaters are a common sensation with myopia or after middle age and are attributable to condensed vitreous fibers. Floaters or spots are not usually significant, but the acute onset of floaters may occur with retinal detachment.

During an interview, the patient states he has the sensation that "everything around him is spinning." The nurse recognizes that the portion of the ear responsible for this sensation is the: a. Cochlea. b. CN VIII. c. Organ of Corti. d. Labyrinth.

D. Labyrinth. If the labyrinth ever becomes inflamed, then it feeds the wrong information to the brain, creating a staggering gait and a strong, spinning, whirling sensation called vertigo.

The nurse is conducting a child safety class for new mothers. Which factor places young children at risk for ear infections? a. Family history b. Air conditioning c. Excessive cerumen d. Passive cigarette smoke

D. Passive cigarette smoke Exposure to passive and gestational smoke is a risk factor for ear infections in infants and children.

In performing an examination of a 3-year-old child with a suspected ear infection, the nurse would: a. Omit the otoscopic examination if the child has a fever. b. Pull the ear up and back before inserting the speculum. c. Ask the mother to leave the room while examining the child. d. Perform the otoscopic examination at the end of the assessment

D. Perform the otoscopic examination at the end of the assessment In addition to its place in the complete examination, eardrum assessment is mandatory for any infant or child requiring care for an illness or fever. For the infant or young child, the timing of the otoscopic examination is best toward the end of the complete examination.

During an assessment of the sclera of a black patient, the nurse would consider which of these an expected finding? a. Yellow fatty deposits over the cornea b. Pallor near the outer canthus of the lower lid c. Yellow color of the sclera that extends up to the iris d. Presence of small brown macules on the sclera

D. Presence of small brown macules on the sclera Normally in dark-skinned people, small brown macules may be observed in the sclera.

A patient comes into the emergency department after an accident at work. A machine blew dust into his eyes, and he was not wearing safety glasses. The nurse examines his corneas by shining a light from the side across the cornea. What findings would suggest that he has suffered a corneal abrasion? a. Smooth and clear corneas b. Opacity of the lens behind the cornea c. Bleeding from the areas across the cornea d. Shattered look to the light rays reflecting off the cornea

D. Shattered look to the light rays reflecting off the cornea A corneal abrasion causes irregular ridges in reflected light, which produce a shattered appearance to light rays. No opacities should be observed in the cornea. The other responses are not correct.

A patient is unable to read even the largest letters on the Snellen chart. The nurse should take which action next? a. Refer the patient to an ophthalmologist or optometrist for further evaluation. b. Assess whether the patient can count the nurse's fingers when they are placed in front of his or her eyes. c. Ask the patient to put on his or her reading glasses and attempt to read the Snellen chart again. d. Shorten the distance between the patient and the chart until the letters are seen, and record that distance.

D. Shorten the distance between the patient and the chart until the letters are seen, and record that distance. If the person is unable to see even the largest letters when standing 20 feet from the chart, then the nurse should shorten the distance to the chart until the letters are seen, and record that distance (e.g., "10/200"). If visual acuity is even lower, then the nurse should assess whether the person can count fingers when they are spread in front of the eyes or can distinguish light perception from a penlight. If vision is poorer than 20/30, then a referral to an ophthalmologist or optometrist is necessary, but the nurse must first assess the visual acuity.

The nurse is performing an eye-screening clinic at a daycare center. When examining a 2-year-old child, the nurse suspects that the child has a "lazy eye" and should: a. Examine the external structures of the eye. b. Assess visual acuity with the Snellen eye chart. c. Assess the child's visual fields with the confrontation test. d. Test for strabismus by performing the corneal light reflex test.

D. Test for strabismus by performing the corneal light reflex test. Testing for strabismus is done by performing the corneal light reflex test and the cover test. The Snellen eye chart and confrontation test are not used to test for strabismus.

The mother of a 2-year-old toddler is concerned about the upcoming placement of tympanostomy tubes in her son's ears. The nurse would include which of these statements in the teaching plan? a. The tubes are placed in the inner ear. b. The tubes are used in children with sensorineural loss. c. The tubes are permanently inserted during a surgical procedure. d. The purpose of the tubes is to decrease the pressure and allow for drainage.

D. The purpose of the tubes is to decrease the pressure and allow for drainage. Polyethylene tubes are surgically inserted into the eardrum to relieve middle ear pressure and to promote drainage of chronic or recurrent middle ear infections. Tubes spontaneously extrude in 6 months to 1 year.

During an otoscopic examination, the nurse notices an area of black and white dots on the tympanic membrane and the ear canal wall. What does this finding suggest? a. Malignancy b. Viral infection c. Blood in the middle ear d. Yeast or fungal infection

D. Yeast or fungal infection A colony of black or white dots on the drum or canal wall suggests a yeast or fungal infection (otomycosis).

The tissue that connects the tongue to the floor of the mouth is the: a.Uvula. b.Palate. c.Papillae. d.Frenulum.

D. frenulum

A 72-year-old patient has a history of hypertension and chronic lung disease. An important question for the nurse to include in the health history would be: a."Do you use a fluoride supplement?" b."Have you had tonsillitis in the last year?" c."At what age did you get your first tooth?" d."Have you noticed any dryness in your mouth?"

D. have you noticed any dryness in your mouth

The nurse is reviewing the development of the newborn infant. Regarding the sinuses, which statement is true in relation to a newborn infant? a.Sphenoid sinuses are full size at birth. b.Maxillary sinuses reach full size after puberty. c.Frontal sinuses are fairly well developed at birth. d.Maxillary and ethmoid sinuses are the only sinuses present at birth.

D. maxillary and ethmoid sinuses are the only sinuses present at birth

While obtaining a health history from the mother of a 1-year-old child, the nurse notices that the baby has had a bottle in his mouth the entire time. The mother states, "It makes a great pacifier." The best response by the nurse would be: a."You're right. Bottles make very good pacifiers." b."Using a bottle as a pacifier is better for the teeth than thumb-sucking." c."It's okay to use a bottle as long as it contains milk and not juice." d."Prolonged use of a bottle can increase the risk for tooth decay and ear infections."

D. prolonged use of a bottle can increase the risk for tooth decay and ear infections

The nurse is obtaining a health history on a 3-month-old infant. During the interview, the mother states, "I think she is getting her first tooth because she has started drooling a lot." The nurse's best response would be: a."You're right, drooling is usually a sign of the first tooth." b."It would be unusual for a 3 month old to be getting her first tooth." c."This could be the sign of a problem with the salivary glands." d."She is just starting to salivate and hasn't learned to swallow the saliva."

D. she is just starting to salivate and hasn't learned to swallow the saliva

During ocular examinations, the nurse keeps in mind that movement of the extraocular muscles is: a. Decreased in the older adult. b. Impaired in a patient with cataracts. c. Stimulated by cranial nerves (CNs) I and II. d. Stimulated by CNs III, IV, and VI.

D. stimulated by CNs III, IV, and VI Movement of the extraocular muscles is stimulated by three CNs: III, IV, and VI.

A pregnant woman states that she is concerned about her gums because she has noticed they are swollen and have started bleeding. What would be an appropriate response by the nurse? a."Your condition is probably due to a vitamin C deficiency." b."I'm not sure what causes swollen and bleeding gums, but let me know if it's not better in a few weeks." c."You need to make an appointment with your dentist as soon as possible to have this checked." d."Swollen and bleeding gums can be caused by the change in hormonal balance in your system during pregnancy."

D. swollen and bleeding gums can be caused by the change in hormonal balance in your system during pregnancy.

A 32-year-old woman is at the clinic for "little white bumps in my mouth." During the assessment, the nurse notes that she has a 0.5 cm white, nontender papule under her tongue and one on the mucosa of her right cheek. What would the nurse tell the patient? a."These spots indicate an infection such as strep throat." b."These bumps could be indicative of a serious lesion, so I will refer you to a specialist." c."This condition is called leukoplakia and can be caused by chronic irritation such as with smoking." d."These bumps are Fordyce granules, which are sebaceous cysts and are not a serious condition."

D. these bumps are fordyce granules, which are sebaceous cysts and are not a serious condition

A mother brings her 4-month-old infant to the clinic with concerns regarding a small pad in the middle of the upper lip that has been there since 1 month of age. The infant has no health problems. On physical examination, the nurse notices a 0.5-cm, fleshy, elevated area in the middle of the upper lip. No evidence of inflammation or drainage is observed. What would the nurse tell this mother? a."This area of irritation is caused from teething and is nothing to worry about." b."This finding is abnormal and should be evaluated by another health care provider." c."This area of irritation is the result of chronic drooling and should resolve within the next month or two." d."This elevated area is a sucking tubercle caused from the friction of breastfeeding or bottle-feeding and is normal."

D. this elevated area is a sucking tubercle caused from the friction of breastfeeding or bottle-feeding and is normal.

The nurse is performing an assessment. Which of these findings would cause the greatest concern? a.Painful vesicle inside the cheek for 2 days b.Presence of moist, nontender Stensen's ducts c.Stippled gingival margins that snugly adhere to the teeth d.Ulceration on the side of the tongue with rolled edges

D. ulceration on the side of the tongue with rolled edges.

During a checkup, a 22-year-old woman tells the nurse that she uses an over-the-counter nasal spray because of her allergies. She also states that it does not work as well as it used to when she first started using it. The best response by the nurse would be: a."You should never use over-the-counter nasal sprays because of the risk of addiction." b."You should try switching to another brand of medication to prevent this problem." c."Continuing to use this spray is important to keep your allergies under control." d."Using these nasal medications irritates the lining of the nose and may cause rebound swelling."

D. using these nasal medications irritates the lining of the nose and may cause rebound swelling.

A patient tells the nurse that she is having a hard time bringing her hand to her mouth when she eats or tries to brush her teeth. The nurse knows that for her to move her hand to her mouth she must perform which movement?

Flexion

The nurse is checking the range of motion in a patient's knee and knows that the knee is capable of which movement(s)?

Flexion and extension

An imaginary line connecting the highest point on each iliac crest would cross the _____ vertebra.

Fourth lumbar

A patient with pleuritis has been admitted to the hospital and complaints of pain with breathing. What other key assessment finding would the nurse expect to find upon auscultation?

Friction rub

A patient's annual physical examination reveals a lateral curvature of the thoracic and lumbar segments of his spine; however, this curvature disappears with forward bending. The nurse knows that this abnormality of the spine is called:

Functional scoliosis

A patient tells the nurse that "all my life I've been called 'Knock Knees'." The nurse knows that another term for "knock knees" is:

Genu valgum

The nurse is providing patient education for a man who has been diagnosed with a rotator cuff injury. The nurse knowns that rotator cuff injury involves the:

Glenohumeral joint

The nurse is examining the hip area of a patient and palpates a flat depression on the upper, lateral side of the thigh when the patient is standing. The nurse interprets this finding as the:

Greater trochanter

The nurse suspects that a patient has carpal tunnel syndrome and wants to perform the Phalen's test. To perform this test, the nurse should instruct the patient to:

Hold both hands back to back while flexing the wrists 90 degrees for 60 seconds.

The functional unites of the musculoskeletal system are the:

Joints

During auscultation of breath sounds, the nurse should use the stethoscope correctly, in which of the following ways?

Listen to at least one full respirations in each location

During a morning assessment, the nurse notices that the patient's sputum is frothy and pink. Which condition could this finding indicate?

Pulmonary edema

A 70-year-old patient is being seen in the clinic for sever exacerbation of his heart failure. Which of these findings is the nurse most likely to observe in this situation?

Shortness of breath, orthopnea, paroxysmal nocturnal dyspnea, andle adema

A patient who has had rheumatoid arthritis for years comes to the clinic to ask about changes in her fingers. The nurse will assess for signs for what problems?

Swan neck deformities

The nurse is teaching a class on osteoporosis prevention to a group of post-menopausal women. Which of these actions is the best way to prevent or delay bone loss in this group?

Taking calcium and vitamin D supplements

When assessing the respiratory system of a 4-year-old child, which of these findings would the nurse expect?

The presence of bronchovesicular breath sounds in the peripheral lung fields

A patient has been diagnosed with a ganglion cyst over the dorsum of his left wrist. He asks that nurse, "What is this thing?" The nurse's best answer would be, "It is:

a common benign tumor"

When performing a respiratory assessment on a patient, the nurse notices a costal angle of approximately 90 degrees. This characteristic is:

a normal finding in a healthy adult

A teenage patient comes to the emergency department with complaints of an inability to breathe and a sharp pain in the left side of his chest. The assessment findings include cyanosis, tachypnea, tracheal deviation to the right, decreased tactile fremitus on the left, hyperresonance on the left, and decreased breath sounds on the left. The nurse interprets that these assessment findings are consistent with:

a pneumothorax

1. When gathering information relative to a complete health assessment, the nurse should include which in the decision-making process? (Select all that apply.) a. Treat the health assessment as a legal document. b. Use line drawings to explain and record pertinent findings. c. Do not document findings on the computer while the patient is present. d. Gather needed equipment before the start of the health assessment. e. Write down "word for word" what the patient says as evidence of reliable documentation.

a, b, d

to palpate the temporomandibular joint, the nurse's fingers should be places in the depression _______ of the ear.

anterior to the tragus

An adult patient with a history of allergies comes to the clinic complaining of wheezing and difficulty in breathing when working in his yard. The assessment findings include tachypnea, use of accessory neck muscles, prolonged expiration, intercostal retractions, decreased breath sounds, and expiratory wheezes. The nurse interprets that these assessment findings are consistent with:

asthma

The nurse is auscultating the lungs of a patient who had been sleeping and notices short, popping, crackling sounds that stop after a few breaths. The nurse recongnizes that these breath sounds are:

atelectatic crackles, and that they are not pathologic.

1. A hospitalized patient does not require a full neurologic examination during every shift assessment. What is a method of assessing the neurologic status of a patient without performing a full neurological examination? a. Palpate the carotid pulse. b. Offer the patient a glass of water. c. Look at the significant other throughout the examination. d. Assign the nursing assistant to ask the patient questions and report the findings.

b

1. A patient will be ready to be discharged from the hospital soon, and the patient's family members are concerned about whether the patient is able to walk safely outside alone. Which tool or test would be best to assess this? a. Lawton IADL scale b. Timed Up and Go Test c. Katz Index of ADL scale d. Geriatric Depression Test

b

1. During a functional assessment of an older person's home environment, which statement or question by the nurse is most appropriate regarding common environmental hazards? a. "These low toilet seats are safe because they are nearer to the ground in case of falls." b. "Do you have a relative or friend who can help to install grab bars in your shower?" c. "These small rugs are ideal for preventing you from slipping on the hard floor." d. "It would be safer to keep the lighting low in this room to avoid glare in your eyes."

b

1. The nurse is assessing the cranial nerves. To assess cranial nerve XII, what should the nurse ask the patient to do? a. Say "ahh". b. Stick out tongue. c. Smile and then frown. d. Follow the nurses fingers through the six cardinal positions of gaze.

b

1. The nurse is preparing to perform an examination of the eyes. Which test will the nurse conduct to assess the patient's vision? a. Weber test b. Snellen test c. Confrontation test d. Corneal light reflex

b

1. The nurse needs to determine an older adult's competence and maintenance of life skills in order to determine the most suitable living situation for them. What tool should the nurse use for this assessment? a. Katz Index of ADL b. Lawton IADL scale c. Montreal Cognitive Assessment (MoCA) d. Mini-Mental State Examination (MMSE)

b

1. What should the examiner do during auscultation of breath sounds? a. Listen with the bell of the stethoscope. b. Compare sounds on the left and right sides. c. Listen only to the posterior chest for adventitious sounds. d. Instruct the patient to breathe in and out through the nose.

b

1. While conducting a musculoskeletal assessment the nurse stands behind the patient and has the patient bend over and touch his or her toes. What is the nurse assessing? a. Balance b. The spine c. Cervical range of motion d. External rotation of hips

b

A 70-year-old woman who loves to garden has small, flat, brown macules over her arms and hands. She asks, "What causes these liver spots?" The nurse tells her, "They are: a. Signs of decreased hematocrit related to anemia. b. Due to the destruction of melanin in your skin from exposure to the sun. c. Clusters of melanocytes that appear after extensive sun exposure. d. Areas of hyperpigmentation related to decreased perfusion and vasoconstriction.

clusters of melanocytes that appear after extensive sun exposure." Liver spots, or senile lentigines, are clusters of melanocytes that appear on the forearms and dorsa of the hands after extensive sun exposure. The other responses are not correct.

A physician has diagnosed a patient with purpura. After leaving the room, a nursing student asks the nurse what the physician saw that led to that diagnosis. The nurse should say, "The physician is referring to: a. Blue dilation of blood vessels in a star-shaped linear pattern on the legs. b. Fiery red, star-shaped marking on the cheek that has a solid circular center. c. Confluent and extensive patch of petechiae and ecchymoses on the feet. d. Tiny areas of hemorrhage that are less than 2 mm, round, discrete, and dark red in color.

confluent and extensive patch of petechiae and ecchymoses on the feet." Purpura is a confluent and extensive patch of petechiae and ecchymoses and a flat macular hemorrhage seen in generalized disorders such as thrombocytopenia and scurvy. The blue dilation of blood vessels in a star-shaped linear pattern on the legs describes a venous lake. The fiery red, star-shaped marking on the cheek that has a solid circular center describes a spider or star angioma. The tiny little areas of hemorrhage that are less than 2 mm, round, discrete, and dark red in color describes petechiae.

When assessing a patient's lungs, the nurse recalls that the left lung:

consists of two lobes

The nurse educator is preparing an education module for the nursing staff on the dermis layer of skin. Which of these statements would be included in the module? The dermis: a. Contains mostly fat cells. b. Consists mostly of keratin. c. Is replaced every 4 weeks. d. Contains sensory receptors.

contains sensory receptors. The dermis consists mostly of collagen, has resilient elastic tissue that allows the skin to stretch, and contains nerves, sensory receptors, blood vessels, and lymphatics. It is not replaced every 4 weeks.

During palpation of the anterior chest wall, the nurse notices a coarse, crackling sensation over the skin surface. On the basis of these findings, the nurse suspects:

crepitus

The primary muscles of respiration include the:

diaphragm and intercostals

A young swimmer comes to the sports clinic complaining of a very sore shoulder. He was running at the pool, slipped on some wet concrete, and tried to catch himself with his outstretched hand. He landed on his outstretched hand and had not been able to move his shoulder since then. The nurse suspects:

dislocated shoulder

A patient comes to the clinic and tells the nurse that he has been confined to his recliner chair for about 3 days with his feet down and he wants the nurse to evaluate his feet. During the assessment, the nurse might expect to find: a. Pallor b. Coolness c. Distended veins d. Prolonged capillary filling time

distended veins. Keeping the feet in a dependent position causes venous pooling, resulting in redness, warmth, and distended veins. Prolonged elevation would cause pallor and coolness. Immobilization or prolonged inactivity would cause prolonged capillary filling time. See Table 12-1.

The nurse is percussing over the lungs of a patient with pneumonia. The nurse knows that percussion over an area of atelectasis in the lungs would reveal:

dullness

The nurse is examining a patient who tells the nurse, "I sure sweat a lot, especially on my face and feet but it doesn't have an odor." The nurse knows that this could be related to: a. Eccrine glands. b. Apocrine glands. c. Disorder of the stratum corneum. d. Disorder of the stratum germinativum.

eccrine glands. The eccrine glands are coiled tubules that open directly onto the skin surface and produce a dilute saline solution called sweat. Apocrine glands are located mainly in the axillae, anogenital area, nipples, and naval and mix with bacterial flora to produce a characteristic musky body odor. The patient's statement is not related to disorders of the stratum corneum or the stratum germinativum.

A patient comes to the clinic and states that he has noticed that his skin is redder than normal. The nurse understands that this condition is due to hyperemia and knows that it can be caused by: a. Decreased amounts of bilirubin in the blood b. Excess blood in the underlying blood vessels c. Decreased perfusion to the surrounding tissues d. Excess blood in the dilated superficial capillaries

excess blood in the dilated superficial capillaries. Erythema is an intense redness of the skin caused by excess blood (hyperemia) in the dilated superficial capillaries.

Which statement about the apices of the lungs is true? The apices of the lungs:

extend 3 to 4 cm above the inner third of the clavicles

A 13-year old girl is interested in obtaining information about the cause of her acne. The nurse would share with her that acne: a. Is contagious. b. Has no known cause. c. Is caused by increased sebum production. d. Has been found to be related to poor hygiene.

has no known cause. About 70% of teens will have acne, and, although the cause is unknown, it is not caused by poor diet, oily complexion, a contagion, or poor hygiene.

During an examination, the nurse asks a patient to bend forward from the waist and notices that the patient has lateral tilting. When his leg is raised straight up, he complains of a pain going down his buttock into his leg. The nurse suspects:

herniated nucleus pulposus

A woman is leaving on a trip to Hawaii and has come in for a checkup. During the examination the nurse notices that she is diabetic and takes oral hypoglycemic agents. The patient needs to be concerned about which possible effect of her medications? a. Increased possibility of bruising b. Skin sensitivity as a result of exposure to salt water c. Lack of availability of glucose-monitoring supplies d. Importance of sunscreen and avoiding direct sunlight

importance of sunscreen and avoiding direct sunlight Drugs that may increase sunlight sensitivity and give a burn response include sulfonamides, thiazide diuretics, oral hypoglycemic agents, and tetracycline.

The nurse is explaining to a patient that there are "shock absorbers" in his back to cushion the spine and to help it move. The nurse is referring to his:

intervertabral disks

The nurse is reviewing the technique of palpating for tactile fremitus with a new graduate. Which statement by the graduate nurse reflects a correct understanding of tactile fremitus? "Tactile fremitus:

is caused by sounds generated from the larynx.

During a skin assessment, the nurse notices that a Mexican-American patient has skin that is yellowish-brown; however, the skin on the hard and soft palate is pink and the patient's scleras are not yellow. From this finding, the nurse could probably rule out: a. Pallor b. Jaundice c. Cyanosis d. Iron deficiency

jaundice. Jaundice is exhibited by a yellow color, which indicates rising amounts of bilirubin in the blood. It is first noticed in the junction of the hard and soft palate in the mouth and in the scleras.

A mother brings her newborn baby boy in for a check-up she tells the nurse that he doesn't seem to be moving his right arm as much as his left and that he seems to have pain when she lifts him up under the arm. The nurse suspects a fractured clavicle and would observe for:

limited range of motion during the Moro's reflex.

The nurse is examining a 3-month-old infant. While holding the thumbs on the infant's inner mid thighs and the fingers outside on the hips, touching the greater trochanter, the nurse adducts the legs until the nurse's thumbs touch and then abducts the legs until the infant's knees touch the table. the nurse does not notice any "clunking" sounds and is confident to record a:

negative Ortolani's sign

An 85-year-old patient comments during his annual physical that he seems to be getting shorter as he ages. The nurse should explain that decreased height occurs with aging because:

of the shortening of the vertebral column

A mother brings her 3-month-old infant to the clinic for evaluation of a cold. She tells the nurse that he had "a runny nose for a week." When performing the physical assessment, the nurse notes that the child has nasal flaring and sternal and intercostal retractions. The nurses next action should be to:

recognize that these are serious signs and contact the physician

A patient is able to flex his right arm forward without difficulty or pain but is unable to abduct his arm because of pain and muscle spasms; the nurse should suspect:

rotator cuff lesions.

A man has come in to the clinic for a skin assessment because he is afraid he might have skin cancer. During the skin assessment the nurse notices several areas of pigmentation that look greasy, dark, and "stuck on" his skin. Which is the best prediction? He probably has: a. Senile lentigines, which do not become cancerous b. Actinic keratoses, which are precursors to basal cell carcinoma c. Acrochordons, which are precursors to squamous cell carcinoma d. Seborrheic keratoses, which do not become cancerous

seborrheic keratoses, which do not become cancerous. Seborrheic keratoses look like dark, greasy, "stuck-on" lesions that develop mostly on the trunk. These lesions do not become cancerous. Senile lentigines are commonly called liver spots and are not precancerous. Actinic (senile or solar) keratoses are lesions that are red-tan scaly plaques that increase over the years to become raised and roughened. They may have a silvery-white scale adherent to the plaque. They occur on sun-exposed surfaces and are directly related to sun exposure. They are premalignant and may develop into squamous cell carcinoma. Acrochordons are "skin tags" and are not precancerous.

A 42-year-old female patient complains that she has noticed several small, slightly raised, bright red dots on her chest. On examination, the nurse expects that the spots are probably: a. Anasarca. b. Scleroderma. c. Senile angiomas. d. Latent myeloma.

senile angiomas. Cherry (senile) angiomas are small, smooth, slightly raised bright red dots that commonly appear on the trunk in adults over 30 years old.

The nurse is observing the auscultation technique of another nurse. The correct method to use when progressing from one auscultatory site on the thorax to another is _____ comparison.

side-to-side

The nurse is teaching a class o osteoporosis prevention to a group of post-menopausal woman. A participant shows that she needs more instruction when she states, " I will:

start swimming to increase my weight-bearing exercise."

The nurse has completed the musculoskeletal examination of a patient's knee and has found a positive bulge sign. The nurse interprets this finding to indicate.:

swelling from fluid in the suprapatellar pouch.

The nurse knows that auscultation of fine crackles would most likely be noticed in:

the immediate newborn period

Which of these statements is true regarding the vertebra prominens? The vertebra prominens is:

the spinous process of C7

A 75-year-old woman who has a history of diabetes and peripheral vascular disease has been trying to remove a corn on the bottom of her foot with a pair of scissors. The nurse will encourage her to stop trying to remove the corn with scissors because: a. The woman could be at increased risk for infection and lesions because of her chronic disease. b. With her diabetes, she has increased circulation to her foot, and it could cause severe bleeding. c. She is 75 years old and is unable to see; consequently, she places herself at greater risk for self-injury with the scissors. d. With her peripheral vascular disease, her range of motion is limited and she may not be able to reach the corn safely.

the woman could be at increased risk for infection and lesions because of her chronic disease. A personal history of diabetes and peripheral vascular disease increases a person's risk for skin lesions in the feet or ankles. The patient needs to see a professional for assistance with corn removal.

A man who has had gout for several years comes to the clinic with a problem with his toe. On examination, the nurse notices the presence of hard, painless nodules over the great toe; one has burst open with a chalky discharge. This finding is known as:

tophi

A woman who has had rheumatoid arthritis for years is starting to notice that her fingers are drifting to the side. The nurse knows that this condition is commonly referred to as:

ulnar deviation

Because hair for humans is no longer needed for protection from cold or trauma, it is called:

vestigial. Hair is vestigial for humans. It no longer is needed for protection from cold or trauma.


Related study sets

Human Sexuality Today Chapter Twelve

View Set

Darby Ch 55 - Respiratory diseases

View Set

Quiz: Module 11 Security in Network Design

View Set

Ch 8: The Cellular Basis of Reproduction and Inhertitance

View Set

What is science and how does it work

View Set